NCLEX

Réussis tes devoirs et examens dès maintenant avec Quizwiz!

Jomari is diagnosed with hyperosmolar hyperglycemic nonketotic syndrome (HHNS) is stabilized and prepared for discharge. When preparing the client for discharge and home management, which of the following statements indicates that the client understands her condition and how to control it? A. "I can avoid getting sick by not becoming dehydrated and by paying attention to my need to urinate, drink, or eat more than usual." B. "If I experience trembling, weakness, and headache, I should drink a glass of soda that contains sugar." C. "I will have to monitor my blood glucose level closely and notify the physician if it's constantly elevated." D. "If I begin to feel especially hungry and thirsty, I'll eat a snack high in carbohydrates."

"I can avoid getting sick by not becoming dehydrated and by paying attention to my need to urinate, drink, or eat more than usual." Inadequate fluid intake during hyperglycemic episodes often leads to HHNS. By recognizing the signs of hyperglycemia (polyuria, polydipsia, and polyphagia) and increasing fluid intake, the client may prevent HHNS. Option B: Drinking a glass of non-diet soda would be appropriate for hypoglycemia. Option C: A client whose diabetes is controlled with oral antidiabetic agents usually doesn't need to monitor blood glucose levels. Option D: A high carbohydrate diet would exacerbate the client's condition, particularly if fluid intake is low.

Nurse Alexandra teaches a client about elastic stockings. Which of the following statements, if made by the client, indicates to the nurse that the teaching was successful? A. "I will wear the stockings until the physician tells me to remove them." B. "I should wear the stockings even when I am asleep." C. "Every four hours I should remove the stockings for a half hour." D. "I should put on the stockings before getting out of bed in the morning."

"I should put on the stockings before getting out of bed in the morning. Promote venous return by applying external pressure on veins. Option A: The stockings may be removed before going to bed and worn again before getting out of bed. Option B: Wearing stockings while sleeping is unnecessary. The mechanisms by which wearing elastic stockings prevent DVT are prevention of blood stasis by increasing the blood flow volume and decrease of the caliber of venous blood vessels by compression of the lower limbs. Option C: The stockings should be worn the whole day and removed before going to sleep.

Stacy was diagnosed with acute lymphoid leukemia (ALL). She was discharged from the hospital following her chemotherapy treatments. Which statement of Stacy's mother indicated that she understands when she will contact the physician? A. "I should contact the physician if Stacy has difficulty in sleeping". B. "I will call my doctor if Stacy has persistent vomiting and diarrhea". C. "My physician should be called if Stacy is irritable and unhappy". D. "Should Stacy have continued hair loss, I need to call the doctor".

"I will call my doctor if Stacy has persistent vomiting and diarrhea". Persistent (more than 24 hours) vomiting, anorexia, and diarrhea are signs of toxicity and the patient should stop the medication and notify the healthcare provider. Option A: Oftentimes, chemotherapy drugs cause patients to feel tired and sleepy throughout the day. Therefore, patients on chemotherapy can end up napping or sleeping during the day and that leads to difficulty sleeping at night or through the night. Option C: Chemotherapy medications can directly impact the way people feel emotionally and physically. Option D: Chemotherapy drugs are powerful medications that attack rapidly growing cancer cells. Unfortunately, these drugs also attack other rapidly growing cells in the body, including those in the hair roots.

The physician has prescribed NPH insulin for a client with diabetes mellitus. Which statement indicates that the client knows when the peak action of the insulin occurs? A. "I will make sure I eat breakfast within 2 hours of taking my insulin." B. "I will need to carry candy or some form of sugar with me all the time." C. "I will eat a snack around three o'clock each afternoon." D. "I can save my dessert from supper for a bedtime snack."

"I will eat a snack around three o'clock each afternoon." NPH insulin peaks in 8-12 hours, so a snack should be offered at that time. It is on the list of essential medicines of the world health organization (WHO).NPH insulin is FDA-approved in the adult and pediatric population for the control of type 1 and type 2 diabetes mellitus. It is currently the most widely used basal insulin that simulates the physiological basal insulin action. Basal insulin provides a constant supply of insulin in the body that is necessary for glucose regulation despite fasting, such as between meals and overnight. Option A: NPH insulin onsets in 90-120 minutes, so answer A is incorrect. American Diabetes Association guidelines recommend NPH insulin dose of 0.4 to 1.0 units/kg/day subcutaneously for the management of type 1 diabetes mellitus. A higher dose is necessary during medical illness and puberty. A dose of 0.1 to 0.2 units/kg/day subcutaneously is recommended in type 2 diabetes mellitus depending on body weight and the patient's hyperglycemia. Option B: NPH insulin is time-released and does not usually cause sudden hypoglycemia. NPH insulin is an isophane suspension of human insulin and categorized as an intermediate-acting insulin. It helps increase the cellular intake of glucose in the liver, adipose tissue, and skeletal muscles. It acts as basal insulin and stimulates the liver to promote hepatic glycogen synthesis, fatty acid metabolism for lipoprotein synthesis. Option D: The client should eat a bedtime snack. NPH cannot classify as ideal basal insulin. It shows much variability in its absorption and action after subcutaneous injection. This situation leads to fluctuations in blood glucose control and causes hypoglycemic episodes. For this reason, it requires meticulous monitoring, including fasting blood glucose, and changes in HbA1c values.

Patrick who is diagnosed with liver cirrhosis is experiencing symptoms of hepatic encephalopathy. The physician ordered 50 ml of Lactulose p.o. every 2 hours. Patrick suddenly develops diarrhea. The nurse best action would be: A. "I'll see if your physician is in the hospital". B. "Maybe you're reacting to the drug; I will withhold the next dose". C. "I'll lower the dosage as ordered so the drug causes only 2 to 4 stools a day". D. "Frequently, bowel movements are needed to reduce sodium level".

"I'll lower the dosage as ordered so the drug causes only 2 to 4 stools a day". Lactulose is given to a patient with hepatic encephalopathy to reduce absorption of ammonia in the intestines by binding with ammonia and promoting more frequent bowel movements. If the patient experiences diarrhea, it indicates overdosage and the nurse must reduce the amount of medication given to the patient. The stool will be mushy or soft. Lactulose is also very sweet and may cause cramping and bloating. Option A: The nurse may inform the physician so that he may order a lower dosage. Option B: Withholding the next dose is unnecessary; the nurse may lower the dosage as ordered. Option D: Lactulose, in this case, is given to reduce absorption of ammonia in the intestines.

Rico with diabetes mellitus must learn how to self-administer insulin. The physician has prescribed 10 U of U-100 regular insulin and 35 U of U-100 isophane insulin suspension (NPH) to be taken before breakfast. When teaching the client how to select and rotate insulin injection sites, the nurse should provide which instruction? A. "Inject insulin into healthy tissue with large blood vessels and nerves." B. "Rotate injection sites within the same anatomic region, not among different regions." C. "Administer insulin into areas of scar tissue or hypertrophy whenever possible." D. "Administer insulin into sites above muscles that you plan to exercise heavily later that day."

"Rotate injection sites within the same anatomic region, not among different regions." The nurse should instruct the client to rotate injection sites within the same anatomic region. Rotating sites among different regions may cause excessive day-to-day variations in the blood glucose level; also, insulin absorption differs from one region to the next.Option A: Insulin should be injected only into healthy tissue lacking large blood vessels, nerves, or scar tissue, or other deviations.Option C: Injecting insulin into areas of hypertrophy may delay absorption. The client shouldn't inject insulin into areas of lipodystrophy (such as hypertrophy or atrophy); to prevent lipodystrophy, the client should rotate injection sites systematically.Option D: Exercise speeds drug absorption, so the client shouldn't inject insulin into sites above muscles that will be exercised heavily.

The nurse prepares a 5-year-old girl for a pre-operative IV insertion. Which statement is most appropriate to reduce the child's anxiety? "Just look at the TV while I do this." "Hold on to your doll. This is going to hurt." "It's going to hurt a little, but I know you're a brave girl." "Tell me if this feels more like a pinch or a bug bite."

"Tell me if this feels more like a pinch or a bug bite."-CORRECT Like all clients, children should be prepared for procedures. Educate them, but don't suggest that there will be pain. Allow them to decide if there is discomfort. Ignoring an explanation or trying to distract the child is rude and interferes with the child's sense of trust.

Following a client's intentional benzodiazepine overdose, the Emergency Department physician prescribes flumazenil 200 mcg IV push STAT. The stock vial contains 0.5 mg flumazenil in 5 mL solution. How many mL should the nurse draw up and administer? Provide an answer to one decimal place. 4.0 mL 2.5 mL 2.0 mL 0.5 mL

2.0 mL The nurse will draw up and administer 2.0 mL. To calculate: 1. Convert mg to mcg: 0.5 mg x 1000 = 500 mcg. 2. Known: 5 mL = 500 mcg. 3. 1 mL = 100 mcg. 4. 2 mL = 200 mcg. 5. You. can also set up an equation: (5 mL x 200 mcg ÷ 500 mcg = 2.0 mL. Flumazenil, a specific benzodiazepine antagonist, is used as an antidote for intentional benzodiazepine overdose, as well as reversing the sedation and respiratory depression from anesthesia.

Mickey, a 6-year-old child with a congenital heart disorder is admitted with congestive heart failure. Digoxin (lanoxin) 0.12 mg is ordered for the child. The bottle of Lanoxin contains .05 mg of Lanoxin in 1 ml of solution. What amount should the nurse administer to the child? A. 1.2 ml B. 2.4 ml C. 3.5 ml D. 4.2 ml

2.4 ml .05 mg/ 1 ml = .12mg/ x ml, .05x = .12, x = 2.4 ml. Option A: 1.2 ml is less than the correct dosage and may not produce the desired effects of the drug. Option C: 3.5 ml is more than the correct dosage as calculated and may produce adverse effects. Option D: 4.2 ml is an incorrect dosage according to the formula used.

Nurse Michelle calculates the IV flow rate for a postoperative client. The client receives 3,000 ml of Ringer's lactate solution IV to run over 24 hours. The IV infusion set has a drop factor of 10 drops per milliliter. The nurse should regulate the client's IV to deliver how many drops per minute? A. 18 B. 21 C. 35 D. 40

21 3000 x 10 divided by 24 x 60. Option A: 18 is incorrect according to the formula used. Option C: 35 is more than the prescribed gtts/minute as calculated. Option D: 40 is more than the correct gtts/minute as calculated.

An infant who weighs 8 pounds at birth would be expected to weigh how many pounds at 1 year? A. 14 pounds B. 16 pounds C. 18 pounds D. 24 pounds

24 pounds By 1 year of age, the infant is expected to triple his birth weight. Between six months and one year, weight gain slows down a little. Most babies double their birth weight by five to six months of age and triple it by the time they are a year old. By one year, the average weight of a baby girl is approximately 19 pounds 10 ounces (8.9 kg), with boys weighing about 21 pounds 3 ounces (9.6 kg). Option A: During the first few days of life, it's normal for both breastfed and bottle-fed newborns to lose weight. A bottle-fed baby may lose up to 5% of his body weight, and an exclusively breastfed newborn can lose up to 10%. Option B: On average, babies gain about one pound each month for the first six months. The average weight at six months is about 16 pounds 2 ounces (7.3 kg) for girls and 17 pounds 8 ounces (7.9 kg) for boys. Option C: Most infants will gain about a pound over their birth weight by month one. At this age, infants are not as sleepy, they begin developing a regular feeding pattern, and they have a stronger suck during feedings.

All of the following are equivalent to 25% EXCEPT 0.25 1/4 one-quarter 25.0

25.0 A percentage is a number that is expressed as a fraction of 100, or parts per 100. Percentages are used in medicine in several ways, including IV solutions (such as Sodium Chloride 0.9%), injectable medications (such as lidocaine hydrochloride 2%), and calculations of an infant's weight loss or gain.

A nurse is caring for a patient with acute lymphoblastic leukemia (ALL). Which of the following is the most likely age range of the patient? A. 3-10 years. B. 25-35 years. C. 45-55 years. D. over 60 years.

3-10 years. The peak incidence of ALL is at 4 years (range 3-10). It is uncommon after the mid-teen years. It is diagnosed in about 4000 people in the United States each year with the majority being under the age of 18. It is the most common malignancy of childhood. The peak age of diagnosis is between two and ten years of age. Option B: ALL is uncommon during young adulthood. Acute Lymphocytic Leukemia is more common in children with Trisomy 21 (Down syndrome), neurofibromatosis type 1, Bloom syndrome, and ataxia telangiectasia. All are common in children between two and three years of age. Option C: The peak incidence of chronic myelogenous leukemia (CML) is 45-55 years. Prognosis is diminished in children when diagnosed in infants less than one year of age and in adults. It is more favorable for children. Association of the MLL gene in children at 11q23 chromosome is associated with poor prognosis. Option D: The peak incidence of acute myelogenous leukemia (AML) occurs at 60 years. Two-thirds of cases of chronic lymphocytic leukemia (CLL) occur after 60 years. Lymphocytic Leukemia is a disease with low incidence overall in population studies. The incidence of Acute Lymphocytic Leukemia is about 3.3 cases per 100,000 children. Survival rates for ALL have improved dramatically since the 1980s, with a current five-year overall survival rate estimated at greater than 85 percent.

A client with a diagnosis of depression is prescribed phenelzine sulfate (Nardil). When he returns to the clinic a week later, he reports that he doesn't feel any better. The nurse explains that when starting a monoamine oxidase inhibitor (MAOI), the client won't experience relief of symptoms for how long? 2 weeks 3-4 months 3-4 weeks 2 months

3-4 weeks A client should be taught that MAOI inhibitors, such as Nardil, require about a month to achieve therapeutic blood levels and demonstrate relief of symptoms. Clients should also be instructed never to discontinue an MAOI drug abruptly without first consulting with their provider.

After your patient dies, the patient's family gathers at the bedside and asks you to step out while their clergy performs a religious rite for the deceased. As the patient's nurse, what is your most appropriate course of action? 1. Educate the family about custody of care and stay in the room. 2. Allow the ceremony but remain as a witness. 3. Allow the ceremony and step out of the room. 4. Inform the family that religious rites are not allowed.

3. Allow the ceremony and step out of the room. -CORRECT The nurse should honor the family's wishes and culture and leave the room. Most hospitals do not have a policy that prohibits religious rites or ceremonies at the time of death. Remaining in the room shows disrespect and lack of trust at a time of grieving.

The nurse is providing postmortem care for a client who was being treated for Staphylococcus aureus. Which transmission-based precautions are indicated? 1. Airborne precautions 2. Droplet precautions 3. Contact precautions 4. Standard precautions

3. Contact Precautions -CORRECT MRSA is transmitted by contact, and MRSA bacteria remain alive for up to 3 days after the host dies. Therefore, contact precautions must still be used after the client dies, including the use of a gown and gloves. The body and bag should also be labeled as MRSA contaminated so other hospital, transportation, and funeral home employees can protect themselves as well.

A patient taking dilantin (Phenytoin) for a seizure disorder is experiencing breakthrough seizures. A blood sample is taken to determine the serum drug level. Which of the following would indicate a sub-therapeutic level? A. 15 mcg/mL. B. 4 mcg/mL. C. 10 mcg/dL. D. 5 mcg/dL.

4 mcg/mL. The therapeutic serum level for Dilantin is 10 - 20 mcg/mL. A level of 4 mcg/mL is subtherapeutic and may be caused by patient non-compliance or increased metabolism of the drug. Therapeutic drug monitoring of phenytoin is necessary to ensure dosage delivery is at therapeutic levels. Option A: A level of 15 mcg/mL is therapeutic. In therapeutic doses, phenytoin is absorbed entirely and reaches peak plasma concentration at 1.5 to 3 hours. However, in settings of acute ingestions, absorption tends to last longer than two weeks; this is potentially attributable to its effects on reducing the gastrointestinal motility and poor water solubility. Option C: Knowledge of its pharmacokinetic properties is crucial for correct interpretation of total serum concentrations when protein binding becomes altered due to hypoalbuminemia, renal failure, or interaction with other protein-bound drugs such as valproate. Option D: Choices C and D are expressed in mcg/dL, which is the incorrect unit of measurement. Theoretical equations such as the Sheiner-Tozer equation have been introduced to calculate adjusted serum concentration levels and avoid inappropriate adjustment of dosage of phenytoin. However, they have not seen broad implementation in clinical practice due to poor patient outcomes.

If a drug is 50% protein-bound, it means that 50% less protein should be eaten. 50% of the drug is available. 50% will pass through the intestines. 50% of the drug destroys protein.

50% of the drug is available The percentage of the drug that is NOT protein bound is the amount of the drug that is free to work as expected. In this case, 50% of the drug is unable to be effective because it is protein-bound. Protein binding has nothing to do with drug excretion, protein in the diet, or the destruction of protein.

Jose is in danger of respiratory arrest following the administration of a narcotic analgesic. An arterial blood gas value is obtained. Nurse Oliver would expect the paco2 to be which of the following values? A. 15 mm Hg B. 30 mm Hg C. 40 mm Hg D. 80 mm Hg

80 mm Hg A client about to go into respiratory arrest will have inefficient ventilation and will be retaining carbon dioxide. The value expected would be around 80 mm Hg. All other values are lower than expected. Option A: 15 mmHg is a low value for a client about to go into respiratory arrest. Option B: 30 mmHg is lower than the expected value because of inefficient ventilation. Option C: 40 mmHg is still less than the expected value for a client who is about to go into respiratory arrest.

When prioritizing care, which of the following clients should the nurse Olivia assess first? A. A 17-year-old client 24-hours post appendectomy. B. A 33-year-old client with a recent diagnosis of Guillain-Barre syndrome. C. A 50-year-old client 3 days post myocardial infarction. D. A 50-year-old client with diverticulitis.

A 33-year-old client with a recent diagnosis of Guillain-Barre syndrome Guillain-Barre syndrome is characterized by ascending paralysis and potential respiratory failure. The order of client assessment should follow client priorities, with disorder of airways, breathing, and then circulation. Option A: The client who is post appendectomy has no signs of hemorrhage or unstable vital signs. Possible complications of appendectomy are bleeding, wound infection, peritonitis, blocked bowels, and injury to nearby organs. Option C: There's no information to suggest the postmyocardial infarction client has an arrhythmia or other complication. About 90% of patients who have an acute MI develop some form of cardiac arrhythmia during or immediately after the event. Option D: There's no evidence to suggest perforation for the client with diverticulitis as a priority of care. Diverticula are small, bulging pouches that can form in the lining of the digestive system.when one or more of the pouches become inflamed, and in some cases infected, that condition is known as diverticulitis.

Which patient should be assigned to the traveling nurse, new to neurologic nursing care, who has been in the neurologic unit for 1 week? A 34-year-old patient newly diagnosed with multiple sclerosis (MS). A 68-year-old patient with chronic amyotrophic lateral sclerosis (ALS). A 56-year-old patient with Guillain-Barre syndrome (GBS) in respiratory distress. A 25-year-old patient admitted with CA level spinal cord injury (SCI).

A 68-year-old patient with chronic amyotrophic lateral sclerosis (ALS). The traveling nurse is relatively new to neurologic nursing and should be assigned to patients whose conditions are stable and not complex. A) The newly diagnosed patient will need to be transferred to the ICU. MS is an immune-mediated inflammatory disease that attacks myelinated axons in the CNS, destroying the myelin and the axon in variable degrees and producing significant physical disability within 20-25 years in more than 30% of patients. The hallmark of MS is symptomatic episodes that occur months or years apart and affect different anatomic locations. C) The patient with GBS is in respiratory distress and should be assigned to an experienced neurological nurse. GBS is a rare disorder in which a person's own immune system damages their nerve cells, causing muscle weakness and sometimes paralysis. GBS can cause symptoms that usually last for a few weeks. D) The patient with C4 SCI is at risk for respiratory arrest. A C4 SCI occurs when damage is dealt about mid-way down the cervical spinal cord - the topmost portion of the spinal cord that is located in the neck and upper shoulders.

A male client with a gunshot wound requires an emergency blood transfusion. His blood type is AB negative. Which blood type would be the safest for him to receive? A. AB Rh-positive B. A Rh-positive C. A Rh-negative D. O Rh-positive

A Rh-negative Human blood can sometimes contain an inherited D antigen. Persons with the D antigen have Rh-positive blood type; those lacking the antigen have Rh-negative blood. It's important that a person with Rh-negative blood receives Rh-negative blood. Option A: A person with Rh-negative blood should also receive Rh-negative blood. Option B: If Rh-positive blood is administered to an Rh-negative person, the recipient develops anti-Rh agglutinins, and subsequent transfusions with Rh-positive blood may cause serious reactions with clumping and hemolysis of red blood cells. Option D: This blood type is still not compatible because it is Rh-positive.

A child is admitted to the hospital several days after stepping on a sharp object that punctured her athletic shoe and entered the flesh of her foot. The physician is concerned about osteomyelitis and has ordered parenteral antibiotics. Which of the following actions is done immediately before the antibiotic is started? A. The admission orders are written. B. A blood culture is drawn. C. A complete blood count with differential is drawn. D. The parents arrive.

A blood culture is drawn. Antibiotics must be started after the blood culture is drawn, as they may interfere with the identification of the causative organism. Prolonged antibiotic therapy is the cornerstone of treatment for osteomyelitis. Results of culture and sensitivity should guide antibiotic treatment if possible, but in the absence of this data, it is reasonable to start empiric antibiotics. Option A: Physician's order for the antibiotics must be written before administration of the antibiotics. A commonly used broad-spectrum empiric antibiotic regimen against both gram-positive and negative organisms including MRSA is vancomycin (15 mg/kg intravenously [IV] every 12 hours) plus a third a generation cephalosporin (e.g., ceftriaxone 2 gm IV daily) or a beta-lactam/beta-lactamase inhibitor combination (e.g., piperacillin/tazobactam 3.375 IV every 8 hours). Option C: The blood count will reveal the presence of infection but does not help identify an organism or guide antibiotic treatment. Laboratory data can be useful in the assessment of osteomyelitis but are usually nonspecific for osteomyelitis. There may or may not be leukocytosis, elevation of ESR, and C-reactive protein (CRP). The CRP level correlates with clinical response to therapy and may be used to monitor treatment. Blood cultures may be positive, especially in hematogenous osteomyelitis involving the vertebrae, clavicle, or pubis. Option D: Parental presence is important for the adjustment of the child but not for the administration of medication. qualified home-bound patients benefit from home health wound nurse visits several times a week, while those who are ambulatory are encouraged to use wound care clinics if available. Family members and caregivers are also an integral part of the care team, providing support and care between healthcare provider visits. Only through such an approach can the morbidity of osteomyelitis be decreased.

A 12-year-old male is admitted to the hospital several days after stepping on a sharp object that punctured his shoe and penetrated the bottom of his foot. Concerned about possible osteomyelitis, the doctor has ordered parenteral antibiotics. Which of the following is done immediately before the antibiotic is started? The first antibiotic dose is held until the parents are present. A blood culture test is drawn. The admission orders are entered into the system. A complete blood count (CBC) with differential is drawn.

A blood culture test is drawn. Antibiotics must not be started until a blood culture test is drawn, because administering antibiotics may interfere with identifying the bacteria and the appropriate therapy. The blood count will reveal the presence of infection but will not help identify the causative organism or guide antibiotic treatment. Parental presence is important for the adjustment of the child but not for the administration of medication.

A leukemia patient has a relative who wants to donate blood for transfusion. Which of the following donor medical conditions would prevent this? A. A history of hepatitis C five years previously. B. Cholecystitis requiring cholecystectomy one year previously. C. Asymptomatic diverticulosis. D. Crohn's disease in remission.

A history of hepatitis C five years previously. Hepatitis C is a viral infection transmitted through bodily fluids, such as blood, causing inflammation of the liver. Patients with hepatitis C may not donate blood for transfusion due to the high risk of infection in the recipient. Transmission can be parenteral, perinatal, and sexual, with the most common mode being the sharing of contaminated needles among IV drug users. Also, other high-risk groups include people who require frequent blood transfusions and organ transplantation of organs from infected donors. Option B: Cholecystitis is inflammation of the gallbladder that occurs most commonly because of an obstruction of the cystic duct by gallstones arising from the gallbladder (cholelithiasis). Ninety percent of cases of cholecystitis involve stones in the gallbladder (ie, calculous cholecystitis), with the other 10% of cases representing acalculous cholecystitis. Option C: Diverticular disease (diverticulosis, diverticulitis) is a general term that refers to the presence of diverticula, small pouches in the large intestinal (colonic) wall. The cause of diverticulosis is unclear, but it has been associated with increased pressure from constipation or increasing abdominal girth in obesity. The classic high-fat and low-fiber diet of the Western culture may be a major contributor to the development of diverticulosis. Option D: Crohn's disease is an idiopathic, chronic inflammatory process that can affect any part of the gastrointestinal tract from the mouth to the anus. Crohn's disease is believed to be the result of an imbalance between proinflammatory and anti-inflammatory mediators. Although genetic susceptibility, luminal antigenic drive, and environmental triggers are also important factors, animal models demonstrate that no single factor is sufficient to induce intestinal inflammation.

A client has an order for streptokinase. Before administering the medication, the nurse should assess the client for: A. A history of streptococcal infections B. Allergies to pineapples and bananas C. Prior therapy with phenytoin D. A history of alcohol abuse

A history of streptococcal infections Clients with a history of streptococcal infections could have antibodies that render the streptokinase ineffective. Due to streptokinase's thrombolytic mechanisms of action, patients need monitoring for bleeding. The patient's thrombin time, prothrombin time, partial thromboplastin time, complete blood count, and any signs of bleeding demand careful surveillance. Patients also need monitoring for signs and symptoms of reinfarction or vessel occlusion. Option B: There is no reason to assess the client for allergies to pineapples or bananas. Streptokinase can precipitate an allergic reaction. Symptoms include fever, shivering, and rash. Patients in rare instances have developed nonfatal anaphylactic reactions. In cases of anaphylaxis, patients should be administered epinephrine immediately and discontinue streptokinase therapy. The toxicity of streptokinase is believed to be because it is a polypeptide derivative of beta-hemolytic streptococci bacteria. Option C: There is no correlation to the use of phenytoin and streptokinase. The thrombolytic nature of streptokinase makes it contraindicated in patients with active internal bleeding, as it can worsen bleeding in some patients. It is also contraindicated in patients with severe uncontrolled hypertension, intracranial neoplasms, surgery within two months, recent stroke, and intraspinal surgery. Option D: A history of alcohol abuse is also not a factor in the order for streptokinase. Streptokinase administered with extreme caution to those who have experienced recent trauma, coagulopathies or hematologic diseases, gastrointestinal bleeding, infective endocarditis, obstetric deliveries, diabetic hemorrhagic retinopathy, organ biopsies, or previous puncture of a non-compressible vessel. Streptokinase is a pregnancy Category C medication. The effects of streptokinase on a fetus are unknown, and patients who are pregnant should only receive streptokinase to prevent life-threatening injury.

A child is admitted to the hospital with suspected rheumatic fever. Which of the following observations is not confirming the diagnosis? A. A reddened rash visible over the trunk and extremities. B. A history of sore throat that was self-limited in the past month. C. A negative antistreptolysin O titer. D. An unexplained fever.

A negative antistreptolysin O titer. Rheumatic fever is caused by an untreated group A B hemolytic Streptococcus infection in the previous 2-6 weeks, confirmed by a positive antistreptolysin O titer. ASO is a test used to detect streptococcal antibodies directed against streptococcal lysin O. An elevated titer is proof of a previous streptococcal infection. It is usually more elevated after a pharyngeal than skin infection, while the ADB is typically elevated regardless of the site of the infection. Option A: Rheumatic fever is characterized by a red rash over the trunk and extremities. The individual lesions of erythema marginatum are evanescent, moving over the skin in serpiginous patterns. Likened to smoke rings, they have a tendency to advance at the margins while clearing in the center. Option B: Although estimates vary, only 35%-60% of patients with rheumatic fever recall having any upper respiratory symptoms, most commonly, sore throat, in the preceding several weeks. Studies in developed countries have established that rheumatic fever followed only pharyngeal infections and that not all serotypes of group A streptococci cause rheumatic fever. Option D: Other symptoms of rheumatic fever include fever. The average duration of an untreated ARF attack is 3 months. Chronic rheumatic fever, generally defined as disease persisting for longer than 6 months, occurs in less than 5% of cases.

The client with color blindness will most likely have problems distinguishing which of the following colors? A. Orange B. Violet C. Red D. White

Violet Clients with color blindness will most likely have problems distinguishing violets, blues, and green. The most common forms are protanopia and deuteranopia, conditions arising from loss of function of one of the cones, leading to dichromic vision. Protanopia is the loss of L cones (red) resulting in green-blue vision only. Deuteranopia is the loss of M cones (green) resulting in red-blue vision only. Option A: Color vision results from the combination of signals from three visual pigment types within cones: that of red, green, and blue, which correspond to cone types L, M, and S (RGB-LMS). Those colors correspond to the wavelengths of peak light absorption intensities of the modified chromophores. L cones have peak absorptions at 555 nm to 565 nm, M cones at 530 nm to 537 nm, and S cones at 415 nm to 430 nm. Option C: Similar to above, but not as severe in its symptoms, is the condition anomalous trichromatic vision (tritanomaly), where all three cones are present but the color vision is aberrant. The two common forms, protanomaly, and deuteranomaly result in L or M cones, respectively, being replaced with a cone of intermediate spectral tuning. Both are X-linked and occur in 7% of males. Option D: In addition to disorders of proper color recognition, many diseases in vision display phototransduction defects affecting many portions of the signal pathway and its regulation. Here, not only is color vision function lessened but scotopic (low-light, rod-associated) vision as well.

The charge nurse on the cardiac unit is planning assignments for the day. Which of the following is the most appropriate assignment for the float nurse that has been reassigned from labor and delivery? A. A one-week postoperative coronary bypass patient, who is being evaluated for placement of a pacemaker prior to discharge. B. A suspected myocardial infarction patient on telemetry, just admitted from the Emergency Department and scheduled for an angiogram. C. A patient with unstable angina being closely monitored for pain and medication titration. D. A postoperative valve replacement patient who was recently admitted to the unit because all surgical beds were filled.

A one-week postoperative coronary bypass patient, who is being evaluated for placement of a pacemaker prior to discharge. The charge nurse planning assignments must consider the skills of the staff and the needs of the patients. The labor and delivery nurse who is not experienced with the needs of cardiac patients should be assigned to those with the least acute needs. The patient who is one-week post-operative and nearing discharge is likely to require routine care. Option B: A new patient admitted with suspected MI and scheduled for angiography would require continuous assessment as well as coordination of care that is best carried out by experienced staff. Nurse-patient assignments are typically allocated based on estimated direct patient care requirements with little consideration for other activities that must be completed throughout a shift. In an effort to improve upon previous assignment methodologies, new measures and metrics were considered in this study to reduce and balance demands placed on nurses through the assignment of required activities. Option C: The unstable patient requires staff that can immediately identify symptoms and respond appropriately. In most hospitals, a unit charge nurse is responsible for the shift assignment of patients to nurses based on experience and past practices. The nurse-patient assignment process is also often a manual process in which the charge nurse must sort through multiple decision criteria in a limited amount of time. Option D: A postoperative patient also requires close monitoring and cardiac experience. Balancing workload among nurses on a hospital unit is important for the satisfaction and safety of nurses and patients. To balance nurse workloads, direct patient care activities, indirect patient care activities, and non-patient care activities that occur throughout a shift must be considered.

Which patient should not be prescribed alendronate (Fosamax) for osteoporosis? A. A female patient being treated for high blood pressure with an ACE inhibitor. B. A patient who is allergic to iodine/shellfish. C. A patient on a calorie restricted diet. D. A patient on bed rest who must maintain a supine position.

A patient on bed rest who must maintain a supine position. Alendronate can cause significant gastrointestinal side effects, such as esophageal irritation, so it should not be taken if a patient must stay in supine position. It should be taken upon rising in the morning with 8 ounces of water on an empty stomach to increase absorption. The patient should not eat or drink for 30 minutes after administration and should not lie down. Option A: Contraindications to alendronate include patients with known hypersensitivity, esophageal abnormalities, delayed esophageal emptying, or achalasia. Severe risk of esophageal morbidity indicates avoidance in patients who are unable to sit or stand upright for at least 30 minutes. Avoid alendronate in patients with hypocalcemia. Option B: ACE inhibitors are not contraindicated with alendronate and there is no iodine allergy relationship. Baseline concentrations of calcium and bone mineral density should be established before therapy begins, with follow-up testing at 6 to 12 months post-therapy. Calcium at baseline and continual monitoring is needed if hypocalcemia risk is recurring. Option C: There is no restriction for alendronate on a patient taking a calorie restricted diet. The accumulation of alendronate in the kidney allows for persistent anti-fracture benefits even after cessation of treatment. Current recommendations suggest tailoring the drug holiday length to the individual patients. The average drug-holiday in low-risk patients is 3 to 5 years.

A patient with Addison's disease asks a nurse for nutrition and diet advice. Which of the following diet modifications is not recommended? A. A diet high in grains. B. A diet with adequate caloric intake. C. A high protein diet. D. A restricted sodium diet

A restricted sodium diet. A patient with Addison's disease requires normal dietary sodium to prevent excess fluid loss. Do not reduce salt in the diet. The client may need to add extra salt to his food during hot and humid weather or after exercise to replace salt lost through sweating. Do not use salt substitutes. Option A: Addison's disease is a rare condition. It develops when the adrenal glands, which are located above the kidneys, do not make enough of certain hormones. These hormones are important for normal body function. They help the body cope with stress, hold salt and water, and maintain blood pressure. The client should include complex carbohydrates in his diet, including grains. Option B: A high protein diet is recommended for the client with Addison's disease. The adrenal fatigue diet aims to stabilize blood sugar and balance cortisol levels by limiting sugar while increasing the intake of protein, healthy fats, veggies, and whole grains. Healthy fats and high-quality proteins slow the blood sugar rollercoaster and promote stable blood sugar levels throughout the day. Option C: Adequate caloric intake is recommended. Refined carbohydrates quickly break down into sugar after you ingest them, which causes a spike in blood sugar followed by a steep decline. The diet discourages foods that are inflammatory or hard to digest and may contribute to gut health issues. The adrenal fatigue diet is more about eating more foods that make the client feel good and nourish the body versus restricting.

A patient who has been diagnosed with vasospastic disorder (Raynaud's disease) complains of cold and stiffness in the fingers. Which of the following descriptions is most likely to fit the patient? A. An adolescent male B. An elderly woman C. A young woman D. An elderly man

A young woman Raynaud's disease is most common in young women and is frequently associated with rheumatologic disorders, such as lupus and rheumatoid arthritis. Secondary Raynaud phenomenon is associated with different etiologies. It is most commonly associated with connective tissue disorders such as scleroderma, systemic lupus erythematosus, Sjogren syndrome, and antiphospholipid syndrome. Option A: Primary Raynaud phenomenon usually occurs in the second or third decade of life, with a baseline prevalence rate of 8% in men. Occupations that result in overt vibrational exposure from vibrating machinery mostly affect males. This is known as hand-arm vibration syndrome. Exposure to polyvinyl chloride, cold injury from work, or ammunition work are other occupational-associated causes of secondary Raynaud phenomenon. Option B: Primary Raynaud phenomenon usually occurs in the second or third decade of life. Secondary Raynaud phenomenon occur more frequently in women (about 20% to 30%), particularly in younger age populations (teens to 20s). The female to male ratio is 9 to 1. Option D: Primary Raynaud phenomenon occurs more frequently in women than in men. In the population of patients older than 60 years, obstructive vascular disease is a frequent cause of the Raynaud phenomenon. Obstructive vascular disease causes include thromboangiitis obliterans, microemboli, diabetic angiopathy, or atherosclerosis

A toddler has recently been diagnosed with cerebral palsy. Which of the following information should the nurse provide to the parents? Note: More than one answer may be correct. Select all that apply. A. Regular developmental screening is important to avoid secondary developmental delays. B. Cerebral palsy is caused by injury to the upper motor neurons and results in motor dysfunction, as well as possible ocular and speech difficulties. C. Developmental milestones may be slightly delayed but usually will require no additional intervention. D. Parent support groups are helpful for sharing strategies and managing health care issues. E. Outdoor activities are prohibited for the child. E. Outdoor activities are prohibited for the child.

A, B & D. Delayed developmental milestones are characteristic of cerebral palsy, so regular screening and intervention is essential. Because of injury to upper motor neurons, children may have ocular and speech difficulties. Parent support groups help families to share and cope. Option A: Encourage age-related play and other activities that strengthen gross and fine motor development, sensory and cognitive development such as letting the child put green balls on the left basket and red balls on the right. These activities enhance growth and development and provide needed stimulation for the child. Option B: Learn patient needs and pay attention to nonverbal cues. The nurse should set aside enough time to attend to all of the details of patient care. Care measures may take longer to complete in the presence of a communication deficit. Provide an alternative means of communication. Alternative forms such as flashcards, whiteboards, hand signs, or a picture board allow the client to express oneself if speaking is difficult to obtain. Option C: Physical therapy and other interventions can minimize the extent of the delay in developmental milestones. Facilitate activities in using fine and gross motor skills (such as giving a ball on hand to encourage throwing, holding a spoon). Perform range-of-motion exercises every 4 hours for the child unable to move body parts. Children with cerebral palsy have a decreased range of motion (ROM) due to limited mobility and the presence of spasticity. ROM exercises promote movement and minimize the risk of contractures. Option D: Encourage the parent to express the impact of their child's condition on the family. Assess the coping ability of the family. This will determine how much support and guidance the family may need. Educate the family on different skills needed to manage the child's care (such as physical rehabilitation, proper nutrition, medication administration, ROM exercises, seizure management). Option E: Outdoor activities, such as bird watching, playing in the playground, or amusement park rides, are not prohibited for the child with cerebral palsy. Most of the activities of daily living and play exercises hasten physical development. Helps the parents to vent their feelings/concerns.

An older patient asks a nurse to recommend strategies to prevent constipation. Which of the following suggestions would be helpful? Note: More than one answer may be correct. Select all that apply. A. Get moderate exercise for at least 30 minutes each day. B. Drink 6-8 glasses of water each day. C. Eat a diet high in fiber. D. Take a mild laxative if you don't have a bowel movement every day. E. Eat a protein-rich diet.

A, B, & C A daily bowel movement is not necessary if the patient is comfortable and the bowels move regularly. Moderate exercise, such as walking, encourages bowel health, as does generous water intake. A diet high in fiber is also helpful. Check on the usual pattern of elimination, including frequency and consistency of stool. It is very crucial to carefully know what is "normal" for each patient. The normal frequency of stool passage ranges from twice daily to once every third or fourth day. Dry and hard feces are common characteristics of constipation. Option A: Urge patient for some physical activity and exercise. Consider isometric abdominal and glute exercises. Movement promotes peristalsis. Abdominal exercises strengthen abdominal muscles that facilitate defecation. Option B: Encourage the patient to take in fluid 2000 to 3000 mL/day, if not contraindicated medically. Sufficient fluid is needed to keep the fecal mass soft. But take note of some patients or older patients having cardiovascular limitations requiring less fluid intake. Option C: Assist the patient to take at least 20 g of dietary fiber (e.g. raw fruits, fresh vegetable, whole grains) per day. Fiber adds bulk to the stool and makes defecation easier because it passes through the intestine essentially unchanged. Option D: Laxatives should be used as a last resort and should not be taken regularly. Over time, laxatives can desensitize the bowel and worsen constipation. The use of laxatives or enemas is indicated for short-term management of constipation. Option E: Protein-rich foods could cause constipation. A balanced diet that comprises adequate fiber, fresh fruits, vegetables, and grains. Twenty grams of fiber per day is suggested. A regular period for elimination and an adequate time for defection. Successful bowel training relies on routine. Facilitating regular time prevents the bowel from emptying sporadically.

A patient is admitted to the same day surgery unit for liver biopsy. Which of the following laboratory tests assesses coagulation? Select all that apply. A. Partial thromboplastin time B. Prothrombin time C. Platelet count D. Hemoglobin E. Complete Blood Count F. White Blood Cell Count

A, B, & C Prothrombin time, partial thromboplastin time, and platelet count are all included in coagulation studies. Option A: PTT tests the function of all clotting factors except factor VII (tissue factor) and factor XIII (fibrin stabilizing factor). PTT is commonly used in clinical practice to monitor patient response to unfractionated heparin infusion, to target therapeutic anticoagulation, and as part of a "coagulation panel" to help elucidate causes of bleeding or clotting disorders. Option B: Prothrombin time (PT) is one of several blood tests routinely used in clinical practice to evaluate the coagulation status of patients. More specifically, PT is used to evaluate the extrinsic and common pathways of coagulation, which would detect deficiencies of factors II, V, VII, and X, and low fibrinogen concentrations. Option C: Clinicians can monitor the function of platelets by evaluating the bleeding time, which evaluates the time between breaking the vasculature and formation of an effective platelet plug. This time may be elevated in conditions like uremia, in which platelet count is normal but demonstrates impaired function. Option D: A hemoglobin test measures the levels of hemoglobin in the blood. Hemoglobin is a protein in the red blood cells that carries oxygen from the lungs to the rest of the body. Hemoglobin variant testing measures by percentage, the relative hemoglobin types present in erythrocytes. This testing allows for the detection of hemoglobin variants and thalassemic disorders. Option E: A complete blood count (CBC) is a blood test used to evaluate the overall health and detect a wide range of disorders, including anemia, infection and leukemia. A complete blood count test measures several components and features of the blood, including red blood cells, white blood cells, hemoglobin, hematocrit, and platelets. Option F: A white blood cell (WBC) count is a test that measures the number of white blood cells in the body. Of note, if a subtype of white blood cells seems to be elevated based on the differential, the actual value of the type of white blood cells should be calculated by multiplying the percentage listed on the differential by the total number of white blood cells.

A nurse in the emergency department assesses a patient who has been taking long-term corticosteroids to treat renal disease. Which of the following is a typical side effect of corticosteroid treatment? Select all that apply. A. Hypertension B. Cushingoid features C. Hyponatremia D. Low serum albumin E. Hypernatremia

A, B, & D Side effects of corticosteroids include weight gain, fluid retention with hypertension, Cushingoid features, a low serum albumin, and suppressed inflammatory response. Patients are encouraged to eat a diet high in protein, vitamins, and minerals and low in sodium. Option A: Corticosteroid use is associated with hypertension, hyperglycemia, obesity, and conflicting evidence exists for hyperlipidemia. Mineralocorticoid activity, which varies by corticosteroid, leads to retention of free water and sodium with excretion of potassium. Option B: Cushing syndrome can occur in patients taking corticosteroids through all routes of administration. Cushingoid features refer to the weight gain and the redistribution of adiposity (dorsocervical fat pad, aka "buffalo hump," facial fat increase, aka "moon facies," and truncal obesity) seen with excess cortisol. These features may develop within the first two months of corticosteroid treatment. Option C: Corticosteroids cause hypernatremia, not hyponatremia. Corticosteroids have varying degrees of mineralocorticoid activity. The corticosteroids with higher levels of mineralocorticoid activity may lead to free water and salt retention in addition to potassium excretion. Thus, fluid and electrolyte levels should be monitored in patients on corticosteroids with higher mineralocorticoid activity. Option D: There was a significant correlation between the extent of plasma protein binding of prednisolone and the serum albumin concentration. Azathioprine did not affect the plasma binding of prednisolone in vitro. The plasma half-life of prednisolone was prolonged in two of three patients with chronic liver disease studied. These, together with low serum albumin concentrations which are associated with higher levels of circulating unbound prednisolone, result in quite different levels of biologically active corticosteroids compared with equivalent doses of prednisone or prednisolone in subjects without liver disease. Option E: Glucocorticoid can induce hypernatremia not only by enhancing sodium retention, but also by increasing electrolyte-free water loss. It has been shown that patients, as well as experimental animals, exhibit polyuria in the presence of excess glucocorticoid hormones.

A teen patient is admitted to the hospital by his physician who suspects a diagnosis of acute glomerulonephritis. Which of the following findings is consistent with this diagnosis? Select all that apply. A. Urine specific gravity of 1.040. B. Urine output of 350 ml in 24 hours. C. Brown ("tea-colored") urine. D. Generalized edema. E. Periorbital swelling.

A, B, C & E Acute glomerulonephritis is characterized by high urine specific gravity related to oliguria as well as dark "tea-colored" urine caused by large amounts of red blood cells. As the glomerular filtration rate (GFR) is decreased, symptoms like edema and hypertension occur, majorly due to the subsequent salt and water retention caused by the activation of the renin-angiotensin-aldosterone system. Option A: Glomerulonephritis and pyelonephritis cause a decreased urine volume and low specific gravity. In these diseases, damage to the kidney's tubules affects the ability of the kidney to reabsorb water. As a result, the urine remains dilute. Option B: About half of the people with acute glomerulonephritis have no symptoms. If symptoms do occur, the first to appear are tissue swelling (edema) due to fluid retention, low urine volume, and production of urine that is dark because it contains blood. Option C: When kidneys are failing, the increased concentration and accumulation of substances in urine lead to a darker color which may be brown, red or purple. The color change is due to abnormal protein or sugar, high levels of red and white blood cells, and high numbers of tube-shaped particles called cellular casts. Option D: There is periorbital edema, but generalized edema is seen in nephrotic syndrome, not acute glomerulonephritis. Edema may first appear as puffiness of the face and eyelids but later is prominent in the legs. This is reported in approximately 85% of pediatric patients; edema may be mild (involving only the face) to severe, bordering on a nephrotic appearance. Option E: Patients often have a normal physical examination and blood pressure; most frequently, however, patients present with a combination of edema, hypertension, and oliguria. The physician should look for signs of fluid overload, like periorbital and/or pedal edema.

A mother calls the clinic to report that her son has recently started medication to treat attention-deficit/hyperactivity disorder (ADHD). The mother fears her son is experiencing side effects of the medicine. Which of the following side effects are typically related to medications used for ADHD? Select all that apply. A. Poor appetite B. Insomnia C. Sleepiness D. Agitation E. Decreased attention span

A, B, D & E ADHD in children is frequently treated with CNS stimulant medications, which increase focus and improve concentration. Children often experience insomnia, agitation, and decreased appetite. ADHD treatment commonly uses a combination of dextroamphetamine and levoamphetamine, as well as pure dextroamphetamine and lisdexamfetamine. Option A: Loss of appetite is among the most common side effects of stimulants for ADHD. Across studies, approximately 20% of patients with ADHD who were treated with stimulants reported a loss of appetite. Weight loss is also quite common, as are digestive problems. Option B: Insomnia or delayed SOL greater than 30 minutes is one of the most common adverse events associated with stimulant medications. This should be distinguished from bedtime resistance, which is when the child refuses to go to bed. Insomnia is a frequent side effect of all stimulant medications, based on parent report or side effects scales completed side effects scales by parents. Option C: Sleepiness is not a side effect of stimulants. Efron et al. compared twice-daily, immediate-release MPH and dextroamphetamine in 125 ADHD youth in a crossover study. Using the parent-completed, Barkley Side Effect Scale, dextroamphetamine, but not MPH, was associated with higher ratings of severe insomnia relative to baseline. Option D: The immediate psychological effects of stimulant administration include a heightened sense of well-being, euphoria, excitement, heightened alertness, and increases in motor activity. Stimulants also reduce food intake, reduce sleep time, and may increase socialization activities. Stimulants may also enhance performance of certain types of psychomotor tasks. High doses may result in restlessness and agitation, and excessive doses may produce stereotypic behaviors (repetitive and automatic acts). Option E: In people with ADHD, stimulants produce a paradoxical calming effect. This results in a reduction in hyperactivity and an improvement in attention span in many patients.

Claudication is a well-known effect of peripheral vascular disease. Which of the following facts about claudication is correct? Select all that apply. A. It results when oxygen demand is greater than oxygen supply. B. It is characterized by pain that often occurs during rest. C. It is a result of tissue hypoxia. D. It is characterized by cramping and weakness. E. It is relieved after a short rest.

A, C, D, & E. Claudication describes the pain experienced by a patient with peripheral vascular disease when oxygen demand in the leg muscles exceeds the oxygen supply. This most often occurs during activity when demand increases in muscle tissue, and usually relieved after rest. The tissue becomes hypoxic, causing cramping, weakness, and discomfort. Option A: Intermittent claudication (IC) typically refers to lower extremity skeletal muscle pain that occurs during exercise. IC presents when there is insufficient oxygen delivery to meet the metabolic requirements of the skeletal muscles. Option B: Pain occurs during activity when demand increases in muscle tissues, not when at rest. IC is commonly localized to the thigh, hip, buttock, and calf muscles. Pain within these muscle groups is reproducibly induced by walking and relieved with rest. The severity of pain can sometimes correlate with the degree of stenosis or blockage in arteries supplying the lower extremities. Option C: The key feature of intermittent claudication is that the muscle discomfort is reproducible. The pain usually comes on during physical activity and subsides after a period of rest. The key reason for the pain is inadequate blood flow. Option D: Physical examination of these patients may show evidence of arterial insufficiency. The affected limb may feel cool and have diminished pulses. The physical examination should include an assessment of femoral, popliteal, dorsalis pedis, and posterior tibial artery pulses. Option E: Structured walking programs improve pain-free walking distance better than pharmacologic therapy alone. It is important to note that continued smoking with walking therapy restricts improvement in these patients.

A female client is taking Cascara Sagrada. Nurse Betty informs the client that the following may be experienced as side effects of this medication: A. GI bleeding B. Peptic ulcer disease C. Abdominal cramps D. Partial bowel obstruction

Abdominal cramps The most frequent side effects of Cascara Sagrada (Laxative) is abdominal cramps and nausea. Cascara sagrada is possibly safe for most adults when taken for less than one week. Side effects include stomach discomfort and cramps. Option A: There is no GI bleeding associated with Cascara sagrada. Cascara sagrada is possibly unsafe when used for more than one week. This could cause more serious side effects including dehydration; low levels of potassium, sodium, chloride, and other "electrolytes" in the blood; heart problems; muscle weakness; and others. Option B: Peptic ulcer disease is not a side effect of Cascara sagrada, however, stomach discomfort may occur when using this drug. Gastrointestinal (GI) disorders such as intestinal obstruction, Crohn disease, ulcerative colitis, appendicitis, stomach ulcers, or unexplained stomach pain: people with any of these conditions should not use cascara sagrada. Option D: Partial bowel obstruction is not associated with the use of Cascara sagrada. Cascara is a type of laxative called a stimulant laxative. Stimulant laxatives speed up the bowels. Taking cascara along with other stimulant laxatives could speed up the bowels too much and cause dehydration and low minerals in the body.

When a provider prescribes an IV infusion medication to be titrated, which of the following elements is NOT required to be part of the order? Objective clinical endpoint or patient response Initial or starting rate of the infusion (dose/minute) Ability of the nurse to determine the units of incremental rate Frequency for increasing or decreasing incremental dose

Ability of the nurse to determine the units of incremental rate. According to the Joint Commission, required elements for medication titration orders include the following: 1. medication name; 2. medication route; 3. initial or starting rate of infusion (dose/min); 4. incremental units the rate can be increased or decreased; 5. frequency for incremental doses (how often the dose (rate) can be increased or decreased); 6. maximum rate (dose) of infusion; 7. objective clinical endpoint or patient response. The nurse follows the titration prescription and does not make independent determinations regarding the infusion.

During the second day of hospitalization of the client after a Myocardial Infarction. Which of the following is an expected outcome? A. Able to perform self-care activities without pain B. Severe chest pain C. Can recognize the risk factors of Myocardial Infarction D. Can Participate in cardiac rehabilitation walking program

Able to perform self-care activities without pain By the 2nd day of hospitalization after suffering a Myocardial Infarction, clients are able to perform care without chest pain. Instruct the patient to report pain immediately. Provide a quiet environment, calm activities, and comfort measures. Approach the patient calmly and confidently. Decreases external stimuli, which may aggravate anxiety and cardiac strain, limit coping abilities, and adjustment to the current situation. Option B: Severe chest pain during the second day of hospitalization of the client should be a cause of immediate concern. Monitor and document characteristics of pain, noting verbal reports, nonverbal cues (moaning, crying, grimacing, restlessness, diaphoresis, clutching of chest) and BP or heart rate changes. Option C: The client may recognize the risks of MI but not necessarily on the second day of hospitalization. Review history of previous angina, anginal equivalent, or MI pain. Discuss family history if pertinent. Delay in reporting pain hinders pain relief and may require an increased dosage of medication to achieve relief. In addition, severe pain may induce shock by stimulating the sympathetic nervous system, thereby creating further damage and interfering with diagnostics and relief of pain. Option D: A cardiac rehabilitation walking program might be too soon for the client, but can be included in the discharge instructions. Explain the pattern of graded increase of activity level: getting up to commode or sitting in a chair, progressive ambulation, and resting after meals. Progressive activity provides a controlled demand on the heart, increasing strength and preventing overexertion.

A client is admitted to the labor and delivery unit. The nurse performs a vaginal exam and determines that the client's cervix is 5 cm dilated with 75% effacement. Based on the nurse's assessment the client is in which phase of labor? A. Active B. Latent C. Transition D. Early

Active The active phase of labor occurs when the client is dilated 4-7cm. Active labor with more rapid cervical dilation generally starts around 6 centimeters of dilation. During the active phase, the cervix typically dilates at a rate of 1.2 to 1.5 centimeters per hour. Multiparas, or women with a history of prior vaginal delivery, tend to demonstrate more rapid cervical dilation. The absence of cervical change for greater than 4 hours in the presence of adequate contractions or six hours with inadequate contractions is considered the arrest of labor and may warrant clinical intervention. Option B: The latent phase is commonly defined as the 0 to 6 cm, while the active phase commences from 6 cm to full cervical dilation. The presenting fetal part also begins the process of engagement into the pelvis during the first stage. Throughout the first stage of labor, serial cervical exams are done to determine the position of the fetus, cervical dilation, and cervical effacement. Option C: The transition phase of labor is 8-10cm in dilation. The second stage of labor commences with complete cervical dilation to 10 centimeters and ends with the delivery of the neonate. This was also defined as the pelvic division phase by Friedman. After cervical dilation is complete, the fetus descends into the vaginal canal with or without maternal pushing efforts. Option D: The latent or early phase of labor is from 1cm to 3cm in dilation. During the latent phase, the cervix dilates slowly to approximately 6 centimeters. The latent phase is generally considerably longer and less predictable with regard to the rate of cervical change than is observed in the active phase. A normal latent phase can last up to 20 hours and 14 hours in nulliparous and multiparous women respectively, without being considered prolonged.

Johnny, a firefighter, was involved in extinguishing a house fire and is being treated for smoke inhalation. He developed severe hypoxia 48 hours after the incident, requiring intubation and mechanical ventilation. He most likely has developed which of the following conditions? A. Adult respiratory distress syndrome (ARDS) B. Atelectasis C. Bronchitis D. Pneumonia

Adult respiratory distress syndrome (ARDS) Severe hypoxia after smoke inhalation is typically related to ARDS. Option B: Atelectasis is not associated with smoke inhalation. Inhaling harmful smoke can inflame the lungs and airway, causing them to swell and block oxygen. This can lead to acute respiratory distress syndrome and failure. Option C: Bronchitis does not develop due to smoke inhalation. However, if the client already has bronchitis, inhalational injuries can worsen its condition. Option D: Pneumonia isn't typically associated with smoke inhalation and severe hypoxia.

Which statement made by the nurse describes the inheritance pattern of autosomal recessive disorders? A. An affected newborn has unaffected parents. B. An affected newborn has one affected parent. C. Affected parents have a one in four chance of passing on the defective gene. D. Affected parents have unaffected children who are carriers.

Affected parents have a one in four chance of passing on the defective gene. Autosomal recessive disorders can be passed from the parents to the infant. If both parents pass the trait, the child will get two abnormal genes and the disease results. Parents can also pass the trait to the infant. Patients affected with autosomal recessive (AR) diseases have a disease allele on each chromosome. The pattern of individuals affected with an AR disease can be traced through a family to determine which individuals are carriers and which individuals are likely to become impacted. Option A: To have an affected newborn, the parents must be carriers. The easiest way to determine the inheritance pattern of a disorder in a family is by looking at a pedigree. Autosomal recessive diseases typically affect both females and males equally. Autosomal recessive patterns manifest by skipping generations as the affected are usually children of unaffected carriers. Option B: Both parents must be carriers. The most common situation of an autosomal recessive disease occurs when the parents are each carrier or heterozygous (Dd). Children of carrier parents have a 25% chance of inheriting the disorder. This value is obtained by using the Punnett square model used in genetics. Option D: The parents might have affected children. Each parent has a 50% chance of passing on the disease allele. Using the multiplication rule of probability, there is a 50% chance that the father passes on his disease allele and a 50% chance that the mother passes on her disease allele; 50% x 50% = 25%. So with the mating of carrier parents, there is a 25% chance that the child will be affected, a 50% chance that the child would be a carrier, and 25% chance that they would be homozygous dominant and unaffected.

A patient admitted to the hospital with myocardial infarction develops severe pulmonary edema. Which of the following symptoms should the nurse expect the patient to exhibit? A. Slow, deep respirations B. Stridor C. Bradycardia D. Air hunger

Air hunger Patients with pulmonary edema experience air hunger, anxiety, and agitation. Patients usually present with shortness of breath, which may be acute in onset (from minutes to hours) or gradual in onset occurring over hours to days, depending upon the etiology of pulmonary edema. Option A: Tachypnea is usually present, with the patient gasping for breath. Dyspnea and tachypnea are usually present, and may be associated with the use of accessory muscles of respiration. After initial airway clearance, oxygenation assessment, and maintenance, management mainly depends upon presentation and should be tailored from patient to patient. Supplemental oxygen is a requirement if the patient is at risk of hypoxemia (SPO2 less than 90% ). Option C: Respiration is fast and shallow and heart rate increases. Tachycardia and hypotension may be present along with jugular venous distention. Auscultation of the heart helps to differentiate between the various causes of valvular lesions causing pulmonary edema. Option B: Stridor is noisy breathing caused by laryngeal swelling or spasm and is not associated with pulmonary edema. Fine crackles are usually heard at the bases of lungs bilaterally, and progress apically as the edema worsens. Ronchi and wheeze may also be presenting signs.

After a motor vehicle accident, Armand, a 22-year-old client, is admitted with a pneumothorax. The surgeon inserts a chest tube and attaches it to a chest drainage system. Bubbling soon appears in the water seal chamber. Which of the following is the most likely cause of the bubbling? A. Air leak B. Adequate suction C. Inadequate suction D. Kinked chest tube

Air leak Bubbling in the water seal chamber of a chest drainage system stems from an air leak. In pneumothorax, an air leak can occur as air is pulled from the pleural space. Option B: Inadequate suction does not cause bubbling. Option C: Bubbling doesn't normally occur with adequate suction or any preexisting bubbling in the water seal chamber. Option D: A kinked chest tube does not cause bubbling in the water seal chamber.

The nurse performs an initial abdominal assessment on a patient newly admitted for abdominal pain. The nurse hears what she describes as "clicks and gurgles in all four quadrants" as well as "swishing or buzzing sound heard in one or two quadrants." Which of the following statements is correct? A. The frequency and intensity of bowel sounds varies depending on the phase of digestion. B. In the presence of intestinal obstruction, bowel sounds will be louder and higher pitched. C. A swishing or buzzing sound may represent the turbulent blood flow of a bruit and is not normal. D. All of the above.

All of the above. All of the statements are true. Abdominal examination can give diagnostic clues regarding most gastrointestinal and genitourinary pathologies and may also give insight regarding abnormalities of other organ systems. A well-performed abdominal examination decreases the need for detailed radiological investigations also plays an important role in patient management. Option A: The gurgles and clicks described in the question represent normal bowel sounds, which vary with the phase of digestion. The diaphragm of the stethoscope should be placed on the right side of the umbilicus to listen to the bowel sounds, and their rate should be calculated after listening for at least two minutes. Normal bowel sounds are low-pitched and gurgling, and the rate is normally 2-5/min. Option B: Intestinal obstruction causes the sounds to intensify as the normal flow is blocked by the obstruction. Absent bowel sounds may indicate paralytic ileus and hyperactive rushes (borborygmi) are usually present in small bowel obstruction and sometimes may be auscultated in lactose intolerance Option C: The swishing and buzzing sound of turbulent blood flow may be heard in the abdomen in the presence of abdominal aortic aneurysm, for example, and should always be considered abnormal. The diaphragm should be placed above the umbilicus to listen for an aortic bruit and then moved 2 cm above and lateral to the umbilicus to listen for a renal bruit. The presence of the former indicates an abdominal aortic aneurysm and the latter indicates renal artery atherosclerosis.

Mr. Jay develops hepatic encephalopathy. Which clinical manifestation is most common with this condition? A. Increased urine output B. Altered level of consciousness C. Decreased tendon reflex D. Hypotension

Altered level of consciousness Changes in behavior and level of consciousness are the first signs of hepatic encephalopathy. Hepatic encephalopathy is caused by liver failure and develops when the liver is unable to convert protein metabolic product ammonia to urea. This results in accumulation of ammonia and other toxic in the blood that damages the cells. Option A: The main cause of renal dysfunction in inpatients with liver disease is prerenal failure; specifically, the cause is a reduction in perfusion of the kidneys. Option C: It must be emphasized that the flapping tremor of the extremities is also observed in patients with uremia, pulmonary insufficiency, and barbiturate toxicity. Option D: A diseased liver can cause portal hypertension, which is high blood pressure in the portal vein. The portal vein supplies the liver with blood. Over time, this pressure causes blood vessels to grow, called collateral blood vessels. These vessels act as channels to divert the blood under high pressure.

Hemoptysis may be present in the client with a pulmonary embolism because of which of the following reasons? A. Alveolar damage in the infarcted area. B. Involvement of major blood vessels in the occluded area. C. Loss of lung parenchyma. D. Loss of lung tissue.

Alveolar damage in the infarcted area. The infarcted area produces alveolar damage that can lead to the production of bloody sputum, sometimes in massive amounts. Option B: Clot formation usually occurs in the legs. This is called deep vein thrombosis, which occurs in one or more of the deep veins in the legs. Option C: Loss of lung parenchyma is not found with hemoptysis in pulmonary embolism. The lung parenchyma comprises a large number of thin-walled alveoli, forming an enormous surface area, which serves to maintain proper gas exchange. Option D: A regional loss of surfactant is one of the consequences in pulmonary embolism.

A client is admitted complaining of chest pain. Which of the following drug orders should the nurse question? A. Nitroglycerin B. Ampicillin C. Propranolol D. Verapamil

Ampicillin There is no indication for an antibiotic such as Ampicillin. Penicillins had been very effective against S. aureus; in the past, however, S. aureus has become capable of exhibiting resistance against them by producing a penicillin hydrolyzing enzyme - penicillinase. After that, subsequent efforts to overcome this issue and extend the antimicrobial coverage of penicillins, ampicillin was developed. It is also resistant to acid so that it can be administered orally. Option A: Nitroglycerin is a vasodilatory drug used primarily to provide relief from anginal chest pain. Nitroglycerin has been FDA approved since 2000 and was first sold by Pfizer under the brand name Nitrostat. It is currently FDA approved for the acute relief of an attack or acute prophylaxis of angina pectoris secondary to coronary artery disease. Option C: B-adrenoceptor antagonists, including propranolol, have been advised to be used for the treatment of heart failure, atrial fibrillation, and coronary artery disease. Furthermore, they have demonstrated to improve mortality and morbidity in those with hypertension that is complicated with heart failure, angina, or any history of previous myocardial infarctions. Option D: Clients with chest pain can be treated with nitroglycerin, a beta-blocker such as propranolol, or Verapamil. Given as initial treatment in patients with: (1) Non-ST elevation acute coronary syndrome(NSTE-ACS, (2) continuing or frequently recurring ischemia and are unable to use beta-blockers (e.g., contraindication, suffered from unacceptable adverse effects, beta-blockers were insufficient for treatment).

A patient who has received chemotherapy for cancer treatment is given an injection of Epoetin. Which of the following should reflect the findings in a complete blood count (CBC) drawn several days later? A. An increase in neutrophil count. B. An increase in hematocrit. C. An increase in platelet count. D. An increase in serum iron.

An increase in hematocrit. Epoetin is a form of erythropoietin, which stimulates the production of red blood cells, causing an increase in hematocrit. Epoetin is given to patients who are anemic, often as a result of chemotherapy treatment. Epoetin alfa is 165 amino acid glycoprotein manufactured by recombinant DNA technology, which has similar biological effects as endogenous erythropoietin. Erythropoietin stimulates red blood cell production in-situ. It is a hormone produced in the kidney and augments the differentiation of erythroid progenitors in the bone marrow. Option A: Application of granulocyte-colony stimulating factor (G-CSF) can improve neutrophil functions and number. Prophylactic use of antibiotics and antifungals is reserved for some forms of alteration in neutrophil function such as chronic granulomatous disease CGD). The utilization of antimicrobials is compulsory if recurrent infections exist. Interferon-gamma has been successfully used to improve the quality of life of the patient suffering from neutropenia. Option C: First-line treatment includes glucocorticoids and intravenous immune globulins; these agents inhibit autoantibody production and platelet degradation. Second-line treatment includes rituximab, immunosuppressive drugs, and splenectomy. Third-line agents are thrombopoietin receptor agonists, which stimulate platelet production. Option D: Iron supplementation should be taken without food to increase absorption. Low gastric pH facilitates iron absorption. Rapid response to treatment is often seen in 14 days. It is manifested by the rise in hemoglobin levels. Iron supplementation is needed for at least three months to replenish tissue iron stores and should proceed for at least a month even after hemoglobin has returned to normal levels.

Nurse Sugar is assessing a client with Cushing's syndrome. Which observation should the nurse report to the physician immediately? A. Pitting edema of the legs B. An irregular apical pulse C. Dry mucous membranes D. Frequent urination

An irregular apical pulse Because Cushing's syndrome causes aldosterone overproduction, which increases urinary potassium loss, the disorder may lead to hypokalemia. Therefore, the nurse should immediately report signs and symptoms of hypokalemia, such as an irregular apical pulse, to the physician. Option A: Edema is an expected finding because aldosterone overproduction causes sodium and fluid retention. Option C: Dry mucous membranes is not a symptom of Cushing's syndrome. Thinning of the skin and mucous membranes occur because cortisol causes the breakdown of some dermal proteins along with the weakening of small blood vessels. Option D: Frequent urination signals dehydration, which isn't associated with Cushing's syndrome. Short term administration of adrenocorticotropic hormone or glucocorticoids causes an increased glomerular filtration rate. Glomerular dysfunction leads to proteinuria and albuminuria.

The following lab results are received for a patient. Which of the following results are abnormal? Select all that apply.

Answer: A & B Option A: CBC includes measurement of hemoglobin level in the blood. Normal concentrations of hemoglobin are approximately 13.5-18.0 grams per deciliter in men and 11.5-16.0 grams per deciliter in women. CBC also measures the size of erythrocytes through the mean corpuscular volume (MCV). Option B: Total cholesterol levels of 200 mg/dL or below are considered normal. Cholesterol level measurement is from serum. A non-fasting lipid test can be done anytime without fasting; a fasting lipid test requires a 12-hour fast except for water. Total and HDL cholesterol are measured directly from serum. Option C: The normal serum protein is 6 to 8 g/dl. A total serum protein test measures the total amount of protein in the blood. It also measures the amounts of two major groups of proteins in the blood: albumin and globulin. A test for total serum protein reports separate values for total protein, albumin, and globulin. Some types of globulin (such as alpha-1 globulin) also may be measured. Option D: The normal glycosylated hemoglobin A1C is between 4% and 5.6%. Glycosylated hemoglobin is a hemoglobin to which glucose is bound. Glycosylated hemoglobin is tested to monitor the long-term control of diabetes mellitus. Option E: The normal WBC count is 4.5 to 11.0x10?/L. The normal range of values for white blood cells is 4,000 to 11,000/mL. Anything below this range is leukopenia, and anything that exceeds this range qualifies as leukocytosis. Clinically, the complete blood count (CBC) test measures leukocytes. A CBC is frequently ordered to provide insight into disease processes and includes measurements of the leukocytes, as well as red blood cell and platelet totals.

After undergoing a cardiac catheterization, Tracy has a large puddle of blood under his buttocks. Which of the following steps should the nurse take first? A. Call for help. B. Obtain vital signs. C. Ask the client to "lift up". D. Apply gloves and assess the groin site.

Apply gloves and assess the groin site. Observing standard precautions is the first priority when dealing with any blood fluid. Assessment of the groin site is the second priority. This establishes where the blood is coming from and determines how much blood has been lost. The goal in this situation is to stop the bleeding. Option A: The nurse would call for help if it were warranted after the assessment of the situation. Option B: After determining the extent of the bleeding, vital signs assessment is important. Option C: The nurse should never move the client, in case a clot has formed. Moving can disturb the clot and cause rebleeding.

Brittany who is undergoing chemotherapy for her throat cancer is experiencing stomatitis. To promote oral hygiene and comfort, the nurse-in-charge should: A. Provide frequent mouthwash with normal saline. B. Apply viscous Lidocaine to oral ulcers as needed. C. Use lemon glycerine swabs every 2 hours. D. Rinse mouth with Hydrogen Peroxide.

Apply viscous Lidocaine to oral ulcers as needed. Stomatitis can cause pain and this can be relieved by applying topical anesthetics such as lidocaine before mouth care. Option A: Before providing oral care, ensure that the patient is comfortable with the procedure first. Option C: Use saline solution mixed with equal parts of water or hydrogen peroxide or oral care. Option D: When the patient is already comfortable, the nurse can proceed with providing the patient with oral rinses of saline solution mixed with equal parts of water or hydrogen peroxide mixed water in 1:3 concentrations to promote oral hygiene. Every 2-4 hours.

When evaluating an arterial blood gas from a male client with a subdural hematoma, the nurse notes the Paco2 is 30 mm Hg. Which of the following responses best describes the result? A. Appropriate; lowering carbon dioxide (CO2) reduces intracranial pressure (ICP). B. Emergent; the client is poorly oxygenated. C. Normal. D. Significant; the client has alveolar hypoventilation.

Appropriate; lowering carbon dioxide (CO2) reduces intracranial pressure (ICP) A normal Paco2 value is 35 to 45 mm Hg CO2 has vasodilating properties; therefore, lowering Paco2 through hyperventilation will lower ICP caused by dilated cerebral vessels. Option B: Oxygenation is evaluated through Pao2 and oxygen saturation. Option C: The normal PaCO2 level is between 35 to 45 mmHg. PaCO2 or the partial pressure of carbon dioxide is the measure of carbon dioxide within arterial or venous blood. Option D: Alveolar hypoventilation would be reflected in an increased Paco2. Alveolar hypoventilation is defined as insufficient ventilation leading to hypercapnia, which is an increase in the partial pressure of carbon dioxide as measured by arterial blood gas analysis.

A patient is admitted to the hospital with suspected polycythemia vera. Which of the following symptoms is consistent with the diagnosis? Select all that apply. A. Weight loss B. Increased clotting time C. Hypertension D. Headaches E. Tinnitus

B, C, D & E Polycythemia vera is a condition in which the bone marrow produces too many red blood cells. This causes an increase in hematocrit and viscosity of the blood. Patients can experience headaches, dizziness, tinnitus, and visual disturbances. Cardiovascular effects include increased blood pressure and delayed clotting time. Option A: Weight loss is not a manifestation of polycythemia vera. Weight loss adversely impacts survival in cancer patients. JAK2 myeloproliferative neoplasms (MPN) upregulate tumor necrosis factor alpha (TNF-?), interleukin-6 (IL-6), and IL-8 and induce decreased leptin levels leading to weight loss. The impact of weight loss in PV patients receiving best supportive care (i.e. frontline hydroxyurea [HU] therapy, phlebotomy) on overall survival (OS) is largely unknown. Option B: Bleeding and thrombotic complications are each observed in 1% of patients. Bleeding events can include epistaxis, gum bleeding, and gastrointestinal (GI) bleeding. Thrombotic events can include deep venous thrombosis (DVT), pulmonary embolism (PE), Budd-Chiari syndrome, splanchnic vein thrombosis, stroke, and arterial thrombosis. Option C: The overproduction of red blood cells and high hematocrit levels associated with polycythemia vera can contribute to systemic hypertension; high hematocrit levels have been found to interfere with the vasodilatory effects of nitric oxide. The treatment of polycythemia, which can involve phlebotomy, can alleviate the systemic hypertension, as well as the physiological consequences of having a high red blood cell count. Option D: As polycythemia vera is a myeloproliferative syndrome, it is based on an autonomic increase in the proliferation of all hematopoietic cells--mostly of erythropoiesis. An increase in blood viscosity induces disturbed microcirculation, resulting in headaches with clinical symptoms. Option E: Symptoms are related to hyperviscosity and thrombosis, impairing oxygen delivery. Physical complaints can include fatigue, headache, dizziness, tinnitus, vision changes, insomnia, claudication, pruritus, gastritis, and early satiety.

A patient in labor and delivery has just received an amniotomy. Which of the following is correct? Select all that apply. A. Frequent checks for cervical dilation will be needed after the procedure. B. Contractions may rapidly become stronger and closer together after the procedure. C. The FHR (fetal heart rate) will be followed closely after the procedure due to the possibility of cord compression. D. The procedure is usually painless and is followed by a gush of amniotic fluid. E. The procedure is without pain.

B, C, D & E. Uterine contractions typically become stronger and occur more closely together following amniotomy. The FHR is assessed immediately after the procedure and followed closely to detect changes that may indicate cord compression. The procedure itself is painless and results in the quick expulsion of amniotic fluid. Option A: Following amniotomy, cervical checks are minimized because of the risk of infection. Amniotomy is easily performed with the use of specially designed hooks intended to grab and tear the amniotic membrane. The two most commonly used devices are (1) an approximately 10-inch rod with a hook on the end of the rod or (2) a finger cot with a hook on the end of the cot. With either device, the practitioner first assesses cervical dilation through the performance of a sterile digital exam. Option B: It is commonly felt that relieving the amniotic sac of amniotic fluid induces uterine contraction activity, increases the strength of contractions, and may augment labor by allowing direct pressure from the fetal scalp on the uterine cervix which may assist in dilating the cervix. Option C: In the case of an unengaged fetal head, rupture of membranes may allow for the umbilical cord to precede the fetal head when the release of amniotic fluid occurs. This will allow the fetal head to compress the section of umbilical cord preceding the head, generally leading to fetal bradycardia and necessitating emergency cesarean section. This complication should be an easily avoidable, iatrogenic cause of emergency delivery. Option D: The nurse plays a vital role during the procedure in monitoring the mother as well as the fetus, she also notes the color of the draining amniotic fluid and documents the findings in the medical chart. After the procedure, she assesses the maternal temperature every two hours and watches out for any signs of infection. The nurse also monitors the fetal heart rate via continuous electronic fetal monitoring and communicates the findings to the provider. Option E: Pain is not associated with amniotomy. Practitioners have believed that artificial rupture of membranes either can assist in inducing labor or augmenting spontaneous labor. It is commonly felt that relieving the amniotic sac of amniotic fluid induces uterine contraction activity, increases the strength of contractions, and may augment labor by allowing direct pressure from the fetal scalp on the uterine cervix which may assist in dilating the cervix.

Nurse Sarah expects to note an elevated serum glucose level in a client with hyperosmolar hyperglycemic nonketotic syndrome (HHNS). Which other laboratory finding should the nurse anticipate? A. Elevated serum acetone level. B. Serum ketone bodies. C. Serum alkalosis. D. Below-normal serum potassium level.

Below-normal serum potassium level. A client with HHNS has an overall body deficit of potassium resulting from diuresis, which occurs secondary to the hyperosmolar, hyperglycemic state caused by the relative insulin deficiency. Option A: An elevated serum acetone level is a symptom of diabetic ketoacidosis. Hepatic metabolism of free fatty acids as an alternative energy source results in accumulation of acidic intermediate and end metabolites (ie, ketones). Ketone bodies have generally included acetone, a true ketone. Option B: Serum ketone bodies are characteristic of diabetic ketoacidosis. Ketone bodies are produced from acetyl coenzyme A mainly in the mitochondria within hepatocytes when carbohydrate utilization is impaired because of relative or absolute insulin deficiency, such that energy must be obtained from fatty acid metabolism. Option C: Metabolic acidosis, not serum alkalosis, may occur in HHNS. A wide anion gap can be observed in patients with HHNS. The mild acidosis in HHNS is often multifactorial and results, in part, from the accumulation of minimal ketoacids in the absence of effective insulin activity.

Cyrill with severe head trauma sustained in a car accident is admitted to the intensive care unit. Thirty-six hours later, the client's urine output suddenly rises above 200 ml/hour, leading the nurse to suspect diabetes insipidus. Which laboratory findings support the nurse's suspicion of diabetes insipidus? A. Above-normal urine and serum osmolality levels. B. Below-normal urine and serum osmolality levels. C. Above-normal urine osmolality level, below-normal serum osmolality level. D. Below-normal urine osmolality level, above-normal serum osmolality level.

Below-normal urine osmolality level, above-normal serum osmolality level In diabetes insipidus, excessive polyuria causes dilute urine, resulting in a below-normal urine osmolality level. At the same time, polyuria depletes the body of water, causing dehydration that leads to an above-normal serum osmolality level. Option A: Urine osmolality level should be below normal because of excessive polyuria. Option B: Serum osmolality levels should be above normal because of dehydration. Option C: For the same reasons, diabetes insipidus doesn't cause above-normal urine osmolality or below-normal serum osmolality levels.

Which of the following classes of medications protects the ischemic myocardium by blocking catecholamines and sympathetic nerve stimulation? Beta-adrenergic blockers Calcium channel blockers Narcotics Nitrates

Beta-adrenergic blockers Beta-adrenergic blockers work by blocking beta receptors in the myocardium, reducing the response to catecholamines and sympathetic nerve stimulation. They protect the myocardium, helping to reduce the risk of another infarction by decreasing the workload of the heart and decreasing myocardial oxygen demand. B) Calcium channel blockers reduce the workload of the heart by decreasing the heart rate. C) Narcotics reduce myocardial oxygen demand, promote vasodilation and decrease anxiety. D) Nitrates reduce myocardial oxygen consumption by decreasing left ventricular end-diastolic pressure (preload) and systemic vascular resistance (afterload).

A client has had a unilateral adrenalectomy to remove a tumor. To prevent complications, the most important measurement in the immediate postoperative period for the nurse to take is: A. Blood pressure B. Temperature C. Output D. Specific gravity

Blood pressure Blood pressure is the best indicator of cardiovascular collapse in the client who has had an adrenal gland removed. The remaining gland might have been suppressed due to the tumor activity. Primary adrenal insufficiency occurs after bilateral adrenalectomy. Signs and symptoms are volume depletion, hypotension, hyponatremia, hyperkalemia, fever, abdominal pain. Patients are managed by replacement therapy based on glucocorticoids (hydrocortisone or cortisone), mineralocorticoids (fludrocortisone) in cases of confirmed corticoids or aldosterone deficiency, respectively. Option B: Temperature would be an indicator of infection. Patients in the adrenal crisis typically present with profoundly impaired well-being, hypotension, nausea and vomiting, and fever responding well to parenteral hydrocortisone administration. Infections are the major precipitating causes of adrenal crisis. Option C: Decreased output would be a clinical manifestation but would take longer to occur than blood pressure changes. The clinician must be able to work-up and manage patients with adrenal masses, both functional and non-functional, to treat these patients with minimal morbidity. When planning for adrenalectomy, considerations of hormonal changes and preoperative preparation for these changes is as important and demands as much of the surgeon's attention as the technical aspects of the case. Option D: Specific gravity changes occur with other disorders. Adrenalectomy has been shown to have a relatively low risk of postoperative complications, with an overall rate of 3.6%. Improved patient outcomes and decreased hospital costs have been demonstrated when adrenalectomy is performed by a high-volume adrenal surgeon (>/=6 adrenalectomies/year).

A nurse assigned to the emergency department evaluates a patient who underwent fiberoptic colonoscopy 18 hours previously. The patient reports increasing abdominal pain, fever, and chills. Which of the following conditions poses the most immediate concern? A. Bowel perforation B. Viral gastroenteritis C. Colon cancer D. Diverticulitis

Bowel perforation Bowel perforation is the most serious complication of fiberoptic colonoscopy. Important signs include progressive abdominal pain, fever, chills, and tachycardia, which indicate advancing peritonitis. Bowel perforation results from insult or injury to the mucosa of the bowel wall resulting from a violation of the closed system. This exposes the structures within the peritoneal cavity to gastrointestinal contents. Patients presenting with abdominal pain and distension, especially in the appropriate historical setting, must be evaluated for this entity as delayed diagnosis can be life-threatening due to the risk of developing infections such as peritonitis. Option B: Several different viruses including rotavirus, norovirus, adenovirus, and astroviruses account for most cases of acute viral gastroenteritis. Most are transmitted via the fecal-oral route, including contaminated food and water. Transmission has also been shown to occur via fomites, vomitus, and possibly airborne methods. Norovirus is more resistant to chlorine and ethanol inactivation than other viruses. Acute gastroenteritis is defined by loose or watery diarrhea that consists of 3 or more bowel movements in a day. Other symptoms may include nausea, vomiting, fever, or abdominal pain Option C: Colon cancer does not cause these symptoms. Tumor location on clinical presentation can be separated on left-sided with more changes in bowel habits and hematochezia, and right-sided with obscured anemia impacting on late stage at diagnosis. The provider should perform a thorough physical examination for signs of ascites, hepatomegaly, and lymphadenopathy. Option D: Diverticulitis may cause pain, fever, and chills, but is far less serious than perforation and peritonitis. Acute diverticulitis is inflammation due to micro-perforation of a diverticulum. The diverticulum is a sac-like protrusion of the colon wall. Diverticulitis can present in about 10% to 25% of patients with diverticulosis. Diet appears to play a significant role. Low fiber, high fat, and red meat diets may increase the risk for development of diverticulosis and possible diverticulitis. Obesity and smoking are known to increase the potential for both diverticulitis and diverticular bleeding.

Francis with leukemia has neutropenia. Which of the following functions must be frequently assessed? A. Blood pressure B. Bowel sounds C. Heart sounds D. Breath sounds

Breath sounds Pneumonia, both viral and fungal, is a common cause of death in clients with neutropenia, so frequent assessment of respiratory rate and breath sounds is required. Option A: Blood pressure should be monitored regularly, but it is not a priority with neutropenia. Option B: Assessing bowel sounds is not associated with neutropenia. Leukemia may affect the small and large bowel and they are usually hemorrhagic or infiltrative. Option C: Although assessing heart sounds is important, it won't help detect neutropenia. Patients with acute leukemia develop a higher rate of congestive heart failure than patients with other cancers.

Which of the following sounds is distinctly heard on auscultation over the abdominal region of an abdominal aortic aneurysm client? Bruit Crackles Dullness Friction Rubs

Bruit A bruit, a vascular sound resembling a heart murmur, suggests partial arterial occlusion. In addition to abdominal palpation, auscultation for abdominal or femoral bruits may be useful for clinical detection of AAA. Auscultation is performed along the course of the aortic and femoral arteries. However, absence of a bruit does not exclude an aneurysm. B) Crackles are indicative of fluid in the lungs. Crackles occur if the small air sacs in the lungs fill with fluid and there's any air movement in the sacs, such as when breathing. The air sacs fill with fluid when a person has pneumonia or heart failure. C) Dullness is heard over solid organs, such as the liver. A dull or thud-like sound is normally heard over dense areas such as the heart or liver. Dullness replaces resonance when fluid or solid tissue replaces air-containing lung tissues, such as occurs with pneumonia, pleural effusions, or tumors D) Friction rubs indicate inflammation of the peritoneal surface. A pericardial friction rub is pathognomonic for acute pericarditis; the rub has a scratching, grating sound similar to leather rubbing against leather. Serial examinations may be necessary for detection, as a friction rub may be transient from one hour to the next and is present in approximately 50% of cases.

Cleo is diagnosed with osteoporosis. Which electrolytes are involved in the development of this disorder? A. Calcium and sodium B. Calcium and phosphorous C. Phosphorus and potassium D. Potassium and sodium

Calcium and phosphorous In osteoporosis, bones lose calcium and phosphate salts, becoming porous, brittle, and abnormally vulnerable to fracture. Option A: Sodium increases calcium excretion and higher calcium excretion is associated with lower bone mineral density, a predictor of osteoporotic fractures. Option C: Potassium is not involved in osteoporosis. Potassium salts aid in bone health. A study, published in the journal Osteoporosis International, also revealed that high intake of potassium salts significantly reduces the excretion of calcium and acid in urine. Option D: Sodium, in the form of sodium chloride, elevates urinary calcium excretion and, at prevailing calcium intakes, evokes compensatory responses that may lead to increased bone remodeling and bone loss. However, potassium was inversely associated with both urinary calcium excretion and intestinal calcium absorption, yielding no significant net change in calcium balance.

The nurse is aware that the following terms used to describe reduced cardiac output and perfusion impairment due to ineffective pumping of the heart is: A. Anaphylactic shock B. Cardiogenic shock C. Distributive shock D. Myocardial infarction (MI)

Cardiogenic shock Cardiogenic shock is shock related to ineffective pumping of the heart. Option A: Anaphylactic shock results from an allergic reaction. This severe reaction happens when an over-release of chemicals puts the person into shock. Option C: Distributive shock results from changes in the intravascular volume distribution and is usually associated with increased cardiac output. Option D: MI isn't a shock state, though in most cases, a lack of oxygen to the heart, usually from a heart attack, damages its main pumping chamber. Without oxygen-rich blood circulating to that area of the heart, the heart muscle can weaken and go into cardiogenic shock.

Which of the following heart muscle diseases is unrelated to other cardiovascular diseases? Cardiomyopathy CAD MI Pericardial Effusion

Cardiomyopathy Cardiomyopathy isn't usually related to underlying heart disease such as atherosclerosis. The etiology in most cases is unknown. Option B: Coronary artery disease is directly related to atherosclerosis. Coronary artery disease is caused by plaque buildup in the wall of the arteries that supply blood to the heart (called coronary arteries). Plaque is made up of cholesterol deposits. Plaque buildup causes the inside of the arteries to narrow over time. This process is called atherosclerosis. Option C: During the progression of atherosclerosis, myeloid cells destabilize lipid-rich plaque in the arterial wall and cause its rupture, thus triggering myocardial infarction. Option D: Pericardial effusion is the escape of fluid into the pericardial sac, a condition associated with pericarditis and advanced heart failure.

A patient arrives at the emergency department complaining of midsternal chest pain. Which of the following nursing actions should take priority? A. A complete history with emphasis on preceding events. B. An electrocardiogram. C. Careful assessment of vital signs. D. Chest exam with auscultation.

Careful assessment of vital signs. The priority nursing action for a patient arriving at the ED in distress is always an assessment of vital signs. This indicates the extent of the physical compromise and provides a baseline by which to plan further assessment and treatment. Monitor vital signs every 5 min during the initial anginal attack. Blood pressure may initially rise because of sympathetic stimulation, then fall if cardiac output is compromised. Tachycardia also develops in response to sympathetic stimulation and may be sustained as a compensatory response if cardiac output falls. Option A: A thorough medical history, including the onset of symptoms, will be necessary. Identify precipitating events, if any: frequency, duration, intensity, and location of the pain. Helps differentiate this chest pain, and aids in evaluating possible progression to unstable angina. Option B: It is likely that an electrocardiogram will be performed as well, but this is not the first priority. Monitor heart rate and rhythm. Patients with unstable angina have an increased risk of acute life-threatening dysrhythmias, which occur in response to ischemic changes and/or stress. Option D: Similarly, chest exams with auscultation may offer useful information after vital signs are assessed. Observe for associated symptoms: dyspnea, nausea, and vomiting, dizziness, palpitations, desire to micturate. Decreased cardiac output (which may occur during an ischemic myocardial episodes) stimulates sympathetic and parasympathetic nervous systems, causing a variety of vague sensations that the patient may not identify as related to the anginal episode.

Thrombolytic therapy is frequently used in the treatment of suspected strokes. Which of the following is a significant complication associated with thrombolytic therapy? A. Air embolism B. Cerebral hemorrhage C. Expansion of the clot D. Resolution of the clot

Cerebral hemorrhage Cerebral hemorrhage is a significant risk when treating a stroke victim with thrombolytic therapy intended to dissolve a suspected clot. Success of the treatment demands that it be instituted as soon as possible, often before the cause of stroke has been determined. Bleeding is the most frequent complication of thrombolytic therapy and can occur in a puncture site or spontaneously anywhere inside the body. Intracranial hemorrhage or hemorrhagic stroke is the greatest concern. Option A: Air embolism is not a concern. Adverse effects of any fibrinolytic agents are almost similar, which include, but are not limited to, bleeding, hypotension, allergic reactions, angioedema, and reperfusion arrhythmias (when used in acute MI). Among all of the fibrinolytic agents, streptokinase is the most antigenic, thus most frequently complicated by allergic reaction and hypotension. Option C: The end goal of this therapy is to convert plasminogen into plasmin which is accomplished at the location of the thrombus and on the surface of fibrin by the binding of tPA to plasminogen. This binding helps the conversion. Option D: Thrombolytic therapy does not lead to expansion of the clot, but to resolution, which is the intended effect. Thrombolytic treatment is also known as fibrinolytic or thrombolysis, to dissolve dangerous intravascular clots to prevent ischemic damage by improving blood flow.

A nurse is caring for patients in the oncology unit. Which of the following is the most important nursing action when caring for a neutropenic patient? A. Change the disposable mask immediately after use. B. Change gloves immediately after use. C. Minimize patient contact. D. Minimize conversation with the patient.

Change gloves immediately after use. The neutropenic patient is at risk of infection. Changing gloves immediately after use protects patients from contamination with organisms picked up on hospital surfaces. This contamination can have serious consequences for an immunocompromised patient. Wear gloves when providing direct care; perform hand hygiene after properly disposing gloves. Option A: Changing the respiratory mask is desirable, but not nearly as urgent as changing gloves. Wear personal protective equipment (PPE) properly. Use masks, goggles, face shields to protect the mucous membranes of your eyes, mouth, and nose during procedures and in direct-care activities (e.g., suctioning secretions) that may generate splashes or sprays of blood, body fluids, secretions, and excretions. Option C: Place the patient in protective isolation if the patient is at high risk of infection. Protective isolation is set when the WBC indicates neutropenia. Provide surgical masks to visitors who are coughing and provide rationale to enforce usage. Instruct visitors to cover mouth and nose (by using the elbows to cover) during coughing or sneezing; use of tissues to contain respiratory secretions with immediate disposal to a no-touch receptacle; perform hand hygiene afterward. Option D: Minimizing conversations are not necessary and may cause nursing staff to miss changes in the patient's symptoms or condition. Educating visitors on the importance of preventing droplet transmission from themselves to others reduces the risk of infection.

Which of the following should the nurse teach the client about the signs of digitalis toxicity? A. Increased appetite B. Elevated blood pressure C. Skin rash over the chest and back D. Visual disturbances such as seeing yellow spots

Visual disturbances such as seeing yellow spots Seeing yellow spots and colored vision are common symptoms of digitalis toxicity. Of note visual changes especially changes involving colors such as seeing a yellow hue are better known and specifically seen in digitalis toxicity. Other visual problems include photophobia, photopsia and diminished visual acuity. Option A: GI symptoms of digitalis toxicity include anorexia and weight loss. Most symptoms are nonspecific findings and include a headache, malaise, insomnia, altered mental status, abdominal pain, nausea, and vomiting. Option B: Hypotension is one of the cardiac symptoms of digitalis toxicity. There is no specific arrhythmia for digoxin toxicity rather a range of arrhythmias can be present such as various degrees of AV block, premature ventricular contractions, bradycardia, and even ventricular tachycardia. Cardiac arrhythmias are the main cause of death for those with digoxin toxicity. Option C: There are no skin rashes in patients with digitalis toxicity. History of exposure is necessary to determine if poisoning is acute or chronic. Most reported poisonings result from chronic toxicity. Clinical signs of toxicity include gastrointestinal, neurological and the most concerning cardiac.

A nurse is evaluating a postoperative patient and notes a moderate amount of serous drainage on the dressing 24 hours after surgery. Which of the following is the appropriate nursing action? A. Notify the surgeon about evidence of infection immediately. B. Leave the dressing intact to avoid disturbing the wound site. C. Remove the dressing and leave the wound site open to air. D. Change the dressing and document the clean appearance of the wound site.

Change the dressing and document the clean appearance of the wound site. A moderate amount of serous drainage from a recent surgical site is a sign of normal healing. Serous drainage is clear, thin, and watery. The production of serous drainage is a typical response from the body during the normal inflammatory healing stage. Option A: Purulent drainage would indicate the presence of infection. Purulent drainage is milky, typically thicker in consistency, and can be gray, green, or yellow in appearance. If the fluid becomes very thick, this can be a sign of infection. Yet, if there is a large amount of serous drainage, it can be the result of a high bioburden count. Option B: A soiled dressing should be changed to avoid bacterial growth and to examine the appearance of the wound. Overall, it should be noted that the dressing selection should be based on the individual patient and wound characteristics. If the wound is not in the normal inflammatory phase of healing, the clinician must investigate what is the root cause and how to manage the drainage. Option C: The surgical site is typically covered by gauze dressings for a minimum of 48-72 hours to ensure that initial healing has begun. Changing the dressing less allows the wound bed to be left undisturbed, which allows for the migration of new cells. When wound beds are left undisturbed in an optimal moist environment, they are able to heal at a faster rate.

A patient with a history of congestive heart failure arrives at the clinic complaining of dyspnea. Which of the following actions is the first the nurse should perform? A. Ask the patient to lie down on the exam table. B. Draw blood for chemistry panel and arterial blood gas (ABG). C. Send the patient for a chest x-ray. D. Check blood pressure.

Check blood pressure. A patient with congestive heart failure and dyspnea may have pulmonary edema, which can cause severe hypertension. Therefore, taking the patient's blood pressure should be the first action. Monitor BP and central venous pressure (CVP). Hypertension and elevated CVP suggest fluid volume excess and may reflect developing pulmonary congestion, HF. Option A: Lying flat on the exam table would likely worsen the dyspnea, and the patient may not tolerate it. Maintain chair or bed rest in semi-Fowler's position during acute phase. Recumbency increases glomerular filtration and decreases production of ADH, thereby enhancing diuresis. Option B: Blood draws for chemistry and ABG will be required, but not prior to the blood pressure assessment. Investigate reports of sudden extreme dyspnea and air hunger, need to sit straight up, sensation of suffocation, feelings of panic or impending doom. Option C: The patient may be sent for an xray after initial assessment. Monitor chest x-ray. Reveals changes indicative of resolution of pulmonary congestion. Maintain fluid and sodium restrictions as indicated. Reduces total body water and prevents fluid reaccumulation.

A patient with leukemia is receiving chemotherapy that is known to depress bone marrow. A CBC (complete blood count) reveals a platelet count of 25,000/microliter. Which of the following actions related specifically to the platelet count should be included on the nursing care plan? A. Monitor for fever every 4 hours. B. Require visitors to wear respiratory masks and protective clothing. C. Consider transfusion of packed red blood cells. D. Check for signs of bleeding, including examination of urine and stool for blood.

Check for signs of bleeding, including examination of urine and stool for blood. A platelet count of 25,000/microliter is severely thrombocytopenic and should prompt the initiation of bleeding precautions, including monitoring urine and stool for evidence of bleeding. Review laboratory results for coagulation status as appropriate: platelet count, prothrombin time/international normalized ratio (PT/INR), activated partial thromboplastin time (aPTT), fibrinogen, bleeding time, fibrin degradation products, vitamin K, activated coagulation time (ACT). Option A: Educate the at-risk patient and caregivers about precautionary measures to prevent tissue trauma or disruption of the normal clotting mechanisms. Thoroughly conform the patient to surroundings; put call light within reach and teach how to call for assistance; respond to call light immediately. Option B: Monitoring for fever and requiring protective clothing are indicated to prevent infection if white blood cells are decreased. Wash hands and teach patient and SO to wash hands before contact with patients and between procedures with the patient; encourage fluid intake of 2,000 to 3,000 mL of water per day, unless contraindicated. Option C: Transfusion of red cells is indicated for severe anemia. Prehospital care focuses on the ABCs (airway, breathing, circulation), which include providing oxygen, controlling severe hemorrhage, and initiating intravenous (IV) fluids to maintain hemodynamic stability; airway control may be necessary for a large intracranial hemorrhage.

Which meal best promotes healing for a patient recovering from a burn injury? Pasta marinara, garlic bread, ginger ale Peanut butter and jelly sandwich, banana, tea Pork chop, fried potatoes, coffee Chicken breast, strawberries, milk

Chicken breast, strawberries, milk- CORRECT The meal with the best nutrition for wound-healing includes protein and vitamin C. Foods that have low nutritional value, such as sugar or those with low or no calories, are not beneficial.

A patient arrives in the emergency department with symptoms of myocardial infarction, progressing to cardiogenic shock. Which of the following symptoms should the nurse expect the patient to exhibit with cardiogenic shock? A. Hypertension B. Bradycardia C. Bounding pulse D. Confusion

Confusion Cardiogenic shock severely impairs the pumping function of the heart muscle, causing diminished blood flow to the organs of the body. This results in diminished brain function and confusion. Cardiogenic shock is a primary cardiac disorder characterized by a low cardiac output state of circulatory failure that results in end-organ hypoperfusion and tissue hypoxia. Option A: If a sufficient area of myocardium undergoes ischemic injury, LV pump function becomes depressed and systemic hypotension develops. The pathophysiology of cardiogenic shock is complex and not fully understood. Ischemia to the myocardium causes derangement to both systolic and diastolic left ventricular function, resulting in a profound depression of myocardial contractility. This, in turn, leads to a potentially catastrophic and vicious spiral of reduced cardiac output and low blood pressure, perpetuating further coronary ischemia and impairment of contractility. Option B: Myocardial ischemia is further exacerbated by impaired myocardial perfusion due to hypotension and tachycardia. The presenting symptoms of cardiogenic shock are variable. The most common clinical manifestations of shock, such as hypotension, altered mental status, oliguria, and cold, clammy skin, can be seen in patients with cardiogenic shock Option C: Peripheral pulses are rapid and faint and may be irregular if arrhythmias are present. Clinical criteria include a systolic blood pressure of less than or equal to 90 mm Hg for greater than or equal to 30 minutes or support to maintain systolic blood pressure less than or equal to 90 mm Hg and urine output less than or equal to 30 mL/hr or cool extremities.

A patient with newly diagnosed diabetes mellitus is learning to recognize the symptoms of hypoglycemia. Which of the following symptoms is indicative of hypoglycemia? A. Polydipsia B. Confusion C. Blurred vision D. Polyphagia

Confusion Hypoglycemia in diabetes mellitus causes confusion, indicating the need for carbohydrates. Neuroglycopenic signs and symptoms are signs and symptoms that result from direct central nervous system (CNS) deprivation of glucose. These include behavioral changes, confusion, fatigue, seizure, coma, and potential death if not immediately corrected. Option A: Neurogenic signs and symptoms can either be adrenergic (tremor, palpitations, anxiety) or cholinergic (hunger, diaphoresis, paresthesias). Neurogenic symptoms and signs arise from sympathoadrenal involvement (either norepinephrine or acetylcholine release) in response to perceived hypoglycemia. Option C: Patients with diabetes mellitus (DM) often experience subjective symptoms of blurred vision associated with hyperglycemia. The nature and origin of this phenomenon are still unclear. Blurred vision during hyperglycemia could be a result of transient refractive alterations due to changes in the lens, but it could also be caused by changes in the retina. Option D: Polydipsia, blurred vision, and polyphagia are symptoms of hyperglycemia. Symptoms of severe hyperglycemia include polyuria, polydipsia, and weight loss. As the patient's blood glucose increases, neurologic symptoms can develop. The patient may experience lethargy, focal neurologic deficits, or altered mental status. The patient can progress to a comatose state.

During a home visit, a client with AIDS tells the nurse that he has been exposed to measles. Which action by the nurse is most appropriate? A. Administer an antibiotic B. Contact the physician for an order for immune globulin C. Administer an antiviral D. Tell the client that he should remain in isolation for 2 weeks

Contact the physician for an order for immune globulin The client who is immunosuppressed and is exposed to measles should be treated with medications to boost his immunity to the virus. If the patient knows that he has been exposed to measles and his CD4 count is less than 200, he should talk to his doctor about whether post-exposure prophylaxis (PEP) with immunoglobulin may be an option. PEP may provide some protection or lessen the severity of infection if it occurs. If the CD4 count is 200 or greater, PEP can also include getting the MMR vaccine. Ideally, PEP should be administered within 72 hours of exposure to measles. Option A: Antibiotics may not be an effective treatment. One important characteristic of measles infection is that it produces more serious illness and increased mortality among immunocompromised individuals, primarily those with defects in T-cell immunity. Because >90% of the human immunodeficiency virus (HIV)-infected children live in regions where measles is still endemic, achieving high rates of measles vaccine coverage is especially important among these populations to suppress excess measles-associated morbidity and mortality. Option C: Antivirals would not be as effective as immunoglobulins for the client with AIDS. Early identification and antiretroviral treatment of HIV-infected infants and children are critical to maximizing measles vaccine immunogenicity and providing protection against other HIV-related complications. Option D: The patient should remain in isolation, but the administration of immunoglobulin is a priority. The impact of HIV-related immunocompromise and subsequent effects of antiretroviral therapy (ART) on immune reconstitution and, ultimately, on vaccine immunogenicity is unclear.

A nurse is assessing a clinic patient with a diagnosis of hepatitis A. Which of the following is the most likely route of transmission? A. Sexual contact with an infected partner B. Contaminated food C. Blood transfusion D. Illegal drug use

Contaminated food Hepatitis A is the only type that is transmitted by the fecal-oral route through contaminated food. Endemic rates are high in developing countries with low socioeconomic conditions and poor sanitation and hygiene practices. Exposure in these developing countries usually occurs in childhood. The incidence of HAV in a given population correlates with socioeconomic properties such as income, the density of housing, sanitation, and water quality. Option A: Hepatitis B infection is a serious global healthcare problem. Often transmitted via body fluids like blood, semen, and vaginal secretions, the hepatitis B virus can cause liver injury. It involves the transmission of HBV through sexual contact or mucosal surface contact. Unprotected sex and injection drug use are major modes of transmission in low to intermediate prevalence areas. Option C: The patients should be told not to donate blood or any organs as the risk of transmission is high. Hepatitis C is a serious infection that has high morbidity and mortality. The management of HCV is prohibitively expensive, and newer antivirals offer a potential cure for the disorder. Option D: Hepatitis B, C, and D are transmitted through infected bodily fluids. Hepatitis D virus infection is an acute and chronic inflammatory process transmitted parenterally. Hepatitis D replicates independently within hepatocytes but requires hepatitis B surface antigen for propagation. Hepatic cell death occurs due to direct cytotoxic effects of hepatitis D virus or a host-mediated immune response. Risk factors include blood transfusions and intravenous drug use.

A client with preeclampsia has been receiving an infusion containing magnesium sulfate for a blood pressure that is 160/80; deep tendon reflexes are 1 plus, and the urinary output for the past hour is 100mL. The nurse should: A. Continue the infusion of magnesium sulfate while monitoring the client's blood pressure B. Stop the infusion of magnesium sulfate and contact the physician C. Slow the infusion rate and turn the client on her left side D. Administer calcium gluconate IV push and continue to monitor the blood pressure

Continue the infusion of magnesium sulfate while monitoring the client's blood pressure The client's blood pressure and urinary output are within normal limits. The only alteration from normal is the decreased deep tendon reflexes. The nurse should continue to monitor the blood pressure and check the magnesium level. The therapeutic level is 4.8-9.6mg/dL. Magnesium levels must be monitored frequently by checking serum levels every 6 to 8 hours or clinically by following patellar reflexes or urinary output. Option B: Do not stop the infusion. If serum concentration levels are low, a proper dose of magnesium sulfate can be given parenterally to replete low serum concentrations with recommended follow up laboratory testing. Option C: There is no need to stop the infusion at this time or slow the rate. If patients exhibit signs and symptoms of hypermagnesemia, the recommendation is to discontinue magnesium sulfate products immediately. If the patient consumed magnesium sulfate orally, then the use of magnesium-free enemas or cathartics can be useful in removing excess magnesium from the GI tract. Option D: Calcium gluconate is the antidote for magnesium sulfate, but there is no data to indicate toxicity. Patients should receive parenteral doses of calcium gluconate to help alleviate symptoms, but continued doses may be necessary as the calcium provides temporary improvement. IV hydration should also occur if clinically appropriate.

The nurse is preparing to administer intravenous procainamide to a client with stable ventricular tachycardia. Which of the following is MOST important to monitor during the infusion? Continuous ECG Glasgow Coma Scale Urine specific gravity Serum potassium

Continuous ECG Procainamide is well-tolerated and is a first-line agent for the treatment of acute, undiagnosed, wide-complex tachycardia. Procainamide blocks sodium channels and also has an effect on potassium channels and so prolongs the effective refractory period, decreases automaticity and slows conduction. The client's ECG should be constantly monitored for cardiotoxicity: widening of QRS (> 50%) and prolonged QT interval, as well as hypotension.

Which of the following classes of drugs is most widely used in the treatment of cardiomyopathy?

Correct Answer: B. Beta-adrenergic blockers By decreasing the heart rate and contractility, beta-adrenergic blockers improve myocardial filling and cardiac output, which are primary goals in the treatment of cardiomyopathy. Option A: Antihypertensives aren't usually indicated because they would decrease cardiac output in clients who are often already hypotensive. Many antihypertensive drugs have their primary action on systemic vascular resistance. Some of these drugs produce vasodilation by interfering with sympathetic adrenergic vascular tone (sympatholytics) or by blocking the formation of angiotensin II or its vascular receptors. Option C: Calcium channel blockers are sometimes used for the same reasons as beta-adrenergic blockers; however, they aren't as effective as beta-adrenergic blockers and cause increased hypotension. These channels are responsible for regulating the influx of calcium into muscle cells, which in turn stimulates smooth muscle contraction and cardiac myocyte contraction. In cardiac nodal tissue, L-type calcium channels play an important role in pacemaker currents, and in phase 0 of the action potentials. Option D: Nitrates aren't' used because of their dilating effects, which would further compromise the myocardium. Nitrates exert their effects by dilating venous vessels, coronary arteries, and small arterioles; its maximal vasodilation is in the venous vessels.

When cleaning the perineal area around the site of an indwelling catheter, the nurse should vigorously wash the periurethral area. apply powder after giving perineal care. scrub the tubing toward the urinary meatus. wipe the catheter away from the urinary meatus.

Wipe the catheter away from the urinary meatus. -CORRECT The catheter should be wiped away from the meatus, to decrease the risk of introducing pathogens into the urinary tract. The perineum should be washed gently with soap and water. Powder can retain moisture, leading to an infection.

Which of the following cardiac conditions does a fourth heart sound (S4) indicate?

Correct Answer: D. Failure of the ventricle to eject all the blood during systole An S4 occurs as a result of increased resistance to ventricular filling after atrial contraction. This increased resistance is related to decreased compliance of the ventricle. Option A: A dilated aorta doesn't cause an extra heart sound, though it does cause a murmur. The aorta is considered pathologically dilated if the diameters of the ascending aorta and the aortic root exceed the norms for a given age and body size. A 50% increase over the normal diameter is considered aneurysmal dilatation. Option C: Decreased myocardial contractility is heard as a third heart sound. Optimal myocardial contractility is dependent on an optimal filling pressure, afterload, and the presence and availability of inotropic substances (eg, epinephrine, norepinephrine, or calcium). Calcium influx and binding to troponin C is essential for cardiac contraction. Option B: An S4 isn't heard in a normally functioning heart. The fourth heart sound is a low-pitched sound coincident with the late diastolic filling of the ventricle due to atrial contraction. It thus occurs shortly before the first heart sound. Although it is also called the atrial sound, and its production requires an effective atrial contraction, the fourth heart sound is the result of vibrations generated within the ventricle.

Which of the following drugs would be ordered by the physician to improve the platelet count in a male client with idiopathic thrombocytopenic purpura (ITP)? Acetylsalicylic acid (ASA) Corticosteroids Methotrexate Vitamin K

Corticosteroids. Can decrease antibody production and phagocytosis of the antibody-coated platelets, retaining more functioning platelets. A) ASA blocks prostaglandin aggregation for about 7-10 days (average platelet lifespan). C) Methotrexate inhibits enzymes responsible for nucleotide synthesis which prevents cell division and leads to anti-inflammatory actions. It causes thrombocytopenia. D) Vitamin K is used to treat an excessive anitcoagulate state from warfarin overload.

A female client is scheduled to receive a heart valve replacement with a porcine valve. Which of the following types of transplant is this? Allogeneic Autologous Syngeneic Xenogeneic

Xenogeneic is between a human and another species. A) Allogeneic is between 2 humans B) Autologous is from the same individual C) Syngeneic is between identical twins.

A nurse is administering IV furosemide to a patient admitted with congestive heart failure. After the infusion, which of the following symptoms is not expected? A. Increased urinary output B. Decreased edema C. Decreased pain D. Decreased blood pressure

Decreased pain Furosemide, a loop diuretic, does not alter pain. The Food and Drug Administration (FDA) has approved the use of furosemide in the treatment of conditions with volume overload and edema secondary to congestive heart failure exacerbation, liver failure, or renal failure including the nephrotic syndrome. Option A: Furosemide acts on the kidneys to increase urinary output. Furosemide inhibits tubular reabsorption of sodium and chloride in the proximal and distal tubules, as well as in the thick ascending loop of Henle by inhibiting sodium-chloride cotransport system resulting in excessive excretion of water along with sodium, chloride, magnesium, and calcium. Option B: Fluid may move from the periphery, decreasing edema. Careful monitoring of the clinical condition of the patient, daily weight, fluids intake, and urine output, electrolytes, i.e., potassium and magnesium, kidney function monitoring with serum creatinine and serum blood urea nitrogen level is vital to monitor the response to furosemide. Replete electrolytes if indicated as diuresis with furosemide lead to electrolyte depletion, and adjust the dose or even hold off on furosemide if laboratory work shows signs of kidney dysfunction. Option D: Fluid load is reduced, lowering blood pressure. Furosemide can be a second-line agent in heart failure patients with symptoms, and in patients with advanced kidney disease with an estimated glomerular filtration rate, less than 30 ml per minute the loop diuretics (furosemide) are preferred over thiazide diuretics to treat hypertension.

JP has been diagnosed with gout and wants to know why colchicine is used in the treatment of gout. Which of the following actions of colchicines explains why it's effective for gout? A. Replaces estrogen. B. Decreases infection. C. Decreases inflammation. D. Decreases bone demineralization.

Decreases inflammation. The action of colchicines is to decrease inflammation by reducing the migration of leukocytes to synovial fluid. Option A: Colchicine does not replace estrogen. Colchicine works by reducing the inflammation caused by crystals of uric acid in the joints. Option B: Decreasing infection is not a mechanism of action of colchicine. The primary mechanism of action of colchicine is tubulin disruption. This leads to subsequent downregulation of multiple inflammatory pathways and modulation of innate immunity. Option D: Colchicine doesn't decrease bone demineralization. The toxic effects of colchicine are related to this anti-mitotic activity within proliferating tissue such as skin, hair, and bone marrow.

Toxicity from which of the following medications may cause a client to see a green halo around lights? Digoxin Furosemide Metoprolol Enalapril

Digoxin One of the most common signs of digoxin toxicity is the visual disturbance known as the green halo sign B) Furosemide does not cause this kind of toxicity. The principal signs and symptoms of overdose with furosemide are dehydration, blood volume reduction, hypotension, electrolyte imbalance, hypokalemia, and hypochloremic alkalosis, and are extensions of its diuretic action. C) Metoprolol is not associated with this effect. Poisoning due to an overdose of metoprolol may lead to severe hypotension, sinus bradycardia, atrioventricular block, heart failure, cardiogenic shock, cardiac arrest, bronchospasm, impairment of consciousness, coma, nausea, vomiting, cyanosis, hypoglycemia, and, occasionally, hyperkalemia. D) This medication isn't associated with such an effect. While there is limited data about enalapril overdose in humans, overdosage may result in marked hypotension and stupor based on the pharmacological properties of the drug. The most common adverse effects of enalapril include cough, hypotension, stupor, headache, dizziness, and fatigue.

Which of the following types of cardiomyopathy can be associated with childbirth? Dilated Hypertrophic Myocarditis Restrictive

Dilated Although the cause isn't entirely known, cardiac dilation and heart failure may develop during the last month of pregnancy of the first few months after birth. The condition may result from a pre-existing cardiomyopathy not apparent prior to pregnancy. Option B: Hypertrophic cardiomyopathy is an abnormal symmetry of the ventricles that has an unknown etiology but a strong familial tendency. Option C: Myocarditis isn't specifically associated with childbirth. Myocarditis is an inflammation of the heart muscle (myocardium). Myocarditis can affect the heart muscle and the heart's electrical system, reducing the heart's ability to pump and causing rapid or abnormal heart rhythms (arrhythmias). Option D: Restrictive cardiomyopathy indicates constrictive pericarditis; the underlying cause is usually myocardial. Restrictive cardiomyopathy (RCM) is a rare disease of the myocardium and is the least common of the three clinically recognized and described cardiomyopathies. It is characterized by diastolic dysfunction with restrictive ventricular physiology, whereas systolic function often remains normal. Atrial enlargement occurs due to impaired ventricular filling during diastole, but the volume and wall thickness of the ventricles are usually normal.

A client with a missed abortion at 29 weeks gestation is admitted to the hospital. The client will most likely be treated with: A. Magnesium sulfate B. Calcium gluconate C. Dinoprostone (Prostin E.) D. Bromocriptine (Parlodel)

Dinoprostone (Prostin E.) The client with a missed abortion will have induction of labor. Prostin E. is a form of prostaglandin used to soften the cervix. Prostaglandin E2 (PGE2), also known by the name dinoprostone, is a naturally occurring compound that is involved in promoting labor, though it is also present in the inflammatory pathway. Prostaglandin E2 is FDA approved for cervical ripening for the induction of labor in patients for which there is a medical indication for induction.Option A: Magnesium sulfate is used for preterm labor and preeclampsia. Magnesium sulfate is a naturally occurring mineral used to control low blood levels of magnesium. Magnesium injection is also used for pediatric acute nephritis and to prevent seizures in severe pre-eclampsia, eclampsia, or toxemia of pregnancy.Option B: Calcium gluconate is the antidote for magnesium sulfate. Calcium gluconate is typically administered intravenously (IV) or orally in the treatment of hypocalcemia, cardiac arrest, or cardiotoxicity due to hyperkalemia or hypermagnesemia. Calcium gluconate has also been used off-label in the management of ?-blocker toxicity, calcium-channel blocker (CCB) toxicity, magnesium toxicity, and hydrofluoric acid burns.Option D: Pardel is a dopamine receptor stimulant used to treat Parkinson's disease. Bromocriptine is also used as an early treatment for PD to delay the onset of the use of levodopa, ultimately delaying the likely dyskinesia and motor fluctuations that occur with chronic use.

In which of the following areas is an abdominal aortic aneurysm most commonly located? Distal to the iliac arteries Distal to the renal arteries Adjacent to the aortic branch Proximal to the renal arteries

Distal to the renal arteries The portion of the aorta distal to the renal arteries is more prone to an aneurysm because the vessel isn't surrounded by stable structures, unlike the proximal portion of the aorta. A) Distal to the iliac arteries, the vessel is again surrounded by stable vasculature, making this an uncommon site for an aneurysm. C) There is no area adjacent to the aortic arch, which bends into the thoracic (descending) aorta. D) The proximal portion is surrounded by stable structures, lessening the risk for rupture.

A patient is admitted to the hospital for routine elective surgery. Included in the list of current medications is warfarin (Coumadin) at a high dose. Concerned about the possible effects of the drug, particularly in a patient scheduled for surgery, the nurse anticipates which of the following actions? A. Draw a blood sample for prothrombin (PT) and international normalized ratio (INR) level. B. Administer vitamin K. C. Draw a blood sample for type and crossmatch and request blood from the blood bank. D. Cancel the surgery after the patient reports stopping the Coumadin one week previously.

Draw a blood sample for prothrombin (PT) and international normalized ratio (INR) level. The effect of Coumadin is to inhibit clotting. The next step is to check the PT and INR to determine the patient's anticoagulation status and risk of bleeding. Patients receiving treatment with warfarin should have close monitoring to ensure the safety and efficacy of the medication. Periodic blood testing is the recommendation to assess the patient's prothrombin time (PT) and the international normalized ratio (INR). Option B: Vitamin K is an antidote to Coumadin and may be used in a patient who is at imminent risk of dangerous bleeding. When managing warfarin toxicity, the initial step would be to discontinue warfarin and then administer vitamin K (phytonadione). The vitamin K may administration can be either via the oral, intravenous, or subcutaneous route. However, the initial administration of oral vitamin K is often preferable in patients without major bleeding or extremely elevated INR. Option C: Preparation for transfusion, as described in option C, is only indicated in the case of significant blood loss. A reduction in INR should occur within 24 hours of administration. After that, the recommendation is for intravenous vitamin K administration if necessary. Subcutaneous vitamin K is often not recommended for warfarin toxicity or reversal due to erratic and unpredictable absorption. Administering agents like prothrombin complex concentrate (PCC), fresh frozen plasma (FFP), and activated Factor VII may be considered for cases of significant bleeding. Option D: If lab results indicate an anticoagulation level that would place the patient at risk of excessive bleeding, the surgeon may choose to delay surgery and discontinue the medication. Patients also require close monitoring for signs and symptoms of active bleeding throughout their treatment. Close monitoring for signs and symptoms of bleeding, such as dark tarry stools, nosebleeds, and hematomas, is necessary. The patient's hemoglobin and hematocrit level should undergo an assessment before initiating warfarin and approximately every six months while on therapy.

A child has recently been diagnosed with Duchenne's muscular dystrophy. The parents are receiving genetic counseling prior to planning another pregnancy. Which of the following statements includes the most accurate information? A. Duchenne's is an X-linked recessive disorder, so daughters have a 50% chance of being carriers and sons a 50% chance of developing the disease. B. Duchenne's is an X-linked recessive disorder, so both daughters and sons have a 50% chance of developing the disease. C. Each child has a 1 in 4 (25%) chance of developing the disorder. D. Sons only have a 1 in 4 (25%) chance of developing the disorder.

Duchenne's is an X-linked recessive disorder, so daughters have a 50% chance of being carriers and sons a 50% chance of developing the disease. The recessive Duchenne gene is located on one of the two X chromosomes of a female carrier. DMD is a genetic disease due to the mutation of the dystrophin gene, located on chromosome Xp21. It is inherited as an X- linked recessive trait; however, approximately 30% of cases are due to new mutations. Carrier females show no evidence of muscular weakness; however, symptomatic female carriers have been described. About 2.5% to 20% of female carriers may be affected. This can be explained by the Lyon hypothesis in which the normal X chromosome becomes inactivated, and the X chromosome with the mutation is expressed. Option B: If her son receives the X bearing the gene he will be affected. Female carriers can become symptomatic if they are associated with Turner's syndrome (45X) or mosaic Turner karyotype, balanced X autosome translocations with breakpoints within the dystrophin gene and preferential inactivation of the normal X, and females with a normal karyotype but with nonrandom X chromosome inactivation with diminished expression of the normal dystrophin allele. Option C: Daughters are not affected, but 50% are carriers because they inherit one copy of the defective gene from the mother. Mutations in the dystrophin gene result in diseases known as dystrophinopathies, which encompass Duchenne muscular dystrophy, Becker muscular dystrophy, and an intermediate form. Option D: There is a 50% chance of a son being affected. Mutations result in a limited production of the dystrophin protein, which results in loss of the myofiber membrane integrity with repeated cycles of necrosis and regeneration. Fibrous connective tissue and fat progressively replace muscle leading to clinical features.

When do coronary arteries primarily receive blood flow? During inspiration During diastole During expiration During systole

During diastole A) Breathing patterns are irrelevant to blood flow. It has been suggested that the diaphragm will preferentially steal blood flow from working locomotor muscles during increased activity. In healthy adults, the cost of breathing is <5% of the total oxygen consumption at low-level exercise but approaches 15% during heavy exercise in young athletes or older fit subjects. Further, reflex vasoconstriction of the locomotor muscles is evident when a substantial respiratory load is applied sufficient to elicit diaphragm fatigue. C) Expiration is not related to the blood flow. The pulmonary system is intimately linked with the cardiovascular system anatomically and hemodynamically and plays a significant role in exercise intolerance through a number of mechanisms. D) There is a little portion of the blood that the coronary arteries receive during systole. During systole, intramuscular blood vessels are compressed and twisted by the contracting heart muscle and blood flow to the left ventricle at its lowest. The force is greatest in the subendocardial layers where it approximates to intramyocardial pressure.

A female client arrives at the ED with chest and stomach pain and a report of black tarry stool for several months. Which of the following orders should the nurse anticipate? Cardiac monitor, oxygen, creatine kinase and lactate dehydrogenase levels Prothrombin time, partial thromboplastin time, fibrinogen and fibrin split product values Electrocardiogram, CBC, testing for occult blood, comprehensive serum metabolic panel Electroencephalogram, alkaline phosphatase, and aspartate aminotransferase levels, basic serum metabolic panel

Electrocardiogram, CBC, testing for occult blood, comprehensive serum metabolic panel. An electrocardiogram evaluates the complaints of chest pain, laboratory tests determine anemia and the stool test for occult blood determines blood in the stool.

Mr. Gonzales was admitted to the hospital with ascites and jaundice. To rule out cirrhosis of the liver which laboratory test indicates liver cirrhosis? A. Decreased red blood cell count B. Decreased serum acid phosphatase level C. Elevated white blood cell count D. Elevated serum aminotransferase

Elevated serum aminotransferase Hepatic cell death causes the release of liver enzymes alanine aminotransferase (ALT), aspartate aminotransferase (AST), and lactate dehydrogenase (LDH) into the circulation. Liver cirrhosis is a chronic and irreversible disease of the liver characterized by generalized inflammation and fibrosis of the liver tissues. Option A: Decreased red blood cell count may indicate anemia. Option B: Serum acid phosphatase is an enzyme that acts to liberate phosphate under acidic conditions. Until now, low values cannot be determined with certainty. Option C: A high white blood cell count usually indicates increased production of white blood cells to fight infection.

A patient is admitted to the hospital with a diagnosis of primary hyperparathyroidism. A nurse checking the patient's lab results would expect which of the following changes in laboratory findings? A. Elevated serum calcium B. Low serum parathyroid hormone (PTH) C. Elevated serum vitamin D D. Low urine calcium

Elevated serum calcium The parathyroid glands regulate the calcium level in the blood. In hyperparathyroidism, the serum calcium level will be elevated. A normal PTH in the presence of hypercalcemia is considered inappropriate and still consistent with PTH-dependent hypercalcemia. PTH levels should be very low in those patients with PTH-independent hypercalcemia. A comprehensive clinical evaluation complemented by routine laboratory and radiologic studies should be sufficient to establish a diagnosis of primary hyperparathyroidism in a patient with persistent hypercalcemia and an elevated serum level of parathyroid hormone. Option B: Parathyroid hormone levels may be high or normal but not low. Patients with primary hyperparathyroidism and other causes of PTH-dependent hypercalcemia often have frankly elevated levels of PTH, while some will have values that fall within the reference range for the general population. It is uncommon for clinically occult malignancies to cause hypercalcemia. Most patients with malignancy-associated hypercalcemia are known to have cancer, or cancer is readily detectable on initial evaluation, and PTH levels will be suppressed. Option C: Parathyroid hormone levels may be high or normal but not low. The body will lower the level of vitamin D in an attempt to lower calcium. Option D: Urine calcium may be elevated, with calcium spilling over from elevated serum levels. This may cause renal stones. A review of previous medical records can often be of significant value in establishing the cause of hypercalcemia. Most patients with hyperparathyroidism have persistent or intermittent hypercalcemia for many years before a definitive diagnosis is established.

The term "pink puffer" refers to the female client with which of the following conditions? A. Adult respiratory distress syndrome (ARDS) B. Asthma C. Chronic obstructive bronchitis D. Emphysema

Emphysema Because of the large amount of energy it takes to breathe, clients with emphysema are usually cachectic. They're pink and usually breathe through pursed lips, hence the term "puffer." Option A: Clients with ARDS are usually acutely short of breath. Option B: Clients with asthma don't have any particular characteristics. Option C: Clients with chronic obstructive bronchitis are bloated and cyanotic in appearance.

Which of the following actions is the first priority care for a client exhibiting signs and symptoms of coronary artery disease? Decrease anxiety. Enhance myocardial oxygenation. Administer sublingual nitroglycerin. Educate the client about his symptoms.

Enhance myocardial oxygenation. Enhancing myocardial oxygenation is always the first priority when a client exhibits signs and symptoms of cardiac compromise. Without adequate oxygen, the myocardium suffers damage. A) Decreasing the client's anxiety is also important, but it is not the priority. When someone is anxious, their body reacts in ways that can put an extra strain on their heart. The physical symptoms of anxiety can be especially damaging among individuals with existing cardiac disease. C) Sublingual nitroglycerin is administered to treat acute angina, but its administration isn't the first priority. Although nitroglycerin has a vasodilatory effect in both arteries and veins, the profound desires effects caused by nitroglycerin are primarily due to venodilation. Venodilation causes pooling of blood within the venous system, reducing preload to the heart, which causes a decrease in cardiac work, reducing anginal symptoms secondary to demand ischemia. D) Although educating the client is important in care delivery, it is not a priority when a client is compromised. Patient education promotes patient-centered care and increases adherence to medication and treatments. An increase in compliance leads to a more efficient and cost-effective healthcare delivery system. Educating patients ensures continuity of care and reduces complications related to the illness.

Instructions for a client with SLE would include information about which of the following blood dyscrasias? Dressler's syndrome Polycythemia Essential thrombocytopenia Von Willebrand's disease

Essential thrombocytopenia Essential thrombocytopenia is linked to immunologic disorders, such as SLE and the human immunodeficiency virus. Option A: Dressler's syndrome is pericarditis that occurs after myocardial infarction and isn't linked to SLE. Option B: Moderate to severe anemia is associated with SLE, not polycythemia. It is found in about 50% of patients, with anemia of chronic disease being the most common form. Option D: Von Willebrand disease is a blood disorder in which the blood doesn't clot properly. Blood contains many proteins that help the body stop bleeding. One of these proteins is called von Willebrand factor.

Nurse Linda is caring for a client with head injury and monitoring the client with decerebrate posturing. Which of the following is a characteristic of this type of posturing? A. Upper extremity flexion with lower extremity flexion B. Upper extremity flexion with lower extremity extension C. Extension of the extremities after a stimulus D. Flexion of the extremities after stimulus

Extension of the extremities after a stimulus Decerebrate posturing is the extension of the extremities after a stimulus which may occur with upper brain stem injury. Decerebrate posturing is described as adduction and internal rotation of the shoulder, extension at the elbows with pronation of the forearm, and flexion of the fingers. Option A: Decerebrate posturing is the extension, not flexion, of extremities. As with decorticate posturing, the lower limbs show extension and internal rotation at the hip, with the extension of the knee and plantar flexion of the feet. Toes are typically abducted and hyperextended. Option B: The upper extremity should be in extension as well as the lower extremity. Decerebrate posturing can be seen in patients with large bilateral forebrain lesions with progression caudally into the diencephalon and midbrain. It can also be caused by a posterior fossa lesion compressing the midbrain or rostral pons. Option D: There is an extension of extremities after a stimulus in decerebrate posturing. Teasdale and Jennett advocated not using the term 'decerebrate' in the assessment of coma due to its association with a specific physio anatomical correlation, but to rather use the term 'extension.'

A patient in the cardiac unit is concerned about the risk factors associated with atherosclerosis. Which of the following are hereditary risk factors for developing atherosclerosis? A. Family history of heart disease B. Overweight C. Smoking D. Age

Family history of heart disease Family history of heart disease is an inherited risk factor that is not subject to a lifestyle change. Having a first-degree relative with heart disease has been shown to significantly increase risk. ASCVD is multifactorial etiology. The most common risk factors include hypercholesterolemia (LDL-cholesterol), hypertension, diabetes mellitus, cigarette smoking, age (male older than 45 years and female older than 55 years), male gender, and strong family history (male relative younger than 55 years and female relative younger than 65 years). Option B: Also, a sedentary lifestyle, obesity, diets high in saturated and trans-fatty acids, and certain genetic mutations contribute to risk. While a low level of high-density lipoprotein (HDL)-cholesterol is considered a risk factor, pharmacological therapy increasing HDL-cholesterol has yielded negative results raising concerns about the role of HDL in ASCVD. Option C: Smoking is a risk factor that is subject to lifestyle change and can reduce risk significantly. For the most part atherosclerosis and its pathology can be prevented. All healthcare workers who look after patients should educate patients on the need to exercise regularly, discontinue smoking, maintain healthy body weight, eat a healthy diet, and remain compliant with the medications used to lower lipids. Option D: Advancing age increases the risk of atherosclerosis but is not a hereditary factor. It has been reported that 75% of acute myocardial infarctions occur from plaque rupture and the highest incidence of plaque rupture was observed in men over 45 years; whereas, in women, the incidence increases beyond age 50 years

A 67-year-old client develops acute shortness of breath and progressive hypoxia requiring right femur. The hypoxia was probably caused by which of the following conditions? A. Asthma attack B. Atelectasis C. Bronchitis D. Fat embolism

Fat embolism Long bone fractures are correlated with fat emboli, which cause shortness of breath and hypoxia. Option A: Asthma attacks do not develop following a femoral fracture. Option B: He could develop atelectasis but it typically doesn't produce progressive hypoxia. Option C: It's unlikely the client has developed bronchitis without a previous history.

A client recently started on hemodialysis wants to know how the dialysis will take the place of his kidneys. The nurse's response is based on the knowledge that hemodialysis works by: A. Passing water through a dialyzing membrane B. Eliminating plasma proteins from the blood C. Lowering the pH by removing nonvolatile acids D. Filtering waste through a dialyzing membrane

Filtering waste through a dialyzing membrane Hemodialysis works by using a dialyzing membrane to filter waste that has accumulated in the blood. Dialysis largely replicates the functions of the kidneys in patients with chronic kidney failure. Hemodialysis takes over the key tasks of the kidneys, removing waste materials, toxins, excess salt and fluids from the body. Option A: In hemodialysis, an artificial kidney (hemodialyzer) is used to remove waste and extra chemicals and fluid from the blood. To get the blood into the artificial kidney, the doctor needs to make an access (entrance) into the blood vessels. This is done by minor surgery to the arm or leg. Option B: The transfer of metabolic toxins through the membrane into the dialysis fluid is based on natural processes. This process is known as diffusion. When blood and dialysis fluid with different concentrations of molecules are separated by a semipermeable membrane, the molecules move through the membrane to the lower concentration. However, large proteins and blood cells are too big to pass through the small membrane-pores, so they remain in the blood. Option C: A dialyzer is an artificial filter containing fine fibers. The fibers are hollow with microscopic pores in the wall, also known as semi-permeable dialysis membrane. To remove toxins during hemodialysis, a special dialysis-fluid flows through the filter, and bathes the fibers from the outside, while the blood flows through the hollow fiber. Due to the semi-permeable dialysis membrane, toxins, urea and other small particles can pass through the membrane.

A hospitalized patient has received transfusions of 2 units of blood over the past few hours. A nurse enters the room to find the patient sitting up in bed, dyspneic and uncomfortable. On assessment, crackles are heard in the bases of both lungs, probably indicating that the patient is experiencing a complication of transfusion. Which of the following complications is most likely the cause of the patient's symptoms? A. Febrile non-hemolytic reaction B. Allergic transfusion reaction C. Acute hemolytic reaction D. Fluid overload

Fluid overload Fluid overload occurs when the fluid volume infused over a short period is too great for the vascular system, causing fluid leak into the lungs. Symptoms include dyspnea, rapid respirations, and discomfort as in the patient described. Transfusion associated circulatory overload includes any four of the following occurring within 6 h of a BT - acute respiratory distress, tachycardia, increased blood pressure (BP), acute or worsening pulmonary edema and evidence of a positive fluid balance. Option A: Febrile non-hemolytic reaction results in fever. Febrile non-hemolytic is generally thought to be caused by cytokines released from blood donor leukocytes (white blood cells). Transfusion reactions range in frequency from relatively common (mild allergic and febrile non-hemolytic reactions) to rare (anaphylaxis, acute hemolytic, and sepsis). Option B: Symptoms of allergic transfusion reaction would include flushing, itching, and a generalized rash. Attributed to hypersensitivity to a foreign protein in the donor product. The severity and incidence vary depending on the type of transfusion reaction, the prevalence of disease in the donor population, and the extent of follow-up care the patient receives. Option C: Acute hemolytic reaction may occur when a patient receives blood that is incompatible with his blood type. It is the most serious adverse transfusion reaction and can cause shock and death. Can result in intravascular or extravascular hemolysis, depending on the specific etiology (cause). Immune-mediated reactions are often a result of recipient antibodies present to blood donor antigens.

Patrick is treated in the emergency department for a Colles' fracture sustained during a fall. What is a Colles' fracture? A. Fracture of the distal radius. B. Fracture of the olecranon. C. Fracture of the humerus. D. Fracture of the carpal scaphoid.

Fracture of the distal radius. Colles' fracture is a fracture of the distal radius, such as from a fall on an outstretched hand. It's most common in women. Option B: Colles' fracture does not involve the olecranon. Most Colles fractures are secondary to a fall on an outstretched hand with a pronated forearm in dorsiflexion. Option C: The humerus is not affected in Colles' fracture. It is a complete fracture of the radius bone of the forearm close to the wrist resulting in an upward displacement of the radius and obvious deformity. Option D: Colles' fracture doesn't refer to a fracture of the carpal scaphoid. A scaphoid fracture is a break in one of the small bones of the wrist.

During the first several hours after a cardiac catheterization, it would be most essential for nurse Cherry to: A. Elevate the client's bed at 45°. B. Instruct the client to cough and deep breathe every 2 hours. C. Frequently monitor the client's apical pulse and blood pressure. D. Monitor client's temperature every hour.

Frequently monitor the client's apical pulse and blood pressure. Blood pressure is monitored to detect hypotension which may indicate shock or hemorrhage. The apical pulse is taken to detect arrhythmias related to cardiac irritability. During the procedure, a nurse is assigned to monitor vital signs. After the procedure, the nurse is also responsible for ensuring that the access site is not bleeding and the distal extremity pulses are intact. Option A: Patients should be kept lying flat for several hours after the procedure so that any serious bleeding can be avoided. Identification of the bleeding source is essential for patients with continued hemodynamic deterioration. These life-threatening bleeds are more frequent when the artery is punctured above the inguinal ligament. Most patients are managed with a reversal of anticoagulation, application of manual compression and volume resuscitation, and observation. Option B: Coughing and deep breathing every 2 hours is unnecessary. For any patient, the complication rate is dependent on multiple factors and is dependent on the demographics of the patient, vascular anatomy, comorbid conditions, clinical presentation, the procedure being performed, and the experience of the operator. The complications can be minor as discomfort at the site of catheterization to major ones like death. Option D: Temperature can be monitored for every shift. Cardiac catheterization is performed using sterile technique, and local or systemic infection is extremely rare. Routine prophylaxis for endocarditis is not recommended during cardiac catheterization procedures.

The physician has ordered a minimal-bacteria diet for a client with neutropenia. The client should be taught to avoid eating: A. Packed fruits B. Salt C. Fresh raw pepper D. Ketchup

Fresh raw pepper Fresh raw or whole pepper is not allowed unless thoroughly cooked in food. A low-bacteria diet is designed to reduce exposure to bacteria and other pathogens that can make one sick. It's often prescribed for people who are at a greater risk of infection because they're currently not making enough white blood cells due to certain illnesses or medical treatments. Option A: Canned fruits are allowed since they are processed and pasteurized. Fresh fruits and vegetables are fine as long as they are washed first or cooked thoroughly. Meat, fish, and eggs should also be fully cooked. Commercially prepared and packaged foods are acceptable but avoid buying foods indented and swollen cans or damaged packaging. Option B: Salt is allowed. The keys to a low-bacteria diet are choosing foods that are less likely to carry bacteria while avoiding the foods that do. Frequent hand washing and paying particular attention to food safety practices are also essential. Option D: Ketchup is also allowed. Bread, ready-to-eat cereals, pancakes, waffles, and crackers are safe to eat. Bottled beverages, hot beverages, and pasteurized fruit and vegetable juices are good as well. Cream cheese, sour cream, mayonnaise, margarine, commercial peanut butter, and chocolate are okay, too.

The mother of a 14-month-old child reports to the nurse that her child will not fall asleep at night without a bottle of milk in the crib and often wakes during the night asking for another. Which of the following instructions by the nurse is correct? A. Allow the child to have the bottle at bedtime, but withhold the one later in the night. B. Put juice in the bottle instead of milk. C. Give only a bottle of water at bedtime. D. Do not allow bottles in the crib.

Give only a bottle of water at bedtime. Babies and toddlers should not fall asleep with bottles containing liquid other than plain water due to the risk of dental decay. Wean one ounce a night. Let's say the child takes three 4 oz bottles a night. Take the last bottle and reduce it by an oz on night one. On night 2, reduce bottle 2 by 1 oz. On night 3 reduce Bottle #1 by 1 oz. When a bottle gets down to 2 oz, substitute a bottle of water. After this step, get rid of the bottle. Don't ever wake up the child if they sleep through a feeding- that is the goal. Option A: If they skip a feeding one night but wake up the following night for that feeding, it is OK to give them the scheduled bottle. Limit the water bottles to 2 oz, simply to reduce the amount of urine produced and wet diapers to deal with. If the child doesn't want the water, that is fine. But don't give in and give the milk. Option B: Sugars in juice remain in the mouth during sleep and cause caries, even in teeth that have not yet erupted. Make slow incremental changes over time. These changes are relatively easy to make and the child will tolerate them well. Option D: The child could have a bottle of water in the crib with close supervision. Bottle fed infants typically can wean off night feeding by 6 months of age. Breast fed infants tend to take longer, up to a year of age. The American Academy of Pediatrics recommends exclusive breastfeeding for six months, with the addition of complementary foods continuing up to a year, or longer "as desired by mother and infant". It's important to note that night weaning can lead to weaning altogether.

A newly diagnosed 8-year-old child with type I diabetes mellitus and his mother are receiving diabetes education prior to discharge. The physician has prescribed Glucagon for emergency use. The mother asks the purpose of this medication. Which of the following statements by the nurse is correct? A. Glucagon enhances the effect of insulin in case the blood sugar remains high one hour after injection. B. Glucagon treats hypoglycemia resulting from insulin overdose. C. Glucagon treats lipoatrophy from insulin injections. D. Glucagon prolongs the effect of insulin, allowing fewer injections.

Glucagon treats hypoglycemia resulting from insulin overdose. Glucagon is given to treat insulin overdose in an unresponsive patient. Patients with decreased levels of consciousness cannot safely consume the oral carbohydrates needed to raise their blood sugar without risk of aspiration, and obtaining IV access can be problematic in the diabetic population, which can prevent prompt administration of IV glucose. Option A: Following Glucagon administration, the patient should respond within 15-20 minutes at which time oral carbohydrates should be given. Similar to epinephrine auto-injectors, a pre-filled glucagon injector has received approval, which injects into the patient's thigh. Healthcare providers will most often encounter the emergency kit formulation necessitating reconstitution before injecting the medication, but the intranasal spray may be encountered with increasing frequency in hypoglycemia kits for layperson use. Option C: Lipoatrophy refers to the effect of repeated insulin injections on subcutaneous fat. Lipoatrophies are considered an adverse immunological side effect of insulin therapy, and in some cases they are mediated by a local high production of tumor necrosis factor-?, which leads to a dedifferentiation of adipocytes in the subcutaneous tissue. Option D: Glucagon reverses rather than enhances or prolongs the effects of insulin. Glucagon is a reliable method of raising the patient's glucose and relieving severe hypoglycemia long enough for more definitive correction of the patient's glucose levels by mouth, particularly when IV access is unavailable to the provider or has failed.

A patient is prescribed a calcium channel blocker to treat primary hypertension. When teaching the patient about the medication, which of these foods will the healthcare provider advise the patient to avoid? Eggs Grapefruit Milk Bananas

Grapefruit Grapefruit and its juice contain furanocoumarins, which inhibit the cytochrome P450 enzyme CYP3A4. CYP3A4 is involved in metabolizing many drugs, including calcium channel blockers. Medication blood levels can increase, becoming toxic. The levels of calcium channel blockers are increased when grapefruit or grapefruit juice is consumed, potentially causing hypotension. Grapefruit can interfere with other drugs too, including statins (atorvastatin, lovastatin, simvastatin), some antibiotics, and some cancer drugs.

A 32- year-old pregnant woman come to the clinic for her prenatal visit. The nurse gathers data about her obstetric history, which includes 3-year-old twins at home and a miscarriage 10 years ago at 12 weeks gestation. How would the nurse accurately document this information? Fill in the blanks.

Gravitas 3 Para 1 Gravida is the number of confirmed pregnancies and each pregnancy is only counted one time, even if the pregnancy was a multiple gestation (I.e., twins, triplets). Para (parity) indicates the total number of pregnancies that have reached viability (20 weeks) regardless of whether the infants were born alive. Thus, for this woman, she is now pregnant, had 2 prior pregnancies, and 1 viable birth (twins).

A client is discharged home with a prescription for Coumadin (sodium warfarin). The client should be instructed to: A. Have a Protime done monthly B. Eat more fruits and vegetables C. Drink more liquids D. Avoid crowds

Have a Protime done monthly Coumadin is an anticoagulant. One of the tests for bleeding time is a Protime. This test should be done monthly. The client will need to have his blood tested to tell how well the medication is working. The blood test, called prothrombin time (PT or protime), is used to calculate the International Normalized Ratio (INR). INR helps the healthcare provider determine how well warfarin is working to prevent blood clots and if the dose needs to be adjusted. Option B: Eating more fruits and vegetables is not necessary, and dark-green vegetables contain vitamin K, which increases clotting. Vitamin K is needed for normal blood clotting. However, large changes in the amount of vitamin K in the diet can change the way warfarin works. If the client eats foods high in vitamin K, it's important to keep a weekly intake of vitamin K-containing foods consistent. Option C: Drinking more liquids could boost the platelet count and increase the body's immunity. Do not start consuming the following herbal teas and supplements because they may affect the INR, causing it to be too high or too low. If the client drinks tea, black tea (such as orange pekoe tea) is acceptable because it is not high in Vitamin K. Option D: Avoiding crowds is important for patients with decreased WBC. Stay away from people who are ill. Avoid contact with anyone who has recently been vaccinated, including infants and children. Avoid crowds as much as possible. When going to places where there are often a lot of people (i.e., church, shopping), try going at off-peak times, when they are not as crowded.

A client with cancer of the pancreas has undergone a Whipple procedure. The nurse is aware that during the Whipple procedure, the doctor will remove the: A. Head of the pancreas B. Proximal third section of the small intestines C. Stomach and duodenum D. Esophagus and jejunum

Head of the pancreas During a Whipple procedure the head of the pancreas, which is a part of the stomach, the jejunum, and a portion of the stomach is removed and anastomosed. It is the most often used surgery to treat pancreatic cancer that's confined to the head of the pancreas. After performing the Whipple procedure, the surgeon reconnects the remaining organs to allow the client to digest food normally after surgery. Option B: Small bowel resection is surgery to remove a part of the small bowel. It is done when part of the small bowel is blocked or diseased. Option C: A gastrectomy is the surgical removal of all or part of the stomach. The stomach is a J-shaped organ in the upper abdomen. It is part of the digestive system, which processes nutrients ( vitamins, minerals, carbohydrates, fats, proteins, and water) in foods that are eaten and help pass waste material out of the body. A partial gastrectomy is the removal of only part of the stomach. The remaining portion then continues with its digestive role. If the entire stomach is removed, the esophagus is connected directly to the small intestine, where digestion now begins. Patients must make significant dietary changes when a gastrectomy is performed. Option D: Jejunal interposition is a procedure in which surgeons replace the missing section of a child's esophagus with a section of the jejunum (the middle part of the small intestine). It is used to treat children who have already undergone failed repair of long-gap esophageal atresia and for whom the Foker process isn't an option

Olivia had coronary artery bypass graft (CABG) surgery 3 days ago. Which of the following conditions is suspected by the nurse when a decrease in platelet count from 230,000 ul to 5,000 ul is noted? Pancytopenia Idiopathic thrombocytopenic purpura (ITP) Disseminated intravascular coagulation (DIC) Heparin-associated thrombosis and thrombocytopenia (HATT)

Heparin-associated thrombosis and thrombocytopenia (HATT) may occur after CABG surgery due to heparin use during surgery. B) ITP is a bleeding condition of unknown cause in which the blood fails to clot adequately because of a low circulating platelet count and shortened platelet lifespan. C) Although DIC causes platelet aggregation and bleeding, it is common in a client after revascularization surgery. A) Pancytopenia is a reduction in all blood cells.

A patient arrives at the emergency department complaining of back pain. He reports taking at least 3 acetaminophen tablets every three hours for the past week without relief. Which of the following symptoms suggests acetaminophen toxicity? A. Tinnitus B. Diarrhea C. Hypertension D. Hepatic damage

Hepatic damage Acetaminophen in even moderately large doses can cause serious liver damage that may result in death. Immediate evaluation of liver function is indicated with consideration of N-acetylcysteine administration as an antidote. Acetaminophen is rapidly absorbed from the gastrointestinal (GI) tract and reaches therapeutic levels in 30 minutes to 2 hours. Overdose levels peak at 4 hours unless other factors could delay gastric emptying, such as a co-ingestion of an agent that slows gastric motility, or if the acetaminophen is in an extended-release form. Option A: Tinnitus is associated with aspirin overdose, not acetaminophen. Acetaminophen has an elimination half-life of 2 hours, but can be as long as 17 hours in patients with hepatic dysfunction. It is metabolized by the liver, where it is conjugated to nontoxic, water-soluble metabolites that are excreted in the urine. Option B: In the third stage (72 hours to 96 hours), liver dysfunction is significant with renal failure, coagulopathies, metabolic acidosis, and encephalopathy. Gastrointestinal (GI) symptoms reappear, and death is most common at this stage. The fourth stage (4 days to 3 weeks) is marked by recovery. Option C: Diarrhea and hypertension are not associated with acetaminophen. The diagnosis of acetaminophen toxicity is based on serum levels of the drug, even if there are no symptoms. Other laboratory studies needed include liver function tests (LFTs) and coagulation profile (PT/INR). If the ingestion is severe, LFTs can rise within 8 to 12 hours of ingestion. Normally LFTS remain elevated in the second stage at 18 to 72 hours.

Which of the following laboratory test results would suggest to the nurse Len that a client has a corticotropin-secreting pituitary adenoma? A. High corticotropin and low cortisol levels B. Low corticotropin and high cortisol levels C. High corticotropin and high cortisol levels D. Low corticotropin and low cortisol levels

High corticotropin and high cortisol levels A corticotropin-secreting pituitary tumor would cause high corticotropin and high cortisol levels. Option A: Cortisol levels should also be elevated in corticotropin-secreting pituitary adenoma. Option B: Low corticotropin and high cortisol levels would be seen if there was a primary defect in the adrenal glands. Option D: A low corticotropin level with a low cortisol level would be associated with hypocortisolism.

Tissue plasminogen activator (tPA) is considered for the treatment of a patient who arrives in the emergency department following the onset of symptoms of myocardial infarction. Which of the following is a contraindication for treatment with t-PA? A. Worsening chest pain that began earlier in the evening. B. History of cerebral hemorrhage. C. History of prior myocardial infarction. D. Hypertension.

History of cerebral hemorrhage. A history of cerebral hemorrhage is a contraindication to tPA because it may increase the risk of bleeding. Bleeding associated with alteplase therapy can be divided into two broad categories. Internal bleeding includes intracranial bleeding (0.4% to 15.4%), retroperitoneal bleeding (less than 1%), gastrointestinal (GI) bleeding (5%), genitourinary bleeding (4%), and respiratory bleeding. Superficial or surface bleeding is observed mainly at invaded or disturbed sites such as venous cutdowns, arterial punctures, and recent surgical intervention sites. Option A: TPA acts by dissolving the clot blocking the coronary artery and works best when administered within 6 hours of onset of symptoms. Alteplase acts within the endogenous fibrinolytic cascade to convert plasminogen to plasmin by hydrolyzing the arginine-valine bond in plasminogen. The activated plasmin then degrades fibrin and fibrinogen, allowing for the dissolution of the clot and re-establishment of blood flow. Option C: Prior MI is not a contraindication to tPA. FDA-approved indications for alteplase include pulmonary embolism, myocardial infarction with ST-segment elevation (STEMI), ischemic stroke when given within 3 hours of the start of symptoms, and re-establishment of patency in occluded intravenous (IV) catheters. Option D: Patients receiving tPA should be observed for changes in blood pressure, as tPA may cause hypotension. There are no therapeutic drug monitoring recommendations that pertain to the efficacy of tPA therapy. If prolonged off-label therapy is occurring in the event of catheter-directed treatment or repeated dosing in valve thrombosis, serial imaging of the thrombus is reasonable. The safety profile is best monitored by prothrombin time (PT), partial thromboplastin time (PTT), Hemoglobin, and hematocrit to assess ongoing bleeding.

Nurse Liza is assigned to care for a client who has returned to the nursing unit after left nephrectomy. Nurse Liza's highest priority would be... A. Hourly urine output B. Temperature C. Able to turn side to side D. Able to sips clear liquid

Hourly urine output After nephrectomy, it is necessary to measure urine output hourly. This is done to assess the effectiveness of the remaining kidney also to detect renal failure early. Immediately after surgery, the health care team will carefully watch the client's blood pressure, electrolytes and fluid balance. These body functions are controlled in part by the kidneys. The client will most likely have a urinary catheter (tube to drain urine) in the bladder for a short time during the recovery. Option B: Monitoring temperature regularly is recommended, but not the highest priority. The client may have discomfort and numbness (caused by severed nerves) near the incision area. Pain relievers are given after the surgical procedure and during the recovery period as needed. Although deep breathing and coughing may be painful because the incision is close to the diaphragm, breathing exercises are important to prevent pneumonia. Option C: The client would be able to turn to his sides with some assistance after surgery. The client will probably remain in the hospital for 1 to 7 days, depending on the method of surgery used. He will be encouraged to return to light activities as soon as he feels up to it. Strenuous activity and heavy lifting should be avoided for 6 weeks following the procedure. Option D: The client would be allowed to take sips of clear liquid as recommended by the physician. The doctor will give the client more detailed instructions about post-operative activities, restrictions and diet. Tests will be done on a regular basis to check how well the remaining kidney is working. A urinalysis (urine test) and blood pressure check should be done every year, and kidney function tests (creatinine, glomerular filtration rate [GFR]) should be checked every few years (or more often if abnormal results are found). Regular urine tests for protein should be performed as well. The presence of protein in the urine may mean that the kidney has some damage.

A 66-year-old client has been complaining of sleeping more, increased urination, anorexia, weakness, irritability, depression, and bone pain that interferes with her going outdoors. Based on these assessment findings, the nurse would suspect which of the following disorders? A. Diabetes mellitus B. Diabetes insipidus C. Hypoparathyroidism D. Hyperparathyroidism

Hyperparathyroidism Hyperparathyroidism is most common in older women and is characterized by bone pain and weakness from excess parathyroid hormone (PTH). Clients also exhibit hypercalciuria-causing polyuria. Option A: Common symptoms of diabetes mellitus include polyuria, polydipsia, and polyphagia Option B: While clients with diabetes insipidus also have polyuria, they don't have bone pain and increased sleeping. Option C: Hypoparathyroidism is characterized by urinary frequency rather than polyuria.

Which of the following types of cardiomyopathy does not affect cardiac output?

Hypertrophic Cardiac output isn't affected by hypertrophic cardiomyopathy because the size of the ventricle remains relatively unchanged. Three explanations for the systolic anterior motion of the mitral valve have been offered, as follows: (1) the mitral valve is pulled against the septum by contraction of the papillary muscles, which occurs because of the valve's abnormal location and septal hypertrophy altering the orientation of the papillary muscles; (2) the mitral valve is pushed against the septum because of its abnormal position in the outflow tract; (3) the mitral valve is drawn toward the septum because of the lower pressure that occurs as blood is ejected at high velocity through a narrowed outflow tract (Venturi effect). Option A: Dilated cardiomyopathy causes a decrease in cardiac output. Progressive dilation can lead to significant mitral and tricuspid regurgitation, which may further diminish the cardiac output and increase end-systolic volumes and ventricular wall stress. In turn, this leads to further dilation and myocardial dysfunction. Option C: Restrictive cardiomyopathy causes decreased cardiac output. Reduced LV filling leads to reduced stroke volume resulting in low cardiac output symptoms such as fatigue and lethargy. Increased filling pressures cause pulmonary and systemic congestion and symptomatic dyspnea. Option D: Obliterative cardiomyopathy may affect cardiac output because a thrombus hampers ventricular filling, which may decrease the cardiac output.

A patient is admitted to the hospital with a calcium level of 6.0 mg/dL. Which of the following symptoms would you not expect to see in this patient? A. Numbness in hands and feet B. Muscle cramping C. Hypoactive bowel sounds D. Positive Chvostek's sign

Hypoactive bowel sounds Normal serum calcium is 8.5 - 10 mg/dL. The patient is hypocalcemic. Increased gastric motility, resulting in hyperactive (not hypoactive) bowel sounds, abdominal cramping and diarrhea is an indication of hypocalcemia. Hypocalcemia is said to be present when the total serum calcium concentration is less than 8.8 mg/dl. The disorder may be acquired or inherited but its presentation can vary- from asymptomatic to life-threatening. Hypocalcemia is commonly seen in hospitalized patients and for the most part, is mild in nature and only requires supportive treatment. Option A: Paresthesias can be perioral or otherwise. Carpopedal spasm, also referred to as Trousseau's sign. It represents increased neuromuscular excitability which may be related to the gating function of calcium ions for ion channels at a cellular level (particularly in neurons). This manifests as a spasm of hand upon routine BP check. Option B: Numbness in hands and feet and muscle cramps are also signs of hypocalcemia. Tetany is generally induced by a rapid decline in serum ionized calcium. Tetany is usually most dangerous and most commonly seen in the presence of respiratory alkalosis causing hypocalcemia. Option D: Positive Chvostek's sign refers to the sustained twitching of facial muscles following tapping in the area of the cheekbone and is a hallmark of hypocalcemia. Another manifestation of heightened neuromuscular excitability. It is the spasm of facial muscles in response to tapping the facial nerve near the angle of the jaw.

A patient with a history of diabetes mellitus is in the second postoperative day following cholecystectomy. She has complained of nausea and isn't able to eat solid foods. The nurse enters the room to find the patient confused and shaky. Which of the following is the most likely explanation for the patient's symptoms? A. Anesthesia reaction B. Hyperglycemia C. Hypoglycemia D. Diabetic ketoacidosis

Hypoglycemia A postoperative diabetic patient who is unable to eat is likely to be suffering from hypoglycemia. The actual treatment recommendations for a given patient should be individualized, based on diabetes classification, usual diabetes regimen, state of glycemic control, nature and extent of surgical procedure, and available expertise. Option A: An anesthesia reaction would not occur on the second postoperative day. Anesthesia and surgery cause a stereotypical metabolic stress response that could overwhelm homeostatic mechanisms in patients with pre-existing abnormalities of glucose metabolism. The invariant features of the metabolic stress response include release of the catabolic hormones epinephrine, norepinephrine, cortisol, glucagons, and growth hormone and inhibition of insulin secretion and action. Option B: Confusion is a late sign of hyperglycemia. Shakiness is not one of its symptoms. The management approach in these categories of patients always includes insulin therapy in combination with dextrose and potassium infusion. Major surgery is defined as one requiring general anesthesia of ?1 h. At a minimum, blood glucose should be monitored before and immediately after surgery in all patients. Those undergoing extensive procedures should have hourly glucose monitoring during and immediately following surgery. Option D: Symptoms of DKA include excessive thirst, frequent urination, abdominal pain, fruity-scented breath, confusion, and shortness of breath. However, shakiness is not a sign of DKA. The stress of surgery itself results in metabolic perturbations that alter glucose homeostasis, and persistent hyperglycemia is a risk factor for endothelial dysfunction, postoperative sepsis, impaired wound healing, and cerebral ischemia. The stress response itself may precipitate diabetic crises (diabetic ketoacidosis [DKA].

During the administration of chemotherapy agents, Nurse Oliver observed that the IV site is red and swollen when the IV is touched Stacy shouts in pain. The first nursing action to take is: A. Notify the physician. B. Flush the IV line with saline solution. C. Immediately discontinue the infusion. D. Apply an ice pack to the site, followed by warm compress.

Immediately discontinue the infusion. Edema or swelling at the IV site is a sign that the needle has been dislodged and the IV solution is leaking into the tissues causing the edema. The patient feels pain as the nerves are irritated by pressure and the IV solution. The first action of the nurse would be to discontinue the infusion right away to prevent further edema and other complications. Option A: After discontinuing the infusion, the nurse should notify the physician. Option B: Flushing may aggravate the edema since the IV cannula might be dislodged. Option D: Compresses may be given as indicated by the physician.

A nurse is counseling patients at a health clinic on the importance of immunizations. Which of the following information is the most accurate regarding immunizations? A. All infectious diseases can be prevented with proper immunization. B. Immunizations provide natural immunity from disease. C. Immunizations are risk-free and should be universally administered. D. Immunization provides acquired immunity from some specific diseases.

Immunization provides acquired immunity from some specific diseases. Immunization is available for the prevention of some, but not all, specific diseases. This type of immunity is "acquired" by causing antibodies to form in response to a specific pathogen. Live vaccines are more effective than killed vaccines because they retain more antigens of the microbes. However, toxoids, including those that cause tetanus and diphtheria, are the most effective bacterial vaccines of all because they are based on inactivated exotoxins that stimulate strong antibody production. Subunit vaccines, including hepatitis B, meningococcal, and Haemophilus influenzae B vaccines are effective when conjugated to carrier proteins such as tetanus toxoid. Option A: Immunizations can prevent some, but not all, infectious diseases. The current immunizations protect against diphtheria, tetanus, pertussis, poliomyelitis, measles, mumps, rubella, pneumococcal pneumonia, smallpox, sepsis, meningitis, hepatitis B, varicella-zoster, tuberculosis, cholera, diarrhea caused by rotavirus, salmonellosis, and dengue. Option B: Natural immunity is present at birth because the infant acquires maternal antibodies. Innate (natural) immunity is so named because it is present at birth and does not have to be learned through exposure to an invader. It thus provides an immediate response to foreign invaders. However, its components treat all foreign invaders in much the same way. Option C: Immunization, like all medication, cannot be risk-free and should be considered based on the risk of the disease in question. Most vaccines have adverse reactions as any drug or medication. For example, BCG vaccination may provoke fever, vomiting, hematuria, lymphadenitis, and redness at the site of injection. HiB vaccine has few adverse reactions, and none of them are dangerous. These reactions include redness, warmth, swelling, and fever over 101 degrees F. A rare and lethal adverse reaction secondary to vaccination is the Guillain-Barre syndrome.

The biopsy of Mr. Gonzales confirms the diagnosis of cirrhosis. Mr. Gonzales is at increased risk for excessive bleeding primarily because of: A. Impaired clotting mechanism B. Varix formation C. Inadequate nutrition D. Trauma of invasive procedure

Impaired clotting mechanism Cirrhosis of the liver results in decreased Vitamin K absorption and formation of clotting factors resulting in impaired clotting mechanism. Option B: Esophageal varices sometimes form when blood flow to the liver is blocked, most often by scar tissue in the liver caused by liver disease. Option C: Inadequate nutrition alone cannot cause excessive bleeding in cirrhosis. Option D: An invasive procedure may cause trauma that may result in bleeding, but the client has not yet undergone any invasive procedure.

Septal involvement occurs in which type of cardiomyopathy?

In hypertrophic cardiomyopathy, hypertrophy of the ventricular septum - not the ventricle chambers - is apparent. Hypertrophic cardiomyopathy (HCM) is a genetic cardiovascular disease. It is defined by an increase in left ventricular wall thickness that is not solely explained by abnormal loading conditions. This disorder is caused by a mutation in cardiac sarcomere protein genes and is most frequently transmitted as an autosomal dominant trait. Option A: In congestive cardiomyopathy, the heart becomes stretched and weakened and is unable to pump effectively. Congestive cardiomyopathy is a clinical state in which an abnormality of ventricular myocardium results in impaired pump function and circulatory congestion. Option B: Dilated cardiomyopathy is also called congestive cardiomyopathy, which is a condition wherein the heart cannot pump effectively due to its weakening state. In dilated cardiomyopathy, the heart's ability to pump blood is decreased because the heart's main pumping chamber, the left ventricle, is enlarged, dilated and weak. At first, the chambers of the heart respond by stretching to hold more blood to pump through the body. This helps to strengthen the heart's contraction and keep the blood moving for a short while. With time, the heart muscle walls weaken and are not able to pump as strongly. Option D: Restrictive cardiomyopathy, the rarest form of cardiomyopathy, is a condition in which the walls of the lower chambers of the heart (the ventricles) are abnormally rigid and lack the flexibility to expand as the ventricles fill with blood.

A nurse is caring for a patient with peripheral vascular disease (PVD). The patient complains of burning and tingling of the hands and feet and cannot tolerate touch of any kind. Which of the following is the most likely explanation for these symptoms? A. Inadequate tissue perfusion leading to nerve damage. B. Fluid overload leading to compression of nerve tissue. C. Sensation distortion due to psychiatric disturbance. D. Inflammation of the skin on the hands and feet.

Inadequate tissue perfusion leading to nerve damage. Patients with peripheral vascular disease often sustain nerve damage as a result of inadequate tissue perfusion. Intermittent claudication results when blood flow distal to the occlusion is sufficiently compromised, resulting in fixed oxygen delivery that is unable to match oxygen demand. The most severe form of PAD is critical limb ischemia, which is defined as limb pain at rest or impending limb loss. Option B: Fluid overload is not characteristic of PVD. Atherosclerotic plaque builds up slowly over decades within the wall of the vessel. Plaque accumulation results in vascular stenosis and frequent vascular dilation to maximize end-organ perfusion. Once the vessel dilation capacity is maximized, the plaque continues to accumulate, which further compromises the lumen occasionally, leading to critical narrowing of the artery. Option C: There is nothing to indicate psychiatric disturbance in the patient. The overall prognosis of patients with peripheral vascular disease must take into account patient risk factors, cardiovascular health, and disease severity. In terms of limb health at 5 years, nearly 80% of patients will have stable claudication symptoms. Only 1% to 2% of patients will progress to critical limb ischemia in 5 years. Twenty to 30% of patients with PAD will die within 5 years, with 75% of those deaths attributed to cardiovascular causes. Option D: Skin changes in PVD are secondary to decreased tissue perfusion rather than primary inflammation. Examination of the limbs should involve assessment for pulselessness, pallor, muscular atrophy, cool or cyanotic skin, or pain with palpation. Lower extremity ulcers may be arterial, venous, neuropathic, or a combination of two or more. Ulcers secondary to arterial insufficiency are tender and typically have ragged borders with a dry base and pale or necrotic centers.

What would be the primary goal of therapy for a client with pulmonary edema and heart failure? A. Enhance comfort B. Increase cardiac output C. Improve respiratory status D. Peripheral edema decreased

Increase cardiac output The primary goal of therapy for the client with pulmonary edema or heart failure is increasing cardiac output. Pulmonary edema is an acute medical emergency requiring immediate intervention. Preload reduction decreases pulmonary capillary hydrostatic pressure and reduces fluid transudation into the pulmonary interstitium and alveoli. Afterload reduction increases cardiac output and improves renal perfusion, which allows for diuresis in the patient with fluid overload. Option A: Enhancing the client's comfort is a goal of therapy but not an urgent one. The initial management of patients with cardiogenic pulmonary edema (CPE) should address the ABCs of resuscitation, that is, airway, breathing, and circulation. Oxygen should be administered to all patients to keep oxygen saturation at greater than 90%. Any associated arrhythmia or MI should be treated appropriately. Option C: Respiratory status would improve following an increase in the cardiac output. Patients who remain hypoxic despite supplemental oxygenation and patients who have severe respiratory distress require ventilatory support in addition to maximal medical therapy. Option D: Peripheral edema is a manifestation of cardiogenic pulmonary edema and right ventricular heart failure, but it is not a primary concern. In general, begin with oral vasodilator therapy, most commonly ACE inhibitors. If the patient was initially treated with inotropic medications, wean the patient off of these as soon as his or her condition is stable, to minimize adverse effects.

While a healthcare provider is caring for a patient following a laryngectomy, the patient suddenly becomes pale and nonresponsive with a BP of 90/40. What should be done first? Administer atropine intravenously. Place the client in the Trendelenburg position. Increase the infusion of dextrose in normal saline (D5NS). Move the emergency cart to the patient's bedside.

Increase the infusion of dextrose in normal saline (D5NS). D5NS infusion is hypertonic, so it will draw fluid into the circulation. Trendelenburg position could compromise the airway in a patient who has had head or neck surgery. Atropine could cause hyponatremia and further hypotension. It is not necessary to move the emergency cart to the patient's bedside at this time.

An older patient with osteoarthritis is preparing for discharge. Which of the following information is correct. A. Increased physical activity and daily exercise will help decrease discomfort associated with the condition. B. Joint pain will diminish after a full night of rest. C. Nonsteroidal anti-inflammatory medications should be taken on an empty stomach. D. Acetaminophen (Tylenol) is a more effective anti-inflammatory than ibuprofen (Motrin).

Increased physical activity and daily exercise will help decrease discomfort associated with the condition. Physical activity and daily exercise can help to improve movement and decrease pain in osteoarthritis. Exercise programs that combine both aerobic and resistance training have been shown to decrease pain and improve physical function in multiple trials and should be encouraged by physicians regularly. Option B: Joint pain and stiffness are often at their worst during the early morning after several hours of decreased movement. Evidence shows that water-based activities can help relieve symptoms and improve joint function, hence consultation with a physical therapist is recommended. Further, many of these patients may benefit from a walking aid. Option C: Ibuprofen is a strong anti-inflammatory, but should always be taken with food to avoid GI distress. NSAIDs are usually prescribed orally or topically and initially, should be started as needed rather than scheduled. Due to gastrointestinal toxicity, and renal and cardiovascular side effects, oral NSAIDs should be used very cautiously and with close monitoring long term. Option D: Acetaminophen is a pain reliever, but does not have anti-inflammatory activity. Pharmacotherapy of OA involves oral, topical, and/or intra-articular options. Acetaminophen and oral NSAIDs are the most popular and affordable options for OA and are usually the initial choice of pharmacologic treatment.

A client hospitalized with MRSA (methicillin-resistant staph aureus) is placed on contact precautions. Which statement is true regarding precautions for infections spread by contact? A. The client should be placed in a room with negative pressure. B. Infection requires close contact; therefore, the door may remain open. C. Transmission is highly likely, so the client should wear a mask at all times. D. Infection requires skin-to-skin contact and is prevented by hand washing, gloves, and a gown.

Infection requires skin-to-skin contact and is prevented by hand washing, gloves, and a gown. The client with MRSA should be placed in isolation. Gloves, a gown, and a mask should be used when caring for the client and hand washing is very important. Option A: The door should remain closed, but a negative-pressure room is not necessary. Whenever possible, patients with MRSA will have a single room or will share a room only with someone else who also has MRSA. Option B: MRSA is spread by contact with blood or body fluid or by touching the skin of the client. Patients are asked to stay in their hospital rooms as much as possible. They should not go to common areas, such as the gift shop or cafeteria. They may go to other areas of the hospital for treatments and tests. Option C: It is cultured from the nasal passages of the client, so the client should be instructed to cover his nose and mouth when he sneezes or coughs. It is not necessary for the client to wear the mask at all times; the nurse should wear the mask.

Prolonged occlusion of the right coronary artery produces an infarction in which of the following areas of the heart? Anterior Apical Inferior Lateral

Inferior The right coronary artery supplies the right ventricle or the inferior portion of the heart. Therefore, prolonged occlusion could produce and infarction in that area. A) The right coronary artery doesn't supply the anterior portion (left ventricle) of the heart. The right coronary artery supplies blood to the right ventricle, the right atrium, and the sinoatrial (SA) and atrioventricular (AV) nodes, which regulate the heart rhythm,. The right coronary artery divides into smaller branches, including the right posterior descending artery and the acute marginal artery. B) The right coronary artery does not supply to the apical portion or the left ventricle of the heart. Together with the left anterior descending artery, the right coronary artery helps supply blood to the middle or septum of the heart. D) The right coronary artery does not supply to the lateral portion (some of the left ventricle and the left atrium) of the heart. The left main coronary artery supplies blood to the left side of the heart muscle (the left ventricle and left atrium).

Which of the following classes of medications maximizes cardiac performance in clients with heart failure by increasing ventricular contractility? Beta-adrenergic blockers Calcium Channel Blockers Diuretics Inotropic Agents

Inotropic Agents Inotropic agents are administered to increase the force of the heart's contractions, thereby increasing ventricular contractility and ultimately increasing cardiac output. A) Beta blockers work by blocking the effects of the hormone epinephrine, also known as adrenaline. They cause the heart to beat more slowly and with less force, which lowers blood pressure. B) Calcium channel blockers decrease the heart rate and ultimately decrease the workload of the heart. C) Diuretics are administered to decrease the overall vascular volume, also decreasing the workload of the heart.

A patient arrives in the emergency department and reports splashing concentrated household cleaner in his eye. Which of the following nursing actions is a priority? A. Irrigate the eye repeatedly with normal saline solution. B. Place fluorescein drops in the eye. C. Patch the eye. D. Test visual acuity.

Irrigate the eye repeatedly with normal saline solution. Emergency treatment following a chemical splash to the eye includes immediate irrigation with normal saline. The irrigation should be continued for at least 10 minutes. Immediate irrigation with copious amounts of an isotonic solution is the mainstay of treatment for chemical burns. Never use any substance to neutralize chemical exposure as the exothermic reaction can lead to secondary thermal injuries. Irrigation should continue until the pH of the eye is between 7.0 to 7.4 and remains within this range for at least 30 minutes after the irrigation has been discontinued. Option B: Fluorescein drops are used to check for scratches on the cornea due to their fluorescent properties and are not part of the initial care of a chemical splash. A topical anesthetic such as tetracaine can be applied directly to the eye, or 10 mL of 1% lidocaine can be added to a liter of irrigating fluid, taking care not to reach a toxic dose if copious irrigation is required. Option C: Patching the eye would not remove the chemical. Severe burns may require upwards of ten liters of irrigation. Irrigation should be gentle, and care should be taken to avoid direct irrigation to the cornea to prevent further injury. Use of a commercial irrigation lens such as a Morgan lens may be helpful. Option D: Following irrigation, visual acuity will be assessed. Ocular burns, particularly any chemical burns with corneal clouding or abrasions, should have prompt ophthalmology evaluation. Topical antibiotic ointment and possibly topical steroids may be prescribed for both chemical and thermal burns, but topical steroids should only be prescribed in consultation with an ophthalmologist.

Nurse Rose is aware that the statement that best explains why furosemide (Lasix) is administered to treat hypertension is: A. It dilates peripheral blood vessels. B. It decreases sympathetic cardio acceleration. C. It inhibits the angiotensin-converting enzymes. D. It inhibits the reabsorption of sodium and water in the loop of Henle.

It inhibits the reabsorption of sodium and water in the loop of Henle. Furosemide is a loop diuretic that inhibits sodium and water reabsorption in the loop Henle, thereby causing a decrease in blood pressure. Option A: Vasodilators cause dilation of peripheral blood vessels, directly relaxing vascular smooth muscle and decreasing blood pressure. Option B: Adrenergic blockers decrease sympathetic cardio acceleration and decrease blood pressure. Option C: Angiotensin-converting enzyme inhibitors decrease blood pressure due to their action on angiotensin.

A nurse is counseling the mother of a newborn infant with hyperbilirubinemia. Which of the following instructions by the nurse is not correct? A. Continue to breastfeed frequently, at least every 2-4 hours. B. Follow up with the infant's physician within 72 hours of discharge for a recheck of the serum bilirubin and exam. C. Watch for signs of dehydration, including decreased urinary output and changes in skin turgor. D. Keep the baby quiet and swaddled, and place the bassinet in a dimly lit area.

Keep the baby quiet and swaddled, and place the bassinet in a dimly lit area. An infant discharged home with hyperbilirubinemia (newborn jaundice) should be placed in a sunny rather than dimly lit area with skin exposed to help process the bilirubin. Phototherapy is started based on risk factors and the serum bilirubin level on the nomogram. Bilirubin absorbs light optimally in the blue-green range (460 to 490 nm) and is either photo isomerized and excreted in the bile or converted into lumirubin and excreted in the urine. During phototherapy, the eyes of the newborn must be covered, and the maximum body surface area exposed to the light. Option A: Frequent feedings will help to metabolize the bilirubin. Breast milk jaundice occurs late in the first week, peaks in the second, and usually resolves by 12 weeks of age. It is due to inhibition of UGT activity and a factor in breast milk with a beta-glucuronidase-like activity that deconjugates conjugated bilirubin in the intestines leading to increased enterohepatic circulation. Option B: A recheck of the serum bilirubin and a physical exam within 72 hours will confirm that the level is falling and the infant is thriving and is well hydrated. Bilirubin levels may be assessed using a transcutaneous measurement device or taking blood for total serum or plasma level determination. Transcutaneous measurement decreases the frequency of blood tests for bilirubin but is limited by dark skin tone and if the neonate has received phototherapy. Option C: Signs of dehydration, including decreased urine output and skin changes, indicate inadequate fluid intake and will worsen hyperbilirubinemia. Breastfeeding jaundice, also known as breastfeeding failure jaundice, occurs in the first week of life and is due to failure of adequate intake of breast milk leading to dehydration and sometimes hypernatremia. Breastfeeding failure leads to decreased intestinal motility and decreases the elimination of bilirubin in the stool or meconium.

A client with hypertension asks the nurse which factors can cause blood pressure to drop to normal levels? A. Kidneys' excretion to sodium only. B. Kidneys' retention of sodium and water. C. Kidneys' excretion of sodium and water. D. Kidneys' retention of sodium and excretion of water.

Kidneys' excretion of sodium and water. The kidneys respond to rise in blood pressure by excreting sodium and excess water. This response ultimately affects systolic blood pressure by regulating blood volume. Option A: As the kidneys excrete sodium, water also goes along with it. Option B: Sodium or water retention would only further increase blood pressure. Option D: Sodium and water travel together across the membrane in the kidneys; one can't travel without the other.

Which of the following landmarks is the correct one for obtaining an apical pulse? Left intercostal space, midaxillary line. Left fifth intercostal space, midclavicular line. Left second intercostal space, midclavicular line. Left seventh intercostal space, midclavicular line.

Left fifth intercostal space, midclavicular line The correct landmark for obtaining an apical pulse. This is the point of maximum impulse and the location of the left ventricular apex. A) Normally, heart sounds aren't heard in the midaxillary line. During systolic contraction of the heart, a high amplitude wave of blood gets ejected through the aortic valve out towards the periphery. This high-pressure wave distends the arteries, especially compliant "elastic" or "conducting" arteries, which tend to be larger and closer to the heart. The subsequent release of that distention somewhat sustains the systolic wave of blood throughout the body, creating a spike followed by a downward sloping plateau in pulse waveform. C)The left second intercostal space in the midclavicular line is where the pulmonic sounds are auscultated. The intensity of the pulse is determined by blood pressure as well as other physiological factors such as ambient temperature. For example, colder temperatures cause vasoconstriction leading to decreased intensity. Besides the normal variation in a rhythm that occurs with the respiratory cycle, the heart rate should be regular in the absence of pathology. D) Heart sounds are not heard in the seventh intercostal space in the midclavicular line. Pulses are accurately measured when the clinician places their fingertips onto the skin overlying the vessel (locations, see below) and focuses on different aspects of the pulse. (NB: although one often hears that utilization of the thumb for measuring pulses is less accurate secondary to increased perception of the clinician's own pulsation during palpation, the author could not find data to support or refute this claim). If possible, the limb under evaluation should have support throughout palpation.

After an anterior wall MI, which of the following problems is indicated by auscultation of crackles in the lungs? Left-sided HF Pulmonic Valve Malfunction Right-sided HF Tricuspid Valve Malfunction

Left-sided HF The left ventricle is responsible for the most cardiac output. An anterior wall MI may result in a decrease in left ventricular function. When the left ventricle doesn't function properly, resulting in left-sided heart failure, fluid accumulates in the interstitial and alveolar spaces in the lungs and causes crackles. B) Pulmonic valve malfunction may cause right-sided heart failure. Pulmonary regurgitation (PR, also called pulmonic regurgitation) is a leaky pulmonary valve. This valve helps control the flow of blood passing from the right ventricle to the lungs. A leaky pulmonary valve allows blood to flow back into the heart chamber before it gets to the lungs for oxygen. C) Right-sided heart failure can be caused by a pulmonic valve or tricuspid valve malfunction. This is the narrowing of the pulmonic valve that limits blood flow out of the right ventricle. D) Tricuspid valve malfunction causes right-sided heart failure. The tricuspid valve doesn't close properly. This causes blood in the right ventricle to flow back into the right atrium.

The primary reason for rapid continuous rewarming of the area affected by frostbite is to: A. Lessen the amount of cellular damage B. Prevent the formation of blisters C. Promote movement D. Prevent pain and discomfort

Lessen the amount of cellular damage Rapid continuous rewarming of frostbite primarily lessens cellular damage. Rapid rewarming is the single most effective therapy for frostbite. Heat conduction and radiation from deeper tissue circulation prevent freezing and ice crystallization until the skin temperature drops below 0°C. Option B: Blisters contain high amounts of thromboxane and prostaglandins. They contract and dry within 2-3 weeks, forming a dark eschar that sloughs off in 4 weeks. Option C: Before movement can be improved, the cell injury should be resolved first. Option D: It does not prevent the formation of blisters. It does promote movement, but this is not the primary reason for rapid rewarming. It might increase pain for a short period of time as the feeling comes back into the extremity.

A patient with a history of atrial fibrillation is scheduled for a left hip hemiarthroplasty at 09:00. A health professional is administering medications at 08:00. Which of these medication orders should the health professional question? Cephalosporin antibiotic intravenously Low molecular weight heparin anticoagulant subcutaneously Beta blocker orally Opioid analgesic intravenously

Low molecular weight heparin anticoagulant subcutaneously Heparin and low-molecular-weight heparins are normally held 12 hours before surgery because of the increased risk of perioperative bleeding. Patients with a history of atrial fibrillation are often on anticoagulation therapy to prevent the formation of blood clots. The health professional should contact the appropriate physician to clarify whether the heparin should be administered.

An 18-year-old male client admitted with heat stroke begins to show signs of disseminated intravascular coagulation (DIC). Which of the following laboratory findings is most consistent with DIC? A. Low platelet count B. Elevated fibrinogen levels C. Low levels of fibrin degradation products D. Reduced prothrombin time

Low platelet count In DIC, platelets and clotting factors are consumed, resulting in microthrombi and excessive bleeding. As clots form, fibrinogen levels decrease and the prothrombin time increases. Option B: Severe, rapidly evolving DIC is diagnosed by demonstrating thrombocytopenia, an elevated partial thromboplastin time and prothrombin time, increased levels of plasma D-dimers, and a decreasing plasma fibrinogen level. Option C: Fibrin degradation products increase as fibrinolysis takes place. Option D: Both PT and aPTT seem prolonged in about 50% of DIC cases which is attributed to the consumption of coagulation factors but can also be prolonged in impaired synthesis of coagulation factors and in massive bleeding.

A patient is brought to the emergency department after a bee sting. The family reports a history of severe allergic reaction, and the patient appears to have some oral swelling. Which of the following is the most urgent nursing action? A. Consult a physician. B. Maintain a patent airway. C. Administer epinephrine subcutaneously. D. Administer diphenhydramine (Benadryl) orally.

Maintain a patent airway. The patient may be experiencing an anaphylactic reaction. Airway management is paramount. Thoroughly examine the patient for airway patency or any indications of an impending loss of airway. Perioral edema, stridor, and angioedema are very high risk, and obtaining a definitive airway is imperative. Delay may reduce the chances of successful intubation as continued swelling occurs, increasing the risk for a surgical airway. Option A: The physician will see the patient as soon as possible with the above actions underway. Often when anaphylaxis is diagnosed co-treatment is initiated with steroids, antihistamines, inhaled bronchodilators, and vasopressors. Glucagon can also be used if indicated. These agents can assist in refractory initial anaphylaxis or aid in the prevention of recurrence and biphasic reactions. Option C: The most urgent action is to maintain an airway, particularly with visible oral swelling, followed by the administration of epinephrine by subcutaneous injection. Epinephrine is given through intramuscular injection and at a dose of 0.3 to 0.5 mL of 1:1,000 concentration of epinephrine. Pediatric dosing is 0.01 mg/kg or 0.15 mg intramuscularly (IM) (epinephrine injection for pediatric dosage). Intramuscular delivery has proven to provide more rapid delivery and produce better outcomes than subcutaneous or intravascular. Option D: Oral diphenhydramine is indicated for mild allergic reactions and is not appropriate for anaphylaxis. Antihistamines are often routinely used; most commonly is Hblocker administration of diphenhydramine 25 to 50 mg IV/IM. While the clinical benefit is unproven in anaphylaxis, its utility is evident in more minor allergic processes.

For a client with Graves' disease, which nursing intervention promotes comfort? A. Restricting intake of oral fluids. B. Placing extra blankets on the client's bed. C. Limiting intake of high-carbohydrate foods. D. Maintaining room temperature in the low-normal range.

Maintaining room temperature in the low-normal range. Graves' disease causes signs and symptoms of hypermetabolism, such as heat intolerance, diaphoresis, excessive thirst and appetite, and weight loss. To reduce heat intolerance and diaphoresis, the nurse should keep the client's room temperature in the low-normal range. Option A: To replace fluids lost via diaphoresis, the nurse should encourage, not restrict, intake of oral fluids. Option B: Placing extra blankets on the bed of a client with heat intolerance would cause discomfort. Option C: To provide needed energy and calories, the nurse should encourage the client to eat high-carbohydrate foods.

The nurse caring for a client in the neonatal intensive care unit administers adult-strength Digitalis to the 3-pound infant. As a result of her actions, the baby suffers permanent heart and brain damage. The nurse can be charged with: A. Negligence B. Tort C. Assault D. Malpractice

Malpractice The nurse could be charged with malpractice, which is failing to perform, or performing an act that causes harm to the client. Giving the infant an overdose falls into this category. In the United States, a patient may allege medical malpractice against a clinician, which is typically defined by the failure the provide the degree of care another clinician in the same position with the same credentials would have performed that resulted in injury to the patient. Option A: Negligence is failing to perform care for the client. a tort is a wrongful act committed. Negligence, in law, the failure to meet a standard of behaviour established to protect society against unreasonable risk. Negligence is the cornerstone of tort liability and a key factor in most personal injury and property-damage trials. Option B: A tort is a wrongful act committed on the client or their belongings. A tort is a civil wrong that causes harm to another person by violating a protected right. A civil wrong is an act or omission that is intentional, accidental, or negligent, other than a breach of contract. The specific rights protected give rise to the unique "elements" of each tort. Tort requires the presence of four elements that are the essential facts required to prove a civil wrong. Option C: Assault is a violent physical or verbal attack. Assault is the intentional act of making someone fear that you will cause them harm. You do not have to actually harm them to commit assault. Threatening them verbally or pretending to hit them are both examples of assault that can occur in a nursing home.

In reducing the risk of endocarditis, good dental care is an important measure. To promote good dental care in clients with endocarditis in a teaching plan should include proper use of? A. Dental floss B. Electric toothbrush C. Manual toothbrush D. Irrigation device

Manual toothbrush The use of an electric toothbrush, irrigation device or dental floss may cause bleeding of gums, allowing bacteria to enter and increasing the risk of endocarditis. Maintaining good oral hygiene and infection control can decrease the incidence of endocarditis in the moderate-risk group of patients and can eliminate the necessity of antibiotic prophylaxis of endocarditis. Option A: Dental floss may injure the gums and cause bleeding that may lead to infection. The gums become inflamed (red and swollen) and often bleed during tooth brushing, flossing, or certain dental procedures involving manipulation of the gums. When gums bleed, the bacteria can enter the bloodstream and can infect other parts of the body. Option B: Electronic toothbrushes cause too much friction to the gums and cause an infection that can increase the risk of endocarditis. In very rare cases, bacteria in the mouth may trigger endocarditis in people at higher risk. Here's what happens: Bacteria found in tooth plaque may multiply and cause gingivitis (gum disease). Option D: An irrigation device uses a stream of pressurized, pulsating water to clean between teeth and below the gum line. The water's pressure may cause injury to sensitive gums and lead to bleeding or an infection. To prevent endocarditis, patients with certain heart conditions receive a single dose of an antibiotic. The patient receives it about one hour prior to certain dental treatments.

Which hereditary disease is most closely linked to an aneurysm?

Marfan's syndrome results in the degeneration of the elastic fibers of the aortic media. Therefore, clients with the syndrome are more likely to develop an aortic aneurysm. Marfan syndrome (MFS) is a spectrum of disorders caused by a heritable genetic defect of connective tissue that has an autosomal dominant mode of transmission. The defect itself has been isolated to the FBN1 gene on chromosome 15, which codes for the connective tissue protein fibrillin. Abnormalities in this protein cause a myriad of distinct clinical problems, of which the musculoskeletal, cardiac, and ocular system problems predominate.

A pregnant client with a history of alcohol addiction is scheduled for a nonstress test. The nonstress test: A. Determines the lung maturity of the fetus B. Measures the activity of the fetus C. Shows the effect of contractions on the fetal heart rate D. Measures the neurological well-being of the fetus

Measures the activity of the fetus A nonstress test is done to evaluate periodic movement of the fetus. Prenatal non-stress test, popularly known as NST, is a method used to test fetal wellbeing before the onset of labor. A prenatal non-stress test functions in overall antepartum surveillance with ultrasound as a part or component of the biophysical profile. The presence of fetal movements and fetal heart rate acceleration is the most critical feature of the non-stress test. Option A: It is not done to evaluate lung maturity as in answer A. It is usually estimated by examination of the amniotic fluid, obtained by transabdominal amniocentesis, for lecithin, lecithin/sphingomyelin (L/S) ratio. The Lecithin-to-Sphingomyelin Ratio (L/S ratio) is one of several methods for clinicians to assess fetal lung maturation. This biochemical test was first introduced in the 1970s, where a sample of amniotic fluid was collected via amniocentesis to determine the risk of the neonate developing respiratory distress syndrome (RDS). Option C: An oxytocin challenge test shows the effect of contractions on fetal heart rate. The oxytocin challenge test (OCT) is a form of antenatal fetal monitoring that is being utilized by obstetricians to assess fetal well-being. Uterine contractions may lower the fetal oxygen supply by decreasing the blood circulation in the intervillous spaces of the placenta. Option D: Nonstress test does not measure neurological well-being of the fetus. Advances in technology over the past decade have enabled researchers to investigate directly the functional development of the fetal brain through two emerging techniques: functional magnetic resonance imaging (fMRI) and fetal magnetoencephalography (MEG).

Which of the following blood vessel layers may be damaged in a client with an aneurysm? Externa Interna Media Interna and Media

Media The factor common to all types of aneurysms is a damaged media. The media has more smooth muscle and less elastic fibers, so it's more capable of vasoconstriction and vasodilation. A) The tunica externa generally has no damage in an aneurysm. The outermost layer, tunica externa, comprises connective tissue providing protection for the vessel. B) The interna generally not damaged in an aneurysm. The tunica intima (New Latin "inner coat"), or intima for short, is the innermost tunica (layer) of an artery or vein. It is made up of one layer of endothelial cells and is supported by an internal elastic lamina. The endothelial cells are in direct contact with the blood flow. D) The media is the most damaged among the blood vessel la

A patient is admitted to the medical unit with a diagnosis of hepatitis. When preparing to administer intravenous medications, the healthcare provider understands that the patient's diagnosis primarily affects which phase of pharmacokinetics? Excretion Distribution Metabolism Absorption

Metabolism The liver is the primary site of drug metabolism, so alterations in liver function can affect the metabolism of medications. Liver disease can make drug metabolism unpredictable, with serious consequences. Liver function can also affect excretion to a lesser degree, if the liver's ability to make medications water soluble is compromised.

When assessing a client for an abdominal aortic aneurysm, which area of the abdomen is most commonly palpated? RUQ Directly over the umbilicus Middle lower abdomen to the left of the midline Midline lower abdomen to the right of the midline

Middle lower abdomen to the left of the midline The aorta lies directly left of the umbilicus; therefore, any other region is inappropriate for palpation. The aortic pulse can be palpated just above and to the left of the umbilicus. The width of the aorta can then be measured by placing both hands palms down on the patient's abdomen, with one index finger on either side of the aorta. Each systole should move the fingers apart. A) The right upper quadrant is a little farther from the appropriate area to be palpated. Abdominal examination includes palpation of the aorta and estimation of the size of the aneurysm. AAAs are palpated in the upper abdomen; the aorta bifurcates into the iliac arteries just above the umbilicus. B) Palpating the umbilicus would yield inappropriate results. The examiner may palpate the periumbilical area for any defect, mass, or an umbilical hernia. The patient can be asked to cough or bear down to feel for any protruding mass. D) The aorta lies to the left, not the right, of the umbilicus. Left lower quadrant tenderness may be a presenting sign of diverticulitis in the elderly. A mass, if present may be due to a tumor of the colon, a left ovarian cyst, or ectopic pregnancy. In the elderly, constipation leading to impacted feces may also present with a mass palpated in the left lower quadrant.

A patient with a spinal cord injury at level C3-4 is being cared for in the ED. What is the priority assessment? Determine the level at which the patient has intact sensation. Assess the level at which the patient has retained mobility. Check blood pressure and pulse for signs of spinal shock. Monitor respiratory effort and oxygen saturation level.

Monitor respiratory effort and oxygen saturation level. The first priority for the patient with an SCi is assessing respiratory patterns and ensuring an adequate airway. The patient with a high cervical injury is at risk for respiratory compromise because the spinal nerves (C3-5) innervate the phrenic nerve, which controls the diaphragm.

A client with acute pancreatitis is experiencing severe abdominal pain. Which of the following orders should be questioned by the nurse? A. Meperidine 100 mg IM q 4 hours PRN pain B. Mylanta 30 ccs q 4 hours via NG C. Cimetidine 300 mg PO q.i.d. D. Morphine 8 mg IM q 4 hours PRN pain

Morphine 8 mg IM q 4 hours PRN pain Morphine is contraindicated in clients with gallbladder disease and pancreatitis because morphine causes spasms of the Sphincter of Oddi. GI obstruction is another important contraindication. It is also considered by many as a contraindication to provide opioids to individuals that have a history of substance misuse, especially if a patient has had a history of abusing opioids. Option A: Clinicians use meperidine is used for the treatment of moderate to severe pain. It has intramuscular, subcutaneous, intravenous injection, syrup, and tablet forms. In the 20th century, it was the drug of choice amongst the opioids in the management of acute pain by most of the physicians and the management of some patients with chronic pain. Option B: Antacids are a group of drugs that have been on the market for many years. They were initially first-line defense against peptic ulcer disease; however, the discovery of proton pump inhibitors revolutionized the treatment of peptic ulcer disease. Currently, antacid use is restricted to the relief of mild intermittent gastroesophageal reflux disease (GERD) associated heartburn. Option C: Cimetidine is ordered for pancreatitis, making answer C incorrect. Cimetidine is a gastric acid reducer used in the short-term treatment of duodenal and gastric ulcers. The drug is effective in managing gastric hypersecretion, and therefore, used for the management of reflux esophagitis disease and in the prevention of stress ulcers. With the development of proton pump inhibitors, such as omeprazole, approved for the same indications, cimetidine is available as an over the counter formulation for the prevention of heartburn or acid indigestion, along with the other H2-receptor antagonists.

Multiple Sclerosis

Multiple sclerosis is a complex disease process. In addition to sensory and visual changes, weakness, coordination problems, or spasticity can present. Other complaints relating to overall health include bladder and bowel dysfunction, depression, cognitive impairment, fatigue, sexual dysfunction, sleep disturbances, and vertigo. The others are more stable.

The nurse knows that neurologic complications of multiple myeloma (MM) usually involve which of the following body systems? A. Brain B. Muscle spasm C. Renal dysfunction D. Myocardial irritability

Muscle spasm Back pain or paresthesia in the lower extremities may indicate impending spinal cord compression from a spinal tumor. This should be recognized and treated promptly as the progression of the tumor may result in paraplegia. Option A: The reasons underlying the relative paucity of CNS invasion by multiple myeloma in comparison with other tumors, whether solid or hematological, remain unknown, but this phenomenon might be the result of underlying biological characteristics, or lack thereof, of malignant plasma cells. Option C: In some cases, renal impairment is caused by the accumulation and precipitation of light chains, which form casts in the distal tubules, resulting in renal obstruction. Option D: Some of the treatments used in MM may also affect cardiovascular health, however, with careful risk assessment, monitoring, and prophylactic therapy, many of these cardiovascular complications can be managed or treated successfully.

Ruby is receiving thyroid replacement therapy, develops the flu, and forgets to take her thyroid replacement medicine. The nurse understands that skipping this medication will put the client at risk for developing which of the following life-threatening complications? A. Exophthalmos B. Thyroid storm C. Myxedema coma D. Tibial myxedema

Myxedema coma Myxedema coma, severe hypothyroidism, is a life-threatening condition that may develop if thyroid replacement medication isn't taken. Option A: Exophthalmos, protrusion of the eyeballs, is seen with hyperthyroidism. If a person's immune system attacks the thyroid gland, it may react by producing extra hormones. The autoimmune antibodies can attack the muscles and soft tissue surrounding the eyes, which can cause them to protrude from the sockets. Option B: Thyroid storm is life-threatening but is caused by severe hyperthyroidism. It is also referred to as thyrotoxic crisis, an acute, life-threatening hypermetabolic state induced by excessive release of thyroid hormones. Option D: Tibial myxedema, peripheral mucinous edema involving the lower leg, is associated with hypothyroidism but isn't life-threatening

Arnold, a 19-year-old client with a mild concussion is discharged from the emergency department. Before discharge, he complains of a headache. When offered acetaminophen, his mother tells the nurse the headache is severe and she would like her son to have something stronger. Which of the following responses by the nurse is appropriate? A. "Your son had a mild concussion, acetaminophen is strong enough." B. "Aspirin is avoided because of the danger of Reye's syndrome in children or young adults." C. "Narcotics are avoided after a head injury because they may hide a worsening condition." D. Stronger medications may lead to vomiting, which increases the intracranial pressure (ICP)."

Narcotics are avoided after a head injury because they may hide a worsening condition. Narcotics may mask changes in the level of consciousness that indicate increased ICP. Option A: Acetaminophen is strong enough ignores the mother's question and therefore isn't appropriate. Option B: Aspirin is contraindicated in conditions that may have bleeding, such as trauma, and for children or young adults with viral illnesses due to the danger of Reye's syndrome. Option D: Stronger medications may not necessarily lead to vomiting but will sedate the client, thereby masking changes in his level of consciousness.

The nurse is aware that the following symptom is most commonly an early indication of stage 1 Hodgkin's disease? A. Pericarditis B. Night sweat C. Splenomegaly D. Persistent hypothermia

Night sweat In stage 1, symptoms include a single enlarged lymph node (usually), unexplained fever, night sweats, malaise, and generalized pruritus. Option A: Pericarditis refers to the inflammation of the pericardium, two thin layers of a sac-like tissue that surround the heart, hold it in place, and help it work. Pericarditis isn't associated with Hodgkin's disease, nor is hypothermia. Moreover, splenomegaly and pericarditis aren't symptoms. Option C: The spleen is involved in 20%-30% of cases of Hodgkin's disease. Patients are usually asymptomatic. Option D: Persistent hypothermia is associated with Hodgkin's but isn't an early sign of the disease.

A patient is admitted to the oncology unit for diagnosis of suspected Hodgkin's disease. Which of the following symptoms is typical of Hodgkin's disease? A. Painful cervical lymph nodes. B. Night sweats and fatigue. C. Nausea and vomiting. D. Weight gain.

Night sweats and fatigue. Symptoms of Hodgkin's disease include night sweats, fatigue, weakness, and tachycardia. Hodgkin lymphoma (HL), formerly called Hodgkin's disease, is a rare monoclonal lymphoid neoplasm with high cure rates. Biological and clinical studies have divided this disease entity into two distinct categories: classical Hodgkin lymphoma and nodular lymphocyte-predominant Hodgkin lymphoma (NLP-HL). Option A: The disease is characterized by painless, enlarged cervical lymph nodes. Patients with Hodgkin lymphoma frequently present with painless supra-diaphragmatic lymphadenopathy (one to two lymph node areas), B symptoms including unexplained profound weight loss, high fevers, and drenching night sweats. Option C: Nausea and vomiting are not typically symptoms of Hodgkin's disease. Chronic pruritus is another disease symptom that may be encountered. If mediastinal nodes enlargement is significant, the mass effect can produce chest pain and shortness of breath. If the patient has an extra-nodal disease, which is less common, related clinical manifestations may occur. Option D: Weight loss occurs early in the disease. Four features characterize Hodgkin lymphomas. They commonly arise in the cervical lymph nodes; the disease is more common in young adults; there are scattered large mononuclear Hodgkin and multinucleated cells (Reed-Sternberg) intermixed in a background of a mixture of non-neoplastic inflammatory cells; finally, T lymphocytes are often observed surrounding the characteristic neoplastic cells.

The physician orders laboratory tests to confirm hyperthyroidism in a female client with classic signs and symptoms of this disorder. Which test result would confirm the diagnosis? A. No increase in the thyroid-stimulating hormone (TSH) level after 30 minutes during the TSH stimulation test. B. A decreased TSH level. C. An increase in the TSH level after 30 minutes during the TSH stimulation test. D. Below-normal levels of serum triiodothyronine (T3) and serum thyroxine (T4) as detected by radioimmunoassay.

No increase in the thyroid-stimulating hormone (TSH) level after 30 minutes during the TSH stimulation test In the TSH test, failure of the TSH level to rise after 30 minutes confirms hyperthyroidism. Option B: A decreased TSH level indicates a pituitary deficiency of this hormone. Option C: If the TSH level rises after 30 minutes, then the client has no hyperthyroidism. Option D: Below-normal levels of T3 and T4, as detected by radioimmunoassay, signal hypothyroidism. A below-normal T4 level also occurs in malnutrition and liver disease and may result from the administration of phenytoin and certain other drugs.

An infant with hydrocele is seen in the clinic for a follow-up visit at 1 month of age. The scrotum is smaller than it was at birth, but fluid is still visible on illumination. Which of the following actions is the physician likely to recommend? A. Massaging the groin area twice a day until the fluid is gone. B. Referral to a surgeon for repair. C. No treatment is necessary; the fluid is reabsorbing normally. D. Keeping the infant in a flat, supine position until the fluid is gone.

No treatment is necessary; the fluid is reabsorbing normally. A hydrocele is a collection of fluid in the scrotum that results from a patent tunica vaginalis. Illumination of the scrotum with a pocket light demonstrates the clear fluid. In most cases the fluid reabsorbed within the first few months of life and no treatment is necessary. Option A: Congenital hydrocele tends to be intermittent as it usually reduces when lying flat due to drainage of hydrocele fluid into the peritoneum. However, applying pressure on the congenital hydrocele does not reduce it. At birth, around 80-90% of term male infants possess a patent processus vaginalis. This figure declines steadily to settle at approximately 25-40% at two years of age. Option B: Surgery is the treatment of choice for hydrocele, and it is warranted when hydrocele becomes complicated or symptomatic. For congenital hydroceles, herniotomy is performed, provided they do not resolve spontaneously. On the other hand, acquired hydroceles subside when the primary underlying condition resolves. Option D: Placing the infant in a supine position would have no effect. The majority of patients with hydrocele present with the complaint of painless scrotal swelling rendering the testes impalpable with positive transillumination and fluctuation. The examiner should look at this swelling in both the supine and upright positions.

A nurse is assigned to the pediatric rheumatology clinic and is assessing a child who has just been diagnosed with juvenile idiopathic arthritis. Which of the following statements about the disease is most accurate? A. The child has a poor chance of recovery without joint deformity. B. Most children progress to adult rheumatoid arthritis. C. Nonsteroidal anti-inflammatory drugs are the first choice in treatment. D. Physical activity should be minimized.

Nonsteroidal anti-inflammatory drugs are the first choice in treatment. Nonsteroidal anti-inflammatory drugs are important first-line treatment for juvenile idiopathic arthritis (formerly known as juvenile rheumatoid arthritis). NSAIDs require 3-4 weeks for the therapeutic anti-inflammatory effects to be realized. Nonsteroidal anti-inflammatory drugs (NSAIDs) are the mainstay of initial symptomatic treatment for all subtypes. The NSAID use in JIA has decreased over time with modern aggressive treatment, including methotrexate and biologics. Option A: Half of the children with the disorder recover without joint deformity. The prognosis of JIA has changed dramatically in recent years thanks to the availability of novel drugs, which can inhibit the biological mechanisms responsible for persistent inflammation selectively. Prompt and accurate diagnosis and treatment are essential to prevent permanent joint damage and preserve joint functionality. Option B: About a third will continue with symptoms into adulthood. A recent study on 168 patients showed the remission of medication in 48.8% of cases, the remission on medication (or minimal disease activity) in 49.9% of cases, and only 1.3% of subjects were no-responders. No association was found between the state and duration of remission and age of patients, clinical features, disease course, or laboratory findings. Option D: Physical activity is an integral part of therapy. Assist parents and child to develop plans and goals for daily ADL and include interventions formed by a physical and occupational therapist. Promotes independence and compliance in self-care.

There are a number of risk factors associated with coronary artery disease. Which of the following is a modifiable risk factor? A. Obesity B. Heredity C. Gender D. Age

Obesity Obesity is an important risk factor for coronary artery disease that can be modified by improved diet and weight loss. Individuals are diagnosed as overweight when body mass index (BMI) is between 25 to 29.8 kg/m^2 and obese when BMI greater than or equal to 30 kg/m^2. Conditions of both overweight and obesity increase the ASCVD risk compared to normal weight. Recommendations include annual calculation of BMI, and lifestyle modification, including calorie restriction and weight loss, based on the BMI values. Option B: Coronary artery disease is a multifactorial phenomenon. Etiologic factors can be broadly categorized into non-modifiable and modifiable factors. Non-modifiable factors include gender, age, family history, and genetics. Option C: In developed countries like the U.S. and the UK, mortality rates due to ischemic heart diseases are decreasing. Nevertheless, according to AHA, 16.5 million people older than 20 in the U.S. had coronary artery disease in 2018, and 55% of them were males. The male gender is more predisposed than the female gender. Option D: Advancing age increases risk but cannot be modified. The incidence of CAD is observed to rise with age, regardless of gender. In the ONACI registry in France, the incidence of CAD was about 1% in the 45 to 65 age group, which increased to about 4% as the age group reached 75 to 84 years.

A nurse is caring for a patient with a platelet count of 20,000/microliter. Which of the following is an important intervention? A. Observe for evidence of spontaneous bleeding. B. Limit visitors to family only. C. Give aspirin in case of headaches. D. Impose immune precautions.

Observe for evidence of spontaneous bleeding. Platelet counts under 30,000/microliter may cause spontaneous petechiae and bruising, particularly in the extremities. When the count falls below 15,000, spontaneous bleeding into the brain and internal organs may occur. The blood clotting cascade is an integral system requiring intrinsic and extrinsic factors. Derangements in any factors can affect clotting ability. These laboratory tests provide important information about the patient's coagulation status and bleeding potential. The specific laboratory values to be monitored will depend on the patient's specific clinical condition. Option B: There is no reason to limit visitors as long as any physical trauma is prevented. Educate the patient and family members about signs of bleeding that need to be reported to a health care provider. Early evaluation and treatment of bleeding by a health care provider reduce the risk for complications from blood loss. Option C: Headaches may be a sign and should be watched for. Aspirin disables platelets and should never be used in the presence of thrombocytopenia. Educate the patient about over-the-counter drugs and avoid products that contain aspirin or NSAIDs such as ibuprofen and naproxen. These drugs not only decrease normal platelet aggregation but also decrease the integrity of gastric mucosa through inhibition of cyclooxygenase (COX)-1 inhibitor and therefore increase the risk for gastrointestinal bleeding. Option D: Thrombocytopenia does not compromise immunity. Educate the at-risk patient about precautionary measures to prevent tissue trauma or disruption of the normal clotting mechanisms. Information about precautionary measures lessens the risk for bleeding. Use a soft-bristled toothbrush and nonabrasive toothpaste. Avoid the use of toothpicks and dental floss.

Dr. Marquez orders a continuous intravenous nitroglycerin infusion for the client suffering from myocardial infarction. Which of the following is the most essential nursing action? A. Monitoring urine output frequently B. Monitoring blood pressure every 4 hours C. Obtaining serum potassium levels daily D. Obtaining infusion pump for the medication

Obtaining infusion pump for the medication Administration of Intravenous nitroglycerin infusion requires a pump for accurate control of medication. There are intravenous (IV) routes of administration for nitroglycerin used most commonly in emergency rooms and intensive care units (ICU). It is administered as a 5% dextrose in water drip and is indicated when sublingual nitroglycerin has failed to provide symptomatic relief or if rapid and continued relief of symptoms is necessary. When administered, its effect requires tight monitoring, as discussed below. Option A: There is no indication to monitor urine output frequently. In the event of overdose, monitoring of vital signs may be necessary to monitor the hemodynamic effects of nitroglycerin. Continuous monitoring of blood pressure, heart rate, respiratory rate, and oxygen saturation is recommended. Option B: Blood pressure must be monitored regularly, but it is not the most essential action. Any testing does not currently monitor nitroglycerin levels as its half-life is approximately 2 to 3 minutes, and the drug undergoes rapid metabolism from the body. When administered as a drip in the emergency room or ICU, its effects are often very closely monitored via an arterial line for real-time blood pressure monitoring. This vigilance is necessary to maximize the effectiveness of the drip and provide rapid feedback on the patient's condition. Option C: Monitoring serum potassium levels daily is unnecessary. Nitroglycerin is both a protein-bound drug and undergoes hepatic metabolism. Therefore it has numerous drug interactions. Before prescribing, providers should determine if the patient is taking any medications that may interact with nitroglycerin.

Right cerebral hemisphere stroke

Often present with neglect syndrome. They lean to the left and when asked, respond that they believe they are sitting up straight. They often neglect the left side of their bodies and ignore food on the left side of their food trays. The nurse would need to remind the student of this phenomenon and discuss the appropriate interventions.

Which of the following symptoms might a client with right-sided HF exhibit? Adequate urine output Polyuria Oliguria Polydipsia

Oliguria Inadequate deactivation of aldosterone by the liver after right-sided heart failure leads to fluid retention, which causes oliguria. Oliguria is a late finding in heart failure, and it is found in patients with markedly reduced cardiac output from severely reduced LV function. A) An adequate urine output indicates that the client does not have fluid retention. The activation of the RAAS leads to salt and water retention, resulting in increased preload and further increases in myocardial energy expenditure. This results in an increase in angiotensin II (Ang II) levels and, in turn, aldosterone levels, causing stimulation of the release of aldosterone. Ang II, along with ET-1, is crucial in maintaining effective intravascular homeostasis as mediated by vasoconstriction and aldosterone-induced salt and water retention. B) Polyuria is possible in a weak heart due to heart failure. A weak heart cannot pump blood efficiently, causing fluids to build up in the body. The kidneys work to rid the body of excess fluid, causing frequent urination. D) Polydipsia is usually associated with diabetes mellitus. In people with diabetes, polydipsia is caused by increased blood glucose levels. When blood glucose levels get high, the kidneys produce more urine in an effort to remove the extra glucose from the body. Meanwhile, because the body is losing fluids, the brain tells you to drink more in order to replace them.

Capillary glucose monitoring is being performed every 4 hours for a client diagnosed with diabetic ketoacidosis. Insulin is administered using a scale of regular insulin according to glucose results. At 2 p.m., the client has a capillary glucose level of 250 mg/dl for which he receives 8 U of regular insulin. Nurse Mariner should expect the dose's: A. Onset to be at 2 p.m. and its peak to be at 3 p.m. B. Onset to be at 2:15 p.m. and its peak to be at 3 p.m. C. Onset to be at 2:30 p.m. and its peak to be at 4 p.m. D. Onset to be at 4 p.m. and its peak to be at 6 p.m.

Onset to be at 2:30 p.m. and its peak to be at 4 p.m. Regular insulin, which is a short-acting insulin, has an onset of 15 to 30 minutes and a peak of 2 to 4 hours. Because the nurse gave the insulin at 2 p.m., the expected onset would be from 2:15 p.m. to 2:30 p.m. and the peak from 4 p.m. to 6 p.m. Option A: 2 p.m. is when the insulin was given; onset does not occur at the same time as the medication was given Option B: The peak starts 2 to 4 hours after the insulin was given, which will be at 4 p.m. Option D: Onset of 4 p.m. is very late; 15 to 30 minutes is the expected onset of insulin. Peak shoud start at 4 p.m.

Coronary Artery Disease

Oral medication administration is a noninvasive, medical treatment for CAD. Nitroglycerin is the most common vasodilator used for acute cases of angina. It works to dilate or widen the coronary arteries, increasing blood flow to the heart muscles and to relax the veins, lessening the amount of blood that returns to the heart from the body. This combination of effects decreases the amount of work for the heart. Cardiac catheterization isn't a treatment but a diagnostic tool. It is a procedure to examine how well the heart is working. A thin, hollow tube called a catheter is inserted into a large blood vessel that leads to the heart. Coronary artery bypass surgery is an invasive procedure. Coronary artery bypass grafting (CABG) is a procedure to improve poor blood flow to the heart. It may be needed when the arteries supplying blood to the heart tissue, called coronary arteries, are narrowed or blocked. This surgery may lower the risk of serious complications for people who have obstructive coronary artery disease, a type of ischemic heart disease. CABG may also be used in an emergency, such as a severe heart attack. Percutaneous transluminal coronary angioplasty (PTCA) is an invasive, surgical treatment. PTCS is a minimally invasive procedure to open up blocked coronary arteries, allowing blood to circulate unobstructed to the heart muscle.

Norma asks for information about osteoarthritis. Which of the following statements about osteoarthritis is correct? A. Osteoarthritis is rarely debilitating. B. Osteoarthritis is a rare form of arthritis. C. Osteoarthritis is the most common form of arthritis. D. Osteoarthritis affects people over 60.

Osteoarthritis is the most common form of arthritis Osteoarthritis is the most common form of arthritis and can be extremely debilitating. It can afflict people of any age, although most are elderly. Option A: Osteoarthritis is an extremely debilitating disease. The cartilage within a joint begins to break down and the underlying bone begins to change. Option B: It is the most common form of arthritis. It affects over 32.5 million US adults. Option D: Osteoarthritis can affect people of any age, but are most common among the elderly. Women are more likely to develop QA than men, especially after the age of 50.

Nurse Nikki knows that laboratory results supports the diagnosis of systemic lupus erythematosus (SLE) is: A. Elevated serum complement level B. Thrombocytosis, elevated sedimentation rate C. Pancytopenia, elevated antinuclear antibody (ANA) titer D. Leukocytosis, elevated blood urea nitrogen (BUN) and creatinine levels

Pancytopenia, elevated antinuclear antibody (ANA) titer Laboratory findings for clients with SLE usually show pancytopenia, elevated ANA titer, and decreased serum complement levels. Option A: Decreased levels of serum complement is usually associated with SLE. The cause of complement activation in SLE is the formation of immune complexes, which in turn activate complement, predominantly by means of the classical pathway. Option B: Thrombocytopenia is one of the components of pancytopenia. It is a condition in which the platelet count is decreased. Option D: Clients may have elevated BUN and creatinine levels from nephritis, but the increase does not indicate SLE. The part of the kidney most frequently troubled by SLE is part of the nephron called the glomerulus, a tuft of capillaries that functions to filter substances from the blood. For this reason, the type of kidney inflammation most commonly experienced in lupus is glomerulonephritis.

Which of the following treatments is a suitable surgical intervention for a client with unstable angina? A. Cardiac catheterization B. Echocardiogram C. Nitroglycerin D. Percutaneous transluminal coronary angioplasty (PTCA)

Percutaneous transluminal coronary angioplasty (PTCA) PTCA can alleviate the blockage and restore blood flow and oxygenation. Option A: Cardiac catheterization is a diagnostic tool - not a treatment. It is a procedure used to diagnose and treat certain cardiovascular conditions. Option B: An echocardiogram is a non-invasive diagnostic test. It is a graphic outline of the heart's movement. Option C: Nitroglycerin is an oral sublingual medication. It is a vasodilatory drug used primarily to provide relief from anginal chest pain.

The nurse is assisting the physician with the removal of a central venous catheter. To facilitate removal, the nurse should instruct the client to: A. Perform the Valsalva maneuver as the catheter is advanced B. Turn his head to the left side and hyperextend the neck C. Take slow, deep breaths as the catheter is removed D. Turn his head to the right while maintaining a sniffing position

Perform the Valsalva maneuver as the catheter is advanced The client who is having a central venous catheter removed should be told to hold his breath and bear down. This prevents air from entering the line. Option B: Although there are many steps in the process of CVC removal, essential elements of the procedure include (for internal jugular and subclavian CVCs), positioning of the patient in the head down (Trendelenburg) position, having the patient perform a Valsalva maneuver as the catheter is being withdrawn, application of pressure to the catheter-entry site as the catheter is being withdrawn, placement of an air-occlusive dressing over the site after removal, and a period of post-procedure monitoring. Option C: The patient is asked to take a deep breath, hold it, and bear down during the removal to avoid introduction of an air embolism. Breath-holding or Valsalva maneuver will increase intracardiac pressures. Immediate occlusion is required to prevent air embolism. Option D: The patient's head should be placed in a supine position with the head of the bed flat to reduce the risk of air embolism. Risk for air embolism increases when the catheter insertion site is above heart level (e.g., in a sitting position), if the patient is hypovolemic or during spontaneous inspiration.

A male client is scheduled for a transsphenoidal hypophysectomy to remove a pituitary tumor. Preoperatively, the nurse should assess for potential complications by doing which of the following? A. Testing for ketones in the urine. B. Testing urine specific gravity. C. Checking temperature every 4 hours. D. Performing capillary glucose testing every 4 hours.

Performing capillary glucose testing every 4 hours. The nurse should perform capillary glucose testing every 4 hours because excess cortisol may cause insulin resistance, placing the client at risk for hyperglycemia. Option A: Urine ketone testing isn't indicated because the client does secrete insulin and, therefore, isn't at risk for ketosis. Option B: Urine specific gravity isn't indicated because although fluid balance can be compromised, it usually isn't dangerously imbalanced. Option C: Temperature regulation may be affected by excess cortisol and isn't an accurate indicator of infection.

A child is seen in the emergency department for scarlet fever. Which of the following descriptions of scarlet fever is not correct? A. Scarlet fever is caused by infection with group A Streptococcus bacteria. B. "Strawberry tongue" is a characteristic sign. C. Petechiae occur on the soft palate. D. The pharynx is red and swollen.

Petechiae occur on the soft palate. Petechiae on the soft palate are characteristic of rubella infection. Postnatal infection with rubella can be asymptomatic in approximately 25% to 50% of the patients, especially in young children. The incubation period ranges from 14 to 21 days and is followed by a prodromal illness characterized by low-grade fever, malaise, anorexia, headaches, sore throat, and adenopathy. Option A: It is caused by streptococcal pyrogenic exotoxins (SPEs) types A, B, and C produced by group A beta-hemolytic streptococci (GABHS) found in secretions and discharge from the nose, ears, throat, and skin. The causative bacteria is Streptococcus pyogenes, which generates an endotoxin mainly responsible for the skin manifestation of the infection. This is further classified as group A and referred to as Group A Strep (GAS). Option B: On day 1 or 2, the tongue is heavily coated with a white membrane through which edematous red papillae protrude (classic appearance of white strawberry tongue). By day 4 or 5, the white membrane sloughs off, revealing a shiny red tongue with prominent papillae (red strawberry tongue). Option D: Red, edematous, exudative tonsils are typically observed if the infection originates in this area. Typically, scarlet fever is associated with acute pharyngitis. As a result, fever, sore throat, pain with swallowing, and cervical adenopathy is present.

A 62-year-old male client was in a motor vehicle accident as an unrestrained driver. He's now in the emergency department complaining of difficulty of breathing and chest pain. On auscultation of his lung field, no breath sounds are present in the upper lobe. This client may have which of the following conditions? A. Bronchitis B. Pneumonia C. Pneumothorax D. Tuberculosis (TB)

Pneumothorax Pneumothorax is defined as the presence of air or gas in the pleural cavity, which can impair oxygenation and/or ventilation. Option A: Rhonchi is heard with bronchitis. Option B: From the trauma the client experienced, it's unlikely he has pneumonia. Pneumonia may produce crackling, bubbling, and rumbling sounds when inhaling upon auscultation. Option D: Bronchial breath sounds with TB would be heard.

What supplemental medication is most frequently ordered in conjunction with furosemide (Lasix)? Chloride Digoxin Potassium Sodium

Potassium Supplemental potassium is given with furosemide because of the potassium loss that occurs as a result of this diuretic. Loop diuretics act at the ascending loop of Henle in the kidney and help the body push out extra fluid that could accumulate in the lungs or legs and ankles when the heart is unable to completely pump blood throughout the body. But they may also cause the body to eliminate excessive amounts of potassium, which might be expected to increase mortality from heart arrhythmias. As a precaution, therefore, many doctors prescribe potassium supplements to their patients receiving loop diuretics. A) Chloride isn't lost during diuresis. Continues use of diuretics will cause some overall sodium and chloride loss. The body, however, has a natural way of compensating for theses losses by reducing the excretion of sodium and chloride and stabilizing the amount of sodium, chloride, and water in the body. In this manner, fluid depletion usually is depleted. B) Digoxin acts to increase contractility but isn't given routinely with furosemide. People with heart failure who take digoxin are commonly given medicines called diuretics. These drugs remove excess fluid from the body. Many diuretics can cause potassium loss. A low level of potassium in the body can increase the risk of digitalis toxicity. D) Sodium is not lost during diuresis. Diuretic drugs increase urine output by the kidney. This is accomplished by altering how the kidney handles sodium. If the kidney excretes more sodium, then water excretion will also increase.

Patrick who is hospitalized following a myocardial infarction asks the nurse why he is taking morphine. The nurse explains that morphine: A. Decrease anxiety and restlessness B. Prevents shock and relieves pain C. Dilates coronary blood vessels D. Helps prevent fibrillation of the heart

Prevents shock and relieves pain Morphine is a central nervous system depressant used to relieve the pain associated with myocardial infarction, it also decreases apprehension and prevents cardiogenic shock. FDA-approved usage of morphine sulfate includes moderate to severe pain that may be acute or chronic. Most commonly used in pain management, morphine provides major relief to patients afflicted with pain. Option A: Benzodiazepines decrease anxiety and restlessness. Indications for benzodiazepine administration include, but are not limited to, anxiety disorders, insomnia, acute status epilepticus, induction of amnesia, spastic disorders, seizure disorders, and agitation. Option C: Calcium channel blockers, such as nitroglycerin, dilate large coronary blood vessels. The dihydropyridines, in therapeutic dosing, have a little direct effect on the myocardium, and instead, are more often peripheral vasodilators, which is why they are useful for hypertension, post-intracranial hemorrhage associated vasospasm, and migraines. Option D: Antiplatelets and anticoagulants help prevent atrial fibrillation by preventing blood clot formation. Antiplatelet medications divide into oral and parenteral agents. Oral agents subdivide further based on the mechanism of action. Aspirin was the first antiplatelet medication and is a cyclooxygenase inhibitor.

Which of the following conditions most commonly causes acute glomerulonephritis? A. A congenital condition leading to renal dysfunction. B. Prior infection with group A Streptococcus within the past 10-14 days. C. Viral infection of the glomeruli. D. Nephrotic syndrome.

Prior infection with group A Streptococcus within the past 10-14 days. Acute glomerulonephritis is most commonly caused by the immune response to a prior upper respiratory infection with group A Streptococcus. PSGN most commonly presents in children 1 to 2 weeks after a streptococcal throat infection, or within 6 weeks following a streptococcal skin infection. Group A Streptococcus (GAS) has been subtyped depending on the surface M protein and opacity factor, which are known to be nephrogenic and can cause PSGN. Option A: Glomerulonephritis is not a congenital condition. Nephrogenic streptococci infection precedes PSGN, which initially affects skin or oropharynx. More recently, PSGN is associated with skin infections (impetigo) more frequently than throat infections (pharyngitis). Option C: Glomerular lesions in acute GN are the result of glomerular deposition or in situ formation of immune complexes. Poor hygiene, overcrowding, and low socioeconomic status are important risk factors for streptococci outbreaks, and this explains the higher incidence of PSGN in impoverished countries. Genetic factors are expected to predispose to the condition since almost 40% of patients with PSGN gave a positive family history. There is no specific gene found to cause PSGN. Option D: Nephrotic syndrome is the combination of nephrotic-range proteinuria with a low serum albumin level and edema. It is caused by increased permeability through the damaged basement membrane in the renal glomerulus, especially infectious or thrombo-embolic. It is the result of an abnormality of glomerular permeability that may be primary with a disease-specific to the kidneys or secondary to congenital infections, diabetes, systemic lupus erythematosus, neoplasia, or certain drug use.

Which of the following strategies is not effective for the prevention of Lyme disease? A. Insect repellant on the skin and clothes when in a Lyme endemic area. B. Long sleeved shirts and long pants. C. Prophylactic antibiotic therapy prior to anticipated exposure to ticks. D. Careful examination of skin and hair for ticks following anticipated exposure.

Prophylactic antibiotic therapy prior to anticipated exposure to ticks. Prophylactic use of antibiotics is not indicated to prevent Lyme disease. Antibiotics are used only when symptoms develop following a tick bite. Specific treatment is dependent upon the age of the patient and stage of the disease. For patients older than 8 years of age with early, localized disease, doxycycline is recommended for 10 days. Patients under the age of 8 should receive amoxicillin or cefuroxime for 14 days to avoid the potential for tooth staining caused by tetracycline use in young children. Option A: Insect repellant should be used on skin and clothing when exposure is anticipated. While there are many repellants on the market, it is best to avoid them as the risk of harm is greater than any benefit. If one is going to use a repellant, DEET is the one product that is safe, however, it is not 100% effective. Option B: Clothing should be designed to cover as much exposed area as possible to provide an effective barrier. The outdoors person should be told to wear appropriate garments and be familiar with the skin features of the tick bite. The nurse should educate the patient on how to remove the tick from the skin and when to seek medical assistance. Option D: Close examination of skin and hair can reveal the presence of a tick before a bite occurs. Nurses should educate parents on how to inspect their children for ticks at the end of an outdoor event, in an endemic area. Pets can also develop Lyme disease and carry the tick. Hence, pet owners should examine their pets on a regular basis and remove the tick. There is no risk of acquiring Lyme disease by removing the tick

Kate, who has undergone mitral valve replacement, suddenly experiences continuous bleeding from the surgical incision during the postoperative period. Which of the following pharmaceutical agents should Nurse Aiza prepare to administer to Kate? A. Protamine Sulfate B. Quinidine Sulfate C. Vitamin C D. Coumadin

Protamine Sulfate Protamine Sulfate is used to prevent continuous bleeding in a client who has undergone open heart surgery. Protamine is a medication used to reverse and neutralize the anticoagulant effects of heparin. Protamine is the specific antagonist that neutralizes heparin-induced anticoagulation. Protamine is a strongly alkaline (nearly two-thirds of the amino acid composition is arginine) polycationic low-molecular-weight protein found in salmon sperm that is also currently available in a recombinant form. Option B: Quinidine sulfate is used to treat or prevent many types of irregular heartbeats such as atrial fibrillation. Quinidine can greatly improve the ability to perform normal activities by decreasing the number of irregular heartbeats. However, it may not stop all irregular heartbeats completely. It works by blocking abnormal heartbeat signals. Option C: Vitamin C, or ascorbic acid, has several important functions: helping to protect cells and keeping them healthy, maintaining healthy skin, blood vessels, bones, and cartilage, and helping with wound healing. Option D: Coumadin is used to treat blood clots and/or prevent new clots from forming in the body. Preventing harmful blood clots helps to reduce the risk of a stroke or heart attack. Conditions that increase the risk of developing blood clots include a certain type of irregular heart rhythm (atrial fibrillation), heart valve replacement, recent heart attack, and certain surgeries (such as hip/knee replacement).

A 23-year-old patient in the 27th week of pregnancy has been hospitalized on complete bed rest for 6 days. She experiences sudden shortness of breath, accompanied by chest pain. Which of the following conditions is the most likely cause of her symptoms? A. Myocardial infarction due to a history of atherosclerosis. B. Pulmonary embolism due to deep vein thrombosis (DVT). C. Anxiety attacks due to worries about her baby's health. D. Congestive heart failure due to fluid overload.

Pulmonary embolism due to deep vein thrombosis (DVT). In a hospitalized patient on prolonged bed rest, the most likely cause of sudden onset shortness of breath and chest pain is pulmonary embolism. Pregnancy and prolonged inactivity both increase the risk of clot formation in the deep veins of the legs. These clots can then break loose and travel to the lungs. Most pulmonary embolisms originate as lower extremity DVTs. Hence, risk factors for pulmonary embolism (PE) are the same as risk factors for DVT. Virchow's triad of hypercoagulability, venous stasis, and endothelial injury provides an understanding of these risk factors. Option A: Smoking and abnormal apolipoprotein ratio showed the strongest association with acute myocardial infarction. The increased risk associated with diabetes and hypertension were found to be higher in women, and the protective effect of exercise and alcohol was also found to be higher in women. Option C: There is no reason to suspect an anxiety disorder in this patient. Pregnancy is one of the most important events in women's lives. Being pleasant, it is one of the most stressful events in a woman's life, as psychologists have cited, pregnancy as an emotional crisis. Though anxiety is a possible cause of her symptoms, the seriousness of pulmonary embolism demands that it be considered first. Option D: Myocardial infarction and atherosclerosis are unlikely in a 27-year-old woman, as is congestive heart failure due to fluid overload. Heart failure incidence has remained stable over the past decades, with more than 650,000 new cases of heart failure cases diagnosed annually, especially for individuals greater than 65 years of age. Because prevalence is greater in this age group, heart failure prevalence is expected to worsen in the near future.

A murmur is heard at the second left intercostal space along the left sternal border. Which valve area is this? Aortic Mitral Pulmonic Tricuspid

Pulmonic Abnormalities of the pulmonic valve are auscultated at the second left intercostal space along the left sternal border. Murmurs of the cardiac system develop due to alterations in blood flow or mechanical operation. Murmurs develop from a multitude of mechanisms. Typical cases include low blood viscosity from anemia, septal defects, failure of the ductus arteriosus to close in newborns, excessive hydrostatic pressure on cardiac valves causing valve failure, hypertrophic obstructive cardiomyopathy, and valvular specific pathologies. A) Aortic valve abnormalities are heard at the second intercostal space, to the right of the sternum. Aortic regurgitation, also known as aortic insufficiency, is a decrescendo blowing diastolic murmur heard best at the left lower sternal border, heard when blood flow retrograde into the left ventricle. This is most commonly seen in aortic root dilation and as sequelae of aortic stenosis. B) Mitral valve abnormalities are heard at the fifth intercostal space in the midclavicular line. Mitral stenosis is a diastolic murmur, best heard at the 5th midclavicular line. It is associated with infective endocarditis and chronic rheumatic heart disease. Mitral regurgitation is a systolic murmur, best heard at the left 5th midclavicular line with possible radiation to the left axilla. It is commonly associated with infective endocarditis, rheumatic heart disease, congenital anomalies, and inferior wall myocardial infarctions. D) Tricuspid valve abnormalities are heard at the third and fourth intercostal spaces along the sternal border. Tricuspid stenosis is best heard at the lower left sternal border. Typical causes include infective endocarditis, seen in intravenous drug users, and carcinoid syndrome. Prolonged tricuspid stenosis may lead to right atrial enlargement and arrhythmias. Tricuspid regurgitation is systolic, auscultated at the lower left sternal border. It is also associated with intravenous drug users and carcinoid syndrome.

The Hodgkin's disease patient described in the question above undergoes a lymph node biopsy for definitive diagnosis. If the diagnosis of Hodgkin's disease were correct, which of the following cells would the pathologist expect to find? A. Reed-Sternberg cells. B. Lymphoblastic cells. C. Gaucher's cells. D. Rieder's cells

Reed-Sternberg cells A definitive diagnosis of Hodgkin's disease is made if Reed-Sternberg cells are found on pathologic examination of the excised lymph node. Four features characterize Hodgkin lymphomas. They commonly arise in the cervical lymph nodes; the disease is more common in young adults; there are scattered large mononuclear Hodgkin and multinucleated cells (Reed-Sternberg) intermixed in a background of a mixture of non-neoplastic inflammatory cells; finally, T lymphocytes are often observed surrounding the characteristic neoplastic cells. Option B: Lymphoblasts are immature cells found in the bone marrow of patients with acute lymphoblastic leukemia. Lymphoblast is an immature white blood cell that gives rise to a type of immune cell known as a lymphocyte. The nucleus contains moderately fine chromatin (readily stainable nuclear material) and has a well-defined nuclear membrane. There are one or two nucleoli, and the cytoplasm is small or moderate in amount. Lymphoblasts that grow and divide uncontrollably cause a type of cancer known as acute lymphoblastic leukemia. Option C: Gaucher's cells are large storage cells found in patients with Gaucher's disease. The Gaucher cell results from the accumulation of excessive glucocerebroside in cells of the monocyte-macrophage system. It is characterized ultrastructurally by the presence of cytoplasmic inclusions which consist of tubule-like structures measuring 130 to 150 Ao in diameter. Option D: Rieder's cells are myeloblasts found in patients with acute myelogenous leukemia. They are abnormal myeloblasts in which the nucleus may be widely or deeply indented or may actually be a bilobate or multilobate structure; frequently observed in acute leukemia.

A client with hypothyroidism asks the nurse if she will still need to take thyroid medication during the pregnancy. The nurse's response is based on the knowledge that: A. There is no need to take thyroid medication because the fetus's thyroid produces a thyroid-stimulating hormone. B. Regulation of thyroid medication is more difficult because the thyroid gland increases in size during pregnancy. C. It is more difficult to maintain thyroid regulation during pregnancy due to a slowing of metabolism. D. Fetal growth is arrested if thyroid medication is continued during pregnancy.

Regulation of thyroid medication is more difficult because the thyroid gland increases in size during pregnancy. During pregnancy, the thyroid gland triples in size. This makes it more difficult to regulate thyroid medication. During pregnancy, there are increased metabolic needs of the maternal body resulting in changes in thyroid physiology. These changes in thyroid physiology reflect in altered thyroid function tests. Option A: There could be a need for thyroid medication during pregnancy. The serum TSH concentration is the initial and most reliable measure of thyroid function during pregnancy. As elaborated above, there are physiologic changes in TSH levels during pregnancy which warrants close monitoring of TSH levels. As per the latest American Thyroid Association (ATA) guidelines, serum TSH levels during pregnancy should be defined using population and trimester-specific based reference ranges. Option C: The thyroid function does not slow. When population and trimester-specific normal ranges are not available, the ATA guidelines recommend reducing the lower limit of TSH by 0.4 mU/L and the upper limit by 0.5 mU/L. It would correspond to the TSH reference range of 0.1 to 4.0 mU/L during the first trimester with a gradual return of TSH towards the non-pregnant normal range during second and third trimesters. Option D: Fetal growth is not arrested if thyroid medication is continued. There is an increase in iodine requirement during pregnancy due to an increase in maternal thyroid hormone production as well as an increase in renal iodine clearance. Along with the above two factors, there is also a fetal iodine requirement; therefore, dietary iodine requirements are higher during pregnancy.

Marco falls off his bicycle and injures his ankle. Which of the following actions shows the initial response to the injury in the extrinsic pathway? Release of calcium. Release of tissue thromboplastin. Conversion of factors XII to factor XIIa. Conversion of factor VII to factor VIIa

Release of tissue thromboplastin. released when damaged tissue come in contact with clotting factors. A) calcium is released to assist the conversion of factors X to Xa. C) conversion of factors XII to XIIa are part of the intrinsic pathway D) conversion factors VII to VIIa are part of the intrinsic pathway

A nurse in the emergency department is observing a 4-year-old child for signs of increased intracranial pressure after a fall from a bicycle, resulting in head trauma. Which of the following signs or symptoms would be cause for concern? A. Bulging anterior fontanel. B. Repeated vomiting. C. Signs of sleepiness at 10 PM. D. Inability to read short words from a distance of 18 inches.

Repeated vomiting. Increased pressure caused by bleeding or swelling within the skull can damage delicate brain tissue and may become life threatening. Repeated vomiting can be an early sign of pressure as the vomit center within the medulla is stimulated. Clinical suspicion for intracranial hypertension should be raised if a patient presents with the following signs and symptoms: headaches, vomiting, and altered mental status varying from drowsiness to coma. Option A: The anterior fontanel is closed in a 4-year-old child. The average closure time of the anterior fontanelle ranges from 13 to 24 months. Infants of African descent statically have larger fontanelles that range from 1.4 to 4.7 cm, and in terms of sex, the fontanelles of male infants will closer sooner compared to female infants. Option C: Evidence of sleepiness at 10 PM is normal for a four year old. Newborns spend most of their day sleeping, and they only wake up to be fed, on the other hand, 1-year-old infants sleep for 10 to 12 hours at night without waking. The coordination between biological rhythm and sleep-wake cycle develops over the first six months of life. Option D: The average 4-year-old child cannot read yet, so this too is normal. Most children learn to read by 6 or 7 years of age. Some children learn at 4 or 5 years of age. Even if a child has a head start, she may not stay ahead once school starts. The other students most likely will catch up during the second or third grade.

Timothy's arterial blood gas (ABG) results are as follows; pH 7.16; Paco2 80 mm Hg; Pao2 46 mm Hg; HCO3- 24 mEq/L; Sao2 81%. This ABG result represents which of the following conditions? A. Metabolic acidosis B. Metabolic alkalosis C. Respiratory acidosis D. Respiratory alkalosis

Respiratory acidosis Because Paco2 is high at 80 mm Hg and the metabolic measure, HCO3- is normal, the client has respiratory acidosis. Option A: If the HCO3- was below 22 mEq/L the client would have metabolic acidosis. Option B: The result of the ABG is less than 7.35, which makes metabolic alkalosis incorrect. Option D: The pH is less than 7.35, academic, which eliminates respiratory alkalosis as a possibility.

Alvin with a massive pulmonary embolism will have an arterial blood gas analysis performed to determine the extent of hypoxia. The acid-base disorder that may be present is? A. Metabolic acidosis B. Metabolic alkalosis C. Respiratory acidosis D. Respiratory alkalosis

Respiratory alkalosis A client with massive pulmonary embolism will have a large region and blow off large amounts of carbon dioxide, which crosses the unaffected alveolar-capillary membrane more readily than does oxygen and results in respiratory alkalosis. Option A: Large amounts of carbon dioxide are blown off, removing the option of metabolic acidosis. Option B: Respiratory, not metabolic, alkalosis is the result of a massive pulmonary embolism. Option C: Acidosis does not occur with pulmonary embolism. Hypocapnia usually is present with an embolism.

Norma has started a new drug for hypertension. Thirty minutes after she takes the drug, she develops chest tightness and becomes short of breath and tachypnea. She has a decreased level of consciousness. These signs indicate which of the following conditions? A. Asthma attack B. Pulmonary embolism C. Respiratory failure D. Rheumatoid arthritis

Respiratory failure The client was reacting to the drug with respiratory signs of impending anaphylaxis, which could lead to eventually respiratory failure. Option A: The symptoms may look like an asthma attack, but it may change because of the new drug ingested. Option B: Although the signs are also related to a pulmonary embolism, consider the new drug first. Option D: Rheumatoid arthritis is an autoimmune and inflammatory disease, which means that the immune system attacks healthy cells in the body by mistake, causing inflammation in the affected parts of the body.

A child is admitted to the hospital with an uncontrolled seizure disorder. The admitting physician writes orders for actions to be taken in the event of a seizure. Which of the following actions would not be included? A. Notify the physician. B. Restrain the patient's limbs. C. Position the patient on his/her side with the head flexed forward. D. Administer rectal diazepam.

Restrain the patient's limbs. During a witnessed seizure, nursing actions should focus on securing the patient's safety and curtailing the seizure. Restraining the limbs is not indicated because strong muscle contractions could cause injury. Use and pad side rails with the bed in lowest position, or place the bed up against the wall and pad floor if rails are not available or appropriate. Option A: The nurse should notify the physician in the event of a seizure so he could prescribe the correct medication. Ascertain knowledge of various stimuli that may precipitate seizure activity. Alcohol, various drugs, and other stimuli (loss of sleep, flashing lights, prolonged television viewing) may increase brain activity, thereby increasing the potential for seizure activity. Option C: A side-lying position with head flexed forward allows for drainage of secretions and prevents the tongue from falling back, blocking the airway. Turn head to side and suction airway as indicated. Insert plastic bite blocks only if the jaw is relaxed. Helps maintain airway patency and reduces the risk of oral trauma but should not be "forced" or inserted when teeth are clenched because dental and soft-tissue damage may result. Note: Wooden tongue blades should not be used because they may splinter and break in the patient's mouth. Option D: Rectal diazepam may be a treatment ordered by the physician, who should be notified of the seizure. Diazepam may be used alone (or in combination with phenobarbital) to suppress status seizure activity. Diastat, a gel, may be administered rectally, even in the home setting, to reduce the frequency of seizures and need for additional medical care.

Dyspnea, cough, expectoration, weakness, and edema are classic signs and symptoms of which of the following conditions?

Restrictive These are the classic symptoms of heart failure. Heart failure is the pathophysiologic state in which the heart, via an abnormality of cardiac function (detectable or not), fails to pump blood at a rate commensurate with the requirements of the metabolizing tissues or is able to do so only with an elevated diastolic filling pressure. Option A: Pericarditis is exhibited by a feeling of fullness in the chest and auscultation of a pericardial friction rub. Option B: Hypertension is usually exhibited by headaches, visual disturbances, and a flushed face. Myocardial infarction causes heart failure but isn't related to these symptoms. Option C: Obliterative cardiomyopathy is very rare. It may result from the end stage of eosinophilic syndromes, in which intracavitary thrombus fills the left ventricular apex and hampers the filling of the ventricles.

If a client requires a pneumonectomy, what fills the area of the thoracic cavity? A. The space remains filled with air only. B. The surgeon fills the space with a gel. C. Serous fluids fill the space and consolidate the region. D. The tissue from the other lung grows over to the other side.

Serous fluids fill the space and consolidate the region Serous fluid fills the space and eventually consolidates, preventing extensive mediastinal shift of the heart and remaining lung. Option A: Air can't be left in space. Air in the chest cavity is called a pneumothorax, and it may cause the lungs to collapse. Option B: There's no gel that can be placed in the pleural space. The pleural cavity is the space that lies between the pleura, the two thin membranes that line and surround the lungs. It contains a small amount of liquid known as pleural fluid. Option D: The tissue from the other lung can't cross the mediastinum, although a temporary mediastinal shift exists until space is filled.

Which of the following groups of symptoms indicated a ruptured abdominal aneurysm?

Severe lower back pain, decreased BP, decreased RBC, increased WBC Severe lower back pain indicates an aneurysm rupture, secondary to pressure being applied within the abdominal cavity. When a rupture occurs, the pain is constant because it can't be alleviated until the aneurysm is repaired. Blood pressure decreases due to the loss of blood. After the aneurysm ruptures, the vasculature is interrupted and blood volume is lost, so blood pressure wouldn't increase. For the same reason, the RBC count has decreased - not increased. The WBC count increases as cells migrate to the site of injury. The pain felt during rupture is severe. Due to the loss of blood, the blood pressure decreases. The WBC count increases because the cells migrate to the site of injury. The pain is not intermittent during an aneurysm; it is constant and severe.

Which of the following groups of symptoms indicates a ruptured abdominal aortic aneurysm? A. Lower back pain, increased blood pressure, decreased red blood cell (RBC) count, increased white blood (WBC) count. B. Severe lower back pain, decreased blood pressure, decreased RBC count, increased WBC count. C. Severe lower back pain, decreased blood pressure, decreased RBC count, decreased RBC count, decreased WBC count. D. Intermittent lower back pain, decreased blood pressure, decreased RBC count, increased WBC count.

Severe lower back pain, decreased blood pressure, decreased RBC count, increased WBC count. Severe lower back pain indicates an aneurysm rupture, secondary to pressure being applied within the abdominal cavity. When rupture occurs, the pain is constant because it can't be alleviated until the aneurysm is repaired. Blood pressure decreases due to the loss of blood. After the aneurysm ruptures, the vasculature is interrupted and blood volume is lost, so blood pressure wouldn't increase. For the same reason, the RBC count has decreased - not increased. The WBC count increases as cells migrate to the site of injury. Option A: The pain is severe due to the ruptured aneurysm; the blood pressure is decreased due to blood loss. Option C: The increase in WBC count is due to the cells migrating to the site of the injury. Option D: The pain in a ruptured aneurysm is constant and can only be alleviated if the aneurysm is repaired.

A patient returns to the emergency department less than 24 hours after having a fiberglass cast applied for a fractured right radius. Which of the following patient complaints would cause the nurse to be concerned about impaired perfusion to the limb? A. Severe itching under the cast. B. Severe pain in the right shoulder. C. Severe pain in the right lower arm. D. Increased warmth in the fingers.

Severe pain in the right lower arm. Impaired perfusion to the right lower arm as a result of a closed cast may cause neurovascular compromise and severe pain, requiring immediate cast removal. When there is an increase in compartmental pressure, there is a reduction in the venous outflow. This causes venous pressure and, thus, venous capillary pressure to increase. If the intracompartmental pressure becomes higher than arterial pressure, a decrease in arterial inflow will also occur. The reduction of venous outflow and arterial inflow result in decreased oxygenation of tissues causing ischemia. Option A: Itching under the cast is common and fairly benign. A cast can cause the client's underlying skin to feel itchy. To relieve itchy skin, turn a hair dryer on a cool setting and aim it under the cast. Option B: Neurovascular compromise in the arm would not cause pain in the shoulder, as perfusion there would not be affected. Pain is typically severe, out of proportion to the injury. Early on, pain may only be present with passive stretching. However, this symptom may be absent in advanced acute compartment syndrome. In the initial stages, pain may be characterized as a burning sensation or as a deep ache of the involved compartment. Option D: Impaired perfusion would cause the fingers to be cool and pale. Increased warmth would indicate increased blood flow or infection. Classically, the presentation of acute compartment syndrome has been remembered by "The Five P's": pain, pulselessness, paresthesia, paralysis, and pallor. However, aside from paresthesia, which may occur earlier in the course of the condition, these are typically late findings.

A nonimmunized child appears at the clinic with a visible rash. Which of the following observations indicates the child may have rubeola (measles)? A. Small blue-white spots are visible on the oral mucosa. B. The rash begins on the trunk and spreads outward. C. There is low-grade fever. D. The lesions have a "teardrop on a rose petal" appearance.

Small blue-white spots are visible on the oral mucosa. Koplik's spots are small blue-white spots visible on the oral mucosa and are characteristic of measles infection. Most cases show the characteristic Koplik spots of the disease, located in the buccal mucosa at the height of the second molar, and appear two to three days before the rash and disappear on the third day. Option B: The body rash typically begins on the face and travels downward. The second phase, the eruptive, is characterized by the appearance of a maculopapular rash, initially fine that subsequently becomes confluent. The rash begins behind the auricle and along the hair implantation line, and extends downward to the face, trunk, and extremities. Option C: High fever is often present. The primary or prodromal phase lasts four to six days and is characterized by the presence of high fever, malaise, coryza, conjunctivitis, palpebral edema, and dry cough. Option D: "Teardrop on a rose petal" refers to the lesions found in varicella (chickenpox). The characteristic chickenpox vesicle, surrounded by an erythematous halo, is described as a dewdrop on a rose petal. Chickenpox is clinically characterized by the presence of active and healing lesions in all stages of development within affected locations. Lesions characteristically heal without scarring, although excoriation or secondary bacterial superinfection predisposes to scar formation.

A client with shortness of breath has decreased to absent breath sounds on the right side, from the apex to the base. Which of the following conditions would best explain this? A. Acute asthma B. Chronic bronchitis C. Pneumonia D. Spontaneous pneumothorax

Spontaneous pneumothorax A spontaneous pneumothorax occurs when the client's lung collapses, causing an acute decrease in the amount of functional lung used in oxygenation. The sudden collapse was the cause of his chest pain and shortness of breath. Option A: An asthma attack would show wheezing breath sounds. Option B: Bronchitis would have rhonchi. Option C: Pneumonia would have bronchial breath sounds over the area of consolidation.

You are helping the patient with an SCI to establish a bladder retraining program. What strategies may stimulate the patient to void? SATA Stroke the patient's inner thigh. Pull on the patient's pubic hair. Initiate intermittent straight catheterization. Pour warm water over the perineum. Tap the bladder to stimulate detrusor muscle.

Stroke the patient's inner thigh, pull on the patient's pubic hair, pour warm water over the perineum, and tap the bladder to stimulate detrusor muscle. All of these strategies, except straight catheterization, may stimulate voiding in patients with SCI.

An infant with congestive heart failure is receiving diuretic therapy at home. Which of the following symptoms would indicate that the dosage may need to be increased? A. Sudden weight gain B. Decreased blood pressure C. Slow, shallow breathing D. Bradycardia

Sudden weight gain Weight gain is an early symptom of congestive heart failure due to accumulation of fluid. Important among these are renal retention of fluid, renin-angiotensin mediated vasoconstriction and sympathetic overactivity. Excessive fluid retention increases the cardiac output by increasing the end diastolic volume (preload), but also results in symptoms of pulmonary and systemic congestion. Option B: Vasoconstriction (increase in afterload) tends to maintain flow to vital organs, but it is disproportionately elevated in patients with CHF and increases myocardial work. Similarly, sympathetic overactivity results in increase in contractility, which also increases myocardial requirements. An understanding of the interplay of the four principal determinants of cardiac output - preload, afterload, contractility and heart rate is essential in optimising the therapy of CHF. It is clinically useful to consider CHF in different age groups separately. Option C: In the past, the most sensitive and specific variables for the presence of CHF (p<0.0001) were, a history of less than 3.5 oz/feed, respiratory rate greater than 50/min, an abnormal respiratory pattern, diastolic filling sounds, and hepatomegaly. Moderate to severe CHF was considered to be present when patients took less than 3 oz/feed or greater than 40 min/feed, had an abnormal respiratory pattern with a resting respiratory rate greater than 60/min, and had a diastolic filling sound and moderate hepatomegaly. Option D: Tachycardia >150/min is common, and heart rates >180/min are abnormal even in the setting of respiratory distress and suggests CHF. Severe CHF was accompanied by a heart rate greater than 170/min, decreased perfusion, and severe hepatomegaly. Thus, the grading of the severity of CHF in infants should include an accurate description of these historical and clinical variables.

Calculate MAP

Systolic + (2) Diastolic / 3

A patient at a mental health clinic is taking Haldol (haloperidol) for treatment of schizophrenia. She calls the clinic to report abnormal movements of her face and tongue. The nurse concludes that the patient is experiencing which of the following symptoms: A. Comorbid depression B. Psychotic hallucinations C. Negative symptoms of schizophrenia D. Tardive dyskinesia

Tardive dyskinesia Abnormal facial movements and tongue protrusion in a patient taking haloperidol is most likely due to tardive dyskinesia, an adverse reaction to the antipsychotic. Tardive dyskinesia is a syndrome that includes a group of iatrogenic movement disorders caused by the blockade of dopamine receptors. The movement disorders include akathisia, dystonia, buccolingual stereotypy, myoclonus, chorea, tics and other abnormal involuntary movements which are commonly caused by the long-term use of typical antipsychotics. Option A: Depression and anxiety is frequently seen in many schizophrenic patients and may be further aggravated or diminished by antipsychotic treatments. Haloperidol is a conventional antipsychotic used in schizophrenia and psychosis. Option B: Psychotic hallucinations may be visual or auditory but do not include abnormal movements. Hallucinations are most often associated with schizophrenia, a mental illness characterized by disordered thoughts and behaviors. Option C: Depression may occur along with schizophrenia and would be characterized by such symptoms as loss of affect, appetite and/or sleep changes, and anhedonia. These depressive changes and lack of volition are part of the negative symptoms of schizophrenia.

A nurse is caring for a patient who has had hip replacement. The nurse should be most concerned about which of the following findings? A. Complaints of pain during repositioning. B. Scant bloody discharge on the surgical dressing. C. Complaints of pain following physical therapy. D. Temperature of 101.8 F (38.7 C).

Temperature of 101.8 F (38.7 C). Post-surgical nursing assessment after hip replacement should be principally concerned with the risk of neurovascular complications and the development of infection. A temperature of 101.8 F (38.7 C) postoperatively is higher than the low grade that is to be expected and should raise concern. The THA postoperative wound complication spectrum ranges from superficial surgical infections (SSIs) such as cellulitis, superficial dehiscence, and/or delayed wound healing, to deep infections resulting in full-thickness necrosis. Deep infections result in returns to the operating room for irrigation, debridement (incision and drainage) and depending on the timing of the infection, may require explanation of THA components. Option A: Joint replacement surgery relieves the pain and stiffness of arthritis for most people. Some people may still have some symptoms of arthritis. For most people, surgery usually provides enough relief of symptoms for most people. Loosening of the new joint over time can cause pain, and sometimes another surgery is needed to fix the problem. Option B: A small amount of bloody drainage on the surgical dressing is a result of normal healing. normal to lose blood during and after hip or knee replacement surgery. Some people need a blood transfusion during surgery or during their recovery period in the hospital. Some surgeries require you to donate blood before surgery. Much of the bleeding during surgery comes from the bone that has been cut. A bruise may occur if blood collects around the new joint or under the skin after surgery. Option C: Some pain following physical therapy is to be expected and can be managed with analgesics. As in its counterpart TKA procedure, aseptic loosening is the result of a confluence of steps involving particulate debris formation, prosthesis micromotion, and macrophage-activated osteolysis. Treatment requires serial imaging and radiographs and/or CT imaging for preoperative planning. Persistent pain requires revision THA surgery.

The Joint Comission

The Joint Commission monitors and advocates for legislation that promotes better patient safety. When it comes to state legislation, The Joint Commission collaborates with patient safety authorities and state regulatory bodies to minimize unrealistic expectations and reform outdated rules. They push state regulatory bodies to rely more on private accreditation instead of mandatory state licensure inspections.

A nurse is performing a routine assessment of an IV site in a patient receiving both IV fluids and medications through the line. Which of the following would indicate the need for discontinuation of the IV line as the next nursing action? A. The patient complains of pain from movement. B. The area proximal to the insertion site is reddened, warm, and painful. C. The IV solution is infusing too slowly, particularly when the limb is elevated. D. A hematoma is visible in the area of the IV insertion site.

The area proximal to the insertion site is reddened, warm, and painful. An IV site that is red, warm, painful and swollen indicates that phlebitis has developed and the line should be discontinued and restarted at another site. Phlebitis is inflammation of a vein. It is usually associated with acidic or alkaline solutions or solutions that have a high osmolarity. Phlebitis can also occur as a result of vein trauma during insertion, use of an inappropriate I.V. catheter size for the vein, or prolonged use of the same I.V. site. Option A: Pain on movement should be managed by maneuvers such as splinting the limb with an IV board or gently shifting the position of the catheter before making a decision to remove the line. Apply warm, moist compresses to the area. Document the patient's condition and interventions. Option C: An IV line that is running slowly may simply need flushing or repositioning. Monitor administration rates and inspect the I.V. site frequently. Change the infusion site according to the facility's policy. Option D: A hematoma at the site is likely a result of minor bleeding at the time of insertion and does not require discontinuation of the line. Avoid veins that are small and/or fragile, veins in areas of flexion, veins in extremities with preexisting edema, or veins in areas with known neurologic impairment.

The client is having electroconvulsive therapy for treatment of severe depression. Which of the following indicates that the client's ECT has been effective? A. The client loses consciousness. B. The client vomits. C. The client's ECG indicates tachycardia. D. The client has a grand mal seizure.

The client has a grand mal seizure. During ECT, the client will have a grand mal seizure. This indicates completion of electroconvulsive therapy. Seizure threshold is established via trial and error via incrementally higher doses of current during the primary treatment session. Following initial dose calculation, the dose at subsequent ECT sessions for bilateral ECT is 1.5 to 2 times seizure threshold, and for right unilateral is six times the seizure threshold. During the course of ECT treatment, the seizure threshold commonly increases as the patient develops tolerance. Option A: Once the patient is rendered unconscious, administration of a muscle relaxant follows, along with bag valve mask ventilation with 100 percent oxygen. A nerve stimulator is utilized to determine the adequacy of muscle relaxation along with the clinical assessment of plantar reflexes and fasciculations in the calves and left foot. Option B: Physiologically, during the tonic phase of the seizure, a 15- to 20-second parasympathetic discharge occurs, which can lead to bradyarrhythmias including premature atrial and ventricular contractions, atrioventricular block, and asystole. Patients with sub convulsive seizures are at higher risk for asystole. Option C: Paradoxically, patients with heart block or underlying arrhythmias are less likely to develop asystole. The clonic phase of the seizure correlates with a catecholamine surge that causes tachycardia and hypertension, which lasts temporally with seizure duration. Hypertension and tachycardia resolve within 10 to 20 minutes of the seizure, although some patients exhibit persistent hypertension that requires medical intervention.

The nurse is planning room assignments for the day. Which client should be assigned to a private room if only one is available? A. The client with Cushing's disease B. The client with diabetes C. The client with acromegaly D. The client with myxedema

The client with Cushing's disease The client with Cushing's disease has adrenocortical hypersecretion. This increase in the level of cortisone causes the client to be immunosuppressed. High cortisol levels also cause immune disruptions; this hormone leads to a decrease in lymphocyte levels and increases the neutrophils. It causes detachment of the marginating pool of neutrophils in the bloodstream and increases the circulating neutrophil levels although there is no increased production of the neutrophils. Option B: The client with diabetes poses no risk to other clients. Hyperglycemia alone can impair pancreatic beta-cell function and contributes to impaired insulin secretion. Consequently, there is a vicious cycle of hyperglycemia leading to the impaired metabolic state. Blood glucose levels above 180 mg/dL are often considered hyperglycemic in this context, though because of the variety of mechanisms, there is no clear cutoff point. Option C: The client has an increase in growth hormone and poses no risk to himself or others. The common effect of the abnormal rise in growth hormone is the production of IGF-1 from the liver. The effect of IGF-1 on body tissues results in the multisystemic manifestation of acromegaly. IGF-1 also known as somatomedin C, is encoded by the IGF-1 gene on chromosome 12q23.2. Option D: The client has hypothyroidism or myxedema and poses no risk to others or himself. Thyroid hormone influences virtually all cells in the body by activating or repressing a variety of genes after binding to thyroid hormone receptors. Ninety percent of the intracellular thyroid hormone that binds to and influences cellular function is T3, which has been converted from T4 by the removal of an iodide ion

A child weighing 30 kg arrives at the clinic with diffuse itching as the result of an allergic reaction to an insect bite. diphenhydramine (Benadryl) 25 mg 3 times a day is prescribed. The correct pediatric dose is 5 mg/kg/day. Which of the following best describes the prescribed drug dose? A. It is the correct dose. B. The dose is too low. C. The dose is too high. D. The dose should be increased or decreased, depending on the symptoms.

The dose is too low. This child weighs 30 kg, and the pediatric dose of diphenhydramine is 5 mg/kg/day (5 X 30 = 150/day). Therefore, the correct dose is 150 mg/day. Diphenhydramine, which is available as an over-the-counter medication, is a first-generation antihistamine that is used in a variety of conditions to treat and prevent dystonias, insomnia, pruritus, urticaria, vertigo, and motion sickness. Option A: The correct dose is 150 mg/day. Diphenhydramine acts as an inverse agonist at the H1 receptor, thereby reversing the effects of histamine on capillaries, reducing allergic reaction symptoms. Given that diphenhydramine is a first-generation antihistamine, it readily crosses the blood-brain barrier and inversely agonizes the H1 CNS receptors, resulting in drowsiness, and suppressing the medullary cough center. Option C: Divided into 3 doses per day, the child should receive 50 mg 3 times a day rather than 25 mg 3 times a day. Diphenhydramine overdose can cause significant toxicity, ranging from agitation to cardiac arrhythmias to rhabdomyolysis and classic anticholinergic toxidrome. Further studies are needed to investigate the potential treatment of diphenhydramine toxicity with the use of sodium bicarbonate and intravenous lipid emulsion therapy. Option D: Dosage should not be titrated based on symptoms without consulting a physician. Diphenhydramine can be given by tablet, capsule, or in solution by mouth; by intramuscular (IM) or intravenous (IV) injection; or topically. Diphenhydramine is now available over the counter and it is important for the pharmacist and nurse practitioner to educate the patient on the safe use of this agent.

A nurse is giving discharge instructions to the parents of a healthy newborn. Which of the following instructions should the nurse provide regarding car safety and the trip home from the hospital? A. The infant should be restrained in an infant car seat, properly secured in the back seat in a rear-facing position. B. The infant should be restrained in an infant car seat, properly secured in the front passenger seat. C. The infant should be restrained in an infant car seat facing forward or rearward in the back seat. D. For the trip home from the hospital, the parent may sit in the back seat and hold the newborn.

The infant should be restrained in an infant car seat, properly secured in the back seat in a rear-facing position. All infants under 1 year of age weighing less than 20 lbs. should be placed in a rear-facing infant car seat secured properly in the back seat. Rear-facing car safety seats for infants are perhaps the least controversial; rear-facing car seats have superior effectiveness in preventing serious injury in infants from car crashes. Children < 24 months riding in rear-facing car seats were 1.76 times less likely be seriously injured from all types of car crashes relative to children riding in forward-facing safety seats Option B: Infant car seats should never be placed in the front passenger seat. States have implemented their own individual mandates for car safety seats, with stringent recommendations from the AAP incorporated into law in New Jersey and Oklahoma and with states including West Virginia (WV) and North Carolina having the most lenient requirements (Bae, Anderson, Silver, & Macinko, 2014; NCPSC, 2013). WV requires that children under 7 years be restrained in a car safety or booster seat, without specifying the timing of the transition. Option C: The infant should always face rearward in the back seat while on a car seat. For side crashes, children < 24 months riding in forward-facing car seats were 5.5 times more likely to get injured as compared to those riding in rear-facing car seats. Accident data (such as from Sweden) indicate that increased duration of rear-facing car safety seat usage can decrease injuries and deaths relating to automobile accidents (SafetyBeltSafe USA, 2013). Option D: Infants should always be placed in an approved car seat during travel, even on that first ride home from the hospital. Consistent with research, the American Academy of Pediatrics (AAP) and National Highway Traffic Safety Administration (NHTSA) have developed evidence-based practice guidelines for car safety seat use, which vary by the size and weight of the child, emphasizing the use of rear seats among infants less than 2 years of age (AAP, 2013; NHTSA, 2014).

Which of the following arteries primarily feeds the anterior wall of the heart? Circumflex artery Internal mammary artery Left anterior descending artery Right coronary artery

The left anterior descending artery is the primary source of blood for the anterior wall of the heart. The left anterior descending artery branches off the left coronary artery and supplies blood to the front of the left side of the heart. A) The circumflex artery supplies the lateral wall. The circumflex artery, fully titled as the circumflex branch of the left coronary artery, is an artery that branches off from the left coronary artery to supply portions of the heart with oxygenated blood. The circumflex artery itself divides into smaller arterial systems. B) The internal mammary artery supplies the mammary. The internal thoracic artery (ITA), previously commonly known as the internal mammary artery (a name still common among surgeons), is an artery that supplies the anterior chest wall and the breasts. D) The right coronary artery supplies the inferior wall of the heart. The right coronary artery supplies blood to the right ventricle and then supplies the underside (inferior wall) and backside (posterior wall) of the left ventricle.

The nurse is conducting nutrition counseling for a patient with cholecystitis. Which of the following information is important to communicate? A. The patient must maintain a low calorie diet. B. The patient must maintain a high protein/low carbohydrate diet. C. The patient should limit sweets and sugary drinks. D. The patient should limit fatty foods.

The patient should limit fatty foods. Cholecystitis, inflammation of the gallbladder, is most commonly caused by the presence of gallstones, which may block bile (necessary for fat absorption) from entering the intestines. Patients should decrease dietary fat by limiting foods like fatty meats, fried foods, and creamy desserts to avoid irritation of the gallbladder. Option A: People who go on an extremely low-calorie diet are more likely to develop gallstones than people on a moderately low-calorie diet. Eating a healthy, well-balanced diet full of fruits and vegetables is the best way to improve and protect the gallbladder's health. Fruits and vegetables are full of nutrients and fiber, the latter of which is essential to a healthy gallbladder. Option B: A low-fat diet with lean proteins is recommended for patients with cholecystitis. Foods with trans fats, like those in processed or commercially baked products, can also be harmful to gallbladder health. Option C: Moderate consumption of sweet drinks can be allowed. Avoiding refined white foods, like white pasta, bread, and sugar, can protect the gallbladder. Eat whole-grain cereals, whole-grain bread, whole-grain crackers, brown rice, or whole-grain pasta. Avoid high-fat foods such as croissants, scones, biscuits, waffles, doughnuts, muffins, granola, and high-fat bread.

A clinic patient has a hemoglobin concentration of 10.8 g/dL and reports sticking to a strict vegetarian diet. Which of the following nutritional advice is appropriate? A. The diet is providing adequate sources of iron and requires no changes. B. The patient should add meat to her diet; a vegetarian diet is not advised. C. The patient should use iron cookware to prepare foods, such as dark green, leafy vegetables and legumes, which are high in iron. D. A cup of coffee or tea should be added to every meal.

The patient should use iron cookware to prepare foods, such as dark green, leafy vegetables and legumes, which are high in iron. Normal hemoglobin values range from 11.5-15.0. This vegetarian patient is mildly anemic. When food is prepared in iron cookware its iron content is increased. Anemia is defined as hemoglobin below two standard deviations of the mean for the age and gender of the patient. Iron is an essential component of the hemoglobin molecule. The most common cause of anemia worldwide is iron deficiency, which results in microcytic and hypochromic red cells on the peripheral smear. Option A: The client is mildly anemic. The cause of iron-deficiency anemia varies based on age, gender, and socioeconomic status. Iron deficiency may result from insufficient iron intake, decreased absorption, or blood loss. Iron-deficiency anemia is most often from blood loss, especially in older patients. Option B: Mild anemia does not require that animal sources of iron be added to the diet. Many non-animal sources are available. Dietary sources of iron are green vegetables, red meat, and iron-fortified milk formulas. It may also be seen with low dietary intake, increased systemic requirements for iron such as in pregnancy, and decreased iron absorption such as in celiac disease. Option D: Coffee and tea increase gastrointestinal activity and inhibit absorption of iron. The iron in food comes from two sources: animals and plants. Iron from animal sources is known as heme iron and is found in meat and fish. Iron from plants is known as nonheme iron, and is found in certain vegetables and in iron-fortified foods such as breakfast cereals. Heme iron is better absorbed by the body than non heme iron.

A patient comes to the emergency department with abdominal pain. Work-up reveals the presence of a rapidly enlarging abdominal aortic aneurysm. Which of the following actions should the nurse expect? A. The patient will be admitted to the medicine unit for observation and medication. B. The patient will be admitted to the day surgery unit for sclerotherapy. C. The patient will be admitted to the surgical unit and resection will be scheduled. D. The patient will be discharged home to follow-up with his cardiologist in 24 hours.

The patient will be admitted to the surgical unit and resection will be scheduled. A rapidly enlarging abdominal aortic aneurysm is at significant risk of rupture and should be resected as soon as possible. Abdominal aortic aneurysm (AAA) is a life-threatening condition which requires monitoring or treatment depending upon the size of the aneurysm and/or symptomatology. AAA may be detected incidentally or at the time of rupture. An arterial aneurysm is defined as a permanent localized dilatation of the vessel at least 150% compared to a relative normal adjacent diameter of that artery Option A: The patient should be admitted but not in the medicine unit. Rupture of an abdominal aortic aneurysm is life-threatening. These patients may present in shock often with diffuse abdominal pain and distension. However, the presentation of patients with this type of ruptured aneurysm can vary from subtle to quite dramatic. Most patients with a ruptured abdominal aortic aneurysm die before hospital arrival. Option B: The patient should undergo resection instead of sclerotherapy. Open surgical repair via transabdominal or retroperitoneal approach has been the gold standard. Endovascular repair from a femoral arterial approach is now applied for a majority of repairs, especially in older and higher risk patients. Endovascular therapy is recommended in patients who are not candidates for open surgery. This includes patients with severe heart disease, and/or other comorbidities that preclude open repair. Option D: The patient should not be discharged because the aneurysm may rupture. A ruptured abdominal aortic aneurysm warrants emergency repair. Endovascular approach for ruptured AAA has demonstrated superior results and survival compared to open repair if the anatomy is suitable, but the mortality rates remain high. The risk of surgery is influenced by the age of the patient, the presence of renal failure, and the status of the cardiopulmonary system.

Gullaine- Barre Syndrome

The priority interventions for the patient with GBS are aimed at maintaining adequate respiratory function. These patients are at risk for respiratory failure, which is urgent. Upon presentation, 40% of patients have a respiratory or oropharyngeal weakness. Ventilatory failure with required respiratory support occurs in up to one-third of patients at some time during the course of their disease. The typical patient with GBS will manifest acute inflammatory demyelinating polyradiculoneuropathy (AIDP), presents 2-4 weeks following a relatively benign respiratory or gastrointestinal illness with complaints of finger dysesthesias and proximal muscle weakness of the lower extremities. Facial weakness and difficulty of speaking are common signs of GBS and must be reported, but it is not a priority. The classic clinical picture of weakness is ascending and symmetrical in nature. The lower limbs are usually involved before the upper limbs. Proximal muscles may be involved earlier than the more distal ones. Trunk, bulbar, and respiratory muscles can be affected as well. A rapid heart rate is important and should be reported to the nurse, but it is not life-threatening. Autonomic nervous system involvement with dysfunction in the sympathetic and parasympathetic systems can be observed in patients with GBS.

A patient on the cardiac telemetry unit unexpectedly goes into ventricular fibrillation. The advanced cardiac life support team prepares to defibrillate. Which of the following choices indicates the correct placement of the conductive gel pads? A. The left clavicle and right lower sternum. B. Right of midline below the bottom rib and the left shoulder. C. The upper and lower halves of the sternum. D. The right side of the sternum just below the clavicle and left of the precordium.

The right side of the sternum just below the clavicle and left of the precordium. One gel pad should be placed to the right of the sternum, just below the clavicle and the other just left of the precordium, as indicated by the anatomic location of the heart. To defibrillate, the paddles are placed over the pads. According to the ILCOR guidelines, the sternal paddle should be placed 'just to the right of the upper sternal border below the clavicle' and the apical paddle 'to the left of the nipple with the centre of the electrode in the mid-axillary line'. Option A: During the gel pad placement study it was noticed that about 50% of doctors placed the rectangular apical paddle vertically upwards, pointing towards the left armpit. The other 50% placed it in a horizontal position across the chest. The present ILCOR guidelines do not specify which orientation should be used for defibrillation. It was hypothesized that, with the paddle method for defibrillation, it would be more difficult to get good skin contact across the curved chest wall with the horizontal orientation, and in a small study this proved to be the case. Option B: In theory, a paddle position that is too superomedial means that less current will traverse the myocardium. When 60 N (the median force used by defibrillator operators in clinical practice) is applied to both paddles, the resulting TTI is 5% greater with the horizontal orientation. Thus, if paddles are used, it is recommended to use a vertical orientation. It is expected that their flexibility will allow better electrode/skin contact across the curved chest wall; however, in the absence of any evidence to the contrary, it is advised to use vertical orientation for this method as well. Option C: Most healthcare workers are not achieving optimal TTI during defibrillation. There is now good evidence that the use of a coupling agent, chest hair removal, placement of the apical paddle in a vertical orientation lateral to the nipple in the mid-axillary line, and application of at least 80 N of force are all measures that help minimize the TTI.

A child is admitted to the hospital with a diagnosis of Wilms tumor, stage II. Which of the following statements most accurately describes this stage? A. The tumor is less than 3 cm. in size and requires no chemotherapy. B. The tumor did not extend beyond the kidney and was completely resected. C. The tumor extended beyond the kidney but was completely resected. D. The tumor has spread into the abdominal cavity and cannot be resected.

The tumor extended beyond the kidney but was completely resected. Stage II, the tumor extends beyond the kidney but is completely resected. Stage II would be a tumor that has grown outside the kidney to some degree, such as into surrounding fatty tissue. Usually, the tumor would be completely removable by surgery, and regional lymph nodes are negative. About 20% of all Wilms tumors are at this stage. Option A: This is not included in the staging of Wilms tumor. Imaging is particularly important in surgical planning. Surgical risk factors include larger tumor size, contralateral tumor extension, and displacement of the great vessels which typically result in longer surgical times, increased blood loss, and higher complication rates. Option B: This described stage I: the tumor is limited to the kidney and completely resected. Stage I indicates the tumor was completely contained within the kidney without any breaks or spillage outside the renal capsule and no vascular invasion. This stage accounts for 40% to 45% of all Wilms tumors. Option D: This described stage IV, hematogenous metastasis has occurred with spread beyond the abdomen. Stage IV tumors are those that have spread through the vascular system to distant organs such as the lungs, liver, brain, or bones, or to distant lymph nodes. These account for about 10% of all Wilms tumors.

As the client reaches 8 cm dilation, the nurse notes late decelerations on the fetal monitor. The FHR baseline is 165-175 bpm with variability of 0-2bpm. What is the most likely explanation of this pattern? A. The baby is asleep. B. The umbilical cord is compressed. C. There is a vagal response. D. There is uteroplacental insufficiency.

There is uteroplacental insufficiency. This information indicates a late deceleration. This type of deceleration is caused by uteroplacental lack of oxygen. Late decelerations are one of the precarious decelerations among the three types of fetal heart rate decelerations during labor. They are caused by decreased blood flow to the placenta and can signify an impending fetal acidemia. Option A: Has no relation to the readings. The primary etiology of a late declaration is found to be uteroplacental insufficiency. Decreased blood flow to the placenta causes a reduced amount of blood and oxygen to the fetus. Option B: Compressed umbilical cord results in a variable deceleration. The central pathophysiology behind late deceleration involves uterine contraction constricting blood vessels in the wall of the uterus which decreases blood flow through the intervillous space of the placenta, reducing diffusion of oxygen into fetal capillaries causing decreased fetal PO2. Option C: A vagal response is indicative of an early deceleration. When fetal PO2 decreases, chemoreceptors initiate an autonomic response in the fetus causing intense vasoconstriction with increased blood pressure. The elevated blood pressure is perceived by the baroreceptors which ultimately stimulate the parasympathetic system to decrease the fetal heart rate, causing late deceleration.

A client with cystic fibrosis is taking pancreatic enzymes. The nurse should administer medication: A. Once per day in the morning B. Three times per day with meals C. Once per day at bedtime D. Four times per day

Three times per day with meals. Pancreatic enzymes should be given with meals for optimal effects. These enzymes assist the body and digesting needed nutrients. Chronic, supportive therapy for patients with CF includes regular pancreatic enzymes, fat soluble vitamins (A, D, E, K), Mucolytics, bronchodilators, antibiotic's, and anti-inflammatory agents. Option a: A new class of medications known as CFTR modulator therapies is designed to correct the dysfunction by improving production, intercellular processing, or function of the CFTR protein caused by the mutated gene. Each medication is targeted at a specific dysfunction caused by a specific gene mutation. Option C: individuals with CF are encouraged to consume a high fat diet with supplemental fat soluble vitamins to compensate for malabsorption. Additionally, patients living with CF are encouraged to consume a high calorie diet to maintain a healthy weight and combat chronic inflammation and frequent infections that are commonly encountered. Option D: according to the cystic fibrosis foundation, women should consume 2500 to 3000 cal a day, woman should consume 3000 to 3700 cal a day. Those living in hot climates or who participate in activities that cause sweating are encouraged to consume additional sodium in their diet.

A 64-year-old male client with a long history of cardiovascular problems including hypertension and angina is to be scheduled for cardiac catheterization. During pre-cardiac catheterization teaching, Nurse Cherry should inform the client that the primary purpose of the procedure is: A. To determine the existence of CHD. B. To visualize the disease process in the coronary arteries. C. To obtain the heart chambers pressure. D. To measure oxygen content of different heart chambers.

To visualize the disease process in the coronary arteries. The lumen of the arteries can be assessed by cardiac catheterization. Angina is usually caused by narrowing of the coronary arteries. Left heart catheterization has a diagnostic as well as therapeutic role. Although it is used for cardiac hemodynamics and assessment of valvular lesions, its main diagnostic role is the assessment of coronary artery disease. In the contemporary era, left heart catheterization, especially selective coronary angiogram, is considered the gold standard test for coronary artery disease diagnosis. Option A: Determining the existence of coronary heart disease includes tests and procedures such as echocardiogram, coronary angiography, blood tests, ECG, and MRI scans. Echocardiography is an ultrasound of the heart. It is a useful and non-invasive mode of testing that is performed in both acute and chronic and inpatient and outpatient settings. In acute settings, it could tell about wall motion, valvular regurgitation and stenosis, infective or autoimmune lesions, and chamber sizes. Option C: Heart chamber pressure can be checked through cardiac catheterization but it is not the primary purpose. In non ACS settings, patients with intermediate pretest probability for CAD are usually the right candidates for it. In the ACS setting, all STEMI patients and selected NSTEMI patients get an emergent cardiac catheterization. This procedure is done in a cardiac catheterization lab, is expertise dependent, and is done under moderate sedation. There is contrast exposure in the procedure which could cause serious allergic reactions and kidney injury. Option D: The oxygen saturation of the blood can be checked through a pulse oximeter. A pulse oximeter can measure oxygen saturation. It is a noninvasive device placed over a person's finger. It measures light wavelengths to determine the ratio of the current levels of oxygenated hemoglobin to deoxygenated hemoglobin. The use of pulse oximetry has become a standard of care in medicine. It is often regarded as a fifth vital sign.

An infant is brought to the clinic by his mother, who has noticed that he holds his head in an unusual position and always faces to one side. Which of the following is the most likely explanation? A. Torticollis, with shortening of the sternocleidomastoid muscle. B. Craniosynostosis, with premature closure of the cranial sutures. C. Plagiocephaly, with flattening of one side of the head. D. Hydrocephalus, with increased head size.

Torticollis, with shortening of the sternocleidomastoid muscle. In torticollis, the sternocleidomastoid muscle is contracted, limiting range of motion of the neck and causing the chin to point to the opposing side. Torticollis or twisted neck (tortum collum) of Italian origin "torti colli" is a vicious attitude of the head and neck, typically presenting with abnormal slope and rotation. There may be several presenting positions, including flexion, extension, right or left tilt. These have names such as horizontal torticollis, vertical, oblique, or torsion. Option B: In craniosynostosis one of the cranial sutures, often the sagittal, closes prematurely, causing the head to grow in an abnormal shape. Craniosynostosis is the result of the early fusion of cranial sutures. These sutures exist to facilitate the passage of the baby through the birth canal and later on allow the expansion and growth of the brain. When one or more sutures close prematurely, the structure of the skull becomes altered, growing on the path of least resistance (perpendicularly to the closed suture) and resulting in an atypically shaped skull leading to increased intracranial pressure (ICP) and having an effect on the respiratory and neurologic systems, as well as the development of the child. Option C: Plagiocephaly refers to the flattening of one side of the head, caused by the infant being placed supine in the same position over time. Plagiocephaly is defined as an asymmetric shape of the head due to unilateral flattening. The term flathead can be used as a synonym. Plagiocephaly can be categorized into synostotic or deformational types. Option D: Hydrocephalus is caused by a build-up of cerebrospinal fluid in the brain resulting in large head size. Hydrocephalus is the symptomatic accumulation of cerebrospinal fluid (CSF) inside the cerebral ventricles. This accumulation may be due to obstruction in the normal flow of the CSF, or to problems with absorption into the venous system by the Pacchionian arachnoid granulations, or due to excessive production of CSF.

A 68-year-old client is diagnosed with a right-sided brain attack and is admitted to the hospital. In caring for this client, the nurse should plan to: A. Application of elastic stockings to prevent flaccid by muscle B. Use hand roll and extend the left upper extremity on a pillow to prevent contractions C. Use a bed cradle to prevent dorsiflexion of feet D. Do passive range of motion exercise

Use hand roll and extend the left upper extremity on a pillow to prevent contractions The left side of the body will be affected in a right-sided brain attack. Begin active or passive ROM to all extremities (including splinted) on admission. Encourage exercises such as quadriceps/glutes exercise, squeezing rubber ball, extension of fingers and legs/feet. Minimizes muscle atrophy, promotes circulation, helps prevent contractures. Reduces risk of hypercalciuria and osteoporosis if the underlying problem is hemorrhage. Option A: The left upper extremity will be affected, application of elastic stockings is unnecessary. Assist the patient with exercise and perform ROM exercises for both the affected and unaffected sides. Teach and encourage the patient to use his unaffected side to exercise his affected side. ROM exercise helps in reducing muscle stiffness and spasticity. It can also help prevent contractures. Option C: There is no dorsiflexion of foot in a right-sided brain attack. Prop extremities in functional position; use footboard during the period of flaccid paralysis. Maintain a neutral position of the head. Prevents contractures and foot drop and facilitates use when function returns. Flaccid paralysis may interfere with ability to support the head, whereas spastic paralysis may lead to deviation of the head to one side. Option D: Active-assistive range of motion exercises would be recommended for the client. Encourage the patient to assist with movement and exercises using unaffected extremity to support and move the weaker side. May respond as if the affected side is no longer part of the body and needs encouragement and active training to "reincorporate" it as a part of own body.

Which of the following complication is indicated by a third heart sound (S3)? Ventricular dilation Systemic Hypertension Aortic Valve Malfunction Increased Atrial Contractions

Ventricular Dilation Rapid filling of the ventricles causes vasodilation that is auscultated as S3. The third heart sound is low-frequency, brief vibration occurring in early diastole at the end of the rapid diastolic filling period of the right or left ventricle. Systemic hypertension can result in a fourth heart sound. The fourth heart sound is a low-pitched sound coincident with late diastolic filling of the ventricle due to atrial contraction. It thus occurs shortly before the first heart sound. Although it is also called the atrial sound, and its production requires an effective atrial contraction, the fourth heart sound is the result of vibrations generated within the ventricle. Aortic Valve Malfunction is heard as a murmur. One in three elderly people have a heart murmur because of the scarring, thickening, or stiffening of their aortic valve. That's aortic sclerosis. It's usually not dangerous, since the valve can work for years after the murmur starts. Increased atrial contractions can cause a fourth heart sound. Common theoretic mechanisms for this condition are based around abnormal automaticity of the cardiac myocyte, increased eliciting events from chemical or physical instigators, and reentry of a retrograde impulse. For these causes, structural heart damage or chemical use may be found during the history and physical examination.

A client who is admitted with an above-the-knee amputation tells the nurse that his foot hurts and itches. Which response by the nurse indicates an understanding of phantom limb pain? A. "The pain will go away in a few days." B. "The pain is due to peripheral nervous system interruptions. I will get you some pain medication." C. "The pain is psychological because your foot is no longer there." D. "The pain and itching are due to the infection you had before the surgery."

"The pain is due to peripheral nervous system interruptions. I will get you some pain medication." Pain-related to phantom limb syndrome is due to a peripheral nervous system interruption. A recent study estimated that there were about 1.6 million people with limb loss in the USA in 2005 and this number was projected to increase by more than double to 3.6 million by the year 2050. Vascular problems, trauma, cancer, and congenital limb deficiency are among the common causes of limb loss. Option A: Phantom limb pain can last several months or indefinitely. The phantom pain and sensation may have its onset immediately or years after the amputation. There are reports of two peak periods of onset, the first within a month and the second a year after amputation. The prevalence is reported to decrease over time after amputation. Option C: The explanation of phantom limb pain is not psychologically related. PLP was once thought to be primarily a psychiatric illness. With the accumulation of evidence from research over the past decades, the paradigm has shifted more towards changes at several levels of the neural axis, especially the cortex. Peripheral mechanisms and central neural mechanisms are among the hypotheses that have gained consensus as proposed mechanisms over the recent years. Option D: Pain and itching are not symptoms of an infection due to surgery. During amputation, peripheral nerves are severed. This results in massive tissue and neuronal injury-causing disruption of the normal pattern of afferent nerve input to the spinal cord. This is followed by a process called deafferentation and the proximal portion of the severed nerve sprouts to form neuromas. There is an increased accumulation of molecules enhancing the expression of sodium channels in these neuromas that results in hype-excitability and spontaneous discharges. This abnormal peripheral activity is thought to be a potential source of stump pain, including phantom pain.

Molly Sue is diagnosed with acute lymphoid leukemia (ALL) and beginning chemotherapy. Her mother states to the nurse that it is hard to see Molly Sue with no hair. The best response for the nurse is: A. "Molly Sue looks very nice wearing a hat". B. "You should not worry about her hair, just be glad that she is alive". C. "Yes, it is upsetting. But try to cover up your feelings when you are with her or else she may be upset". D. "This is only temporary; Molly Sue will re-grow new hair in 3-6 months but may be different in texture".

"This is only temporary; Stacy will re-grow new hair in 3-6 months but may be different in texture". This is the appropriate response. The nurse should help the mother how to cope with her own feelings regarding the child's disease so as not to affect the child negatively. When the hair grows back, it is still of the same color and texture. Option A: It can be an option for the client to use a hat, but the nurse should be open about explanations regarding the side effects of chemotherapy. Option B: Avoid reassuring the client. Statements of fact would be a good response. Option C: The mother should be open and honest with the child, and providing an honest and true response would be a big help to both of them.

During a routine visit to the clinic, a client tells the nurse that they feel lightheaded after taking nifedipine for hypertension. Which question is MOST important for the nurse to ask the client? "How much potassium are you taking?" "Do you take a nap every day?" "When do you take this medication?" "Are you drinking grapefruit juice?"

"When do you take this medication?" The nurse should ask the client when they take the medication. Nifedipine (Procardia, Adalat) is a calcium channel blocker. One of the common side effects is lightheadedness or dizziness. Other side effects include headache, fatigue, and shortness of breath. Calcium channel blockers may be taken at any time during the day, as long as the schedule is consistent. Taking it in the evening may minimize side effects. Grapefruit and grapefruit juice should not be taken with calcium channel blockers, but this is not related to lightheadedness. Potassium intake does not cause lightheadedness. Naps or rest periods are not related to dizziness.

The health care provider (HCP) prescribes "250 mg/5 mL amoxicillin PO q8h for 10 days" for a child's ear infection. The child's parent asks the nurse how much amoxicillin to give the child in each dose. Which of the following is the equivalent of 5 mL? 1 teaspoon 1/2 teaspoon 1/2 tablespoon 1 tablespoon

1 Teaspoon 5 mL = 1 teaspoon. The parent should give the child 1 teaspoon of amoxicillin every 8 hours. The parent should use a medication spoon or dropper to measure the dose, not a household spoon. The nurse should educate the parent about completing the prescription even if the child starts to feel better. 1 tablespoon = 3 teaspoons (15 mL), which would be a triple dose.

Nurse Patricia is aware that the average length of time from human immunodeficiency virus (HIV) infection to the development of acquired immunodeficiency syndrome (AIDS)? A. Less than 5 years B. 5 to 7 years C. 10 years D. More than 10 years

10 years Epidemiologic studies show the average time from initial contact with HIV to the development of AIDS is 10 years. The interval from HIV infection to the diagnosis of AIDS ranges from about 9 months to 20 years or longer, with a median of 12 years. Option A: Less than 5 years is too short a time for the development of AIDS. Option B: 5 to 7 years is not the average time when an HIV infection develops into AIDS. Option D: More than 10 years is more than the average time for HIV to develop into AIDS.

At 6:00 a.m., the healthcare provider administers NPH insulin to a patient with diabetes. How soon may the patient show any signs of hypoglycemia? 9:00 a.m. 7:00 a.m. 8:00 a.m. 10:00 a.m.

10:00 a.m. NPH insulin is an intermediate-acting insulin, usually given once or twice a day. The peak effect of NPH insulin occurs 4-12 hours after administration, so the nurse should start to monitor for signs of hypoglycemia at 10:00 a.m. Hypoglycemia (blood glucose below 70 mg/dl) can have a rapid onset. Signs include shakiness, dizziness, anxiety, confusion, sweating, chills, and clammy skin. The patient's pulse may increase. The patient may complain of blurred vision, headache, fatigue, hunger, or nausea.

Which lab value indicates hypokalemia? 1. 5.6 mEq/L 2. 3.2 mEq/L 3. 4.2 mEq/L 4. 3.6 mEq/L

2. 3.2 - CORRECT The normal serum potassium (K) range is 3.5-5.0 mEq/L. For this question, the only correct option is 3.2 mEq/L. Hypokalemia is associated with an increased risk of cardiac arrhythmias. The nurse should report this result immediately and be prepared to administer a potassium supplement.

A client comes to the clinic, complaining of severe gastrointestinal distress. Which abdominal physical assessment step does the nurse do first? 1. Percussion 2. Inspection 3. Auscultation 4. Palpation

2. Inspection- CORRECT The correct sequence for physical assessment of the abdomen is as follows: 1. Inspect. 2. Auscultate. 3. Percuss. 4. Palpate. Remember this sequence with the phrase "I Am a People Person." The order is different from the physical assessment of the body systems, for which you inspect, then palpate, percuss, and auscultate.

A physician has diagnosed acute gastritis in a clinic patient. Which of the following medications would be contraindicated for this patient? A. naproxen sodium (Naprosyn) B. calcium carbonate (Tums) C. clarithromycin (Biaxin) D. furosemide (Lasix)

naproxen sodium (Naprosyn) Naproxen sodium is a nonsteroidal anti-inflammatory drug that can cause inflammation of the upper GI tract. For this reason, it is contraindicated in a patient with gastritis. COX-1 and COX-2 inhibition lead to decreased prostaglandin synthesis in the gastric mucosa. The prostaglandins maintain mucosal integrity, therefore decreased synthesis causes reduced protection to the tissue. However, studies indicate COX-1 has a more significant effect on the integrity of the mucosa; consequently, selective COX-2 inhibitors such as Celecoxib do not have as much of an effect on gastric tissue. Option B: Calcium carbonate is used as an antacid for the relief of indigestion and is not contraindicated. Calcium carbonate is an inorganic salt primarily used in the management and treatment of low calcium conditions, GERD, CKD, and a variety of other indicated conditions. It is classified as a calcium supplement, antacid, and as a phosphate binder. Option C: Clarithromycin is an antibacterial often used for the treatment of Helicobacter pylori in gastritis. Clarithromycin is in a class of medications called macrolide antibiotics. It works by stopping the growth of bacteria. Clarithromycin is used to treat certain bacterial infections, such as pneumonia (a lung infection), bronchitis (infection of the tubes leading to the lungs), and infections of the ears, sinuses, skin, and throat. Option D: Furosemide is a loop diuretic and is contraindicated in a patient with gastritis. The Food and Drug Administration (FDA) has approved the use of furosemide in the treatment of conditions with volume overload and edema secondary to congestive heart failure exacerbation, liver failure, or renal failure including nephrotic syndrome.

A nurse cares for a patient who has a nasogastric tube attached to low suction because of a suspected bowel obstruction. Which of the following arterial blood gas results might be expected in this patient? A. pH 7.52, PCO2 54 mmHg. B. pH 7.42, PCO2 40 mmHg. C. pH 7.25, PCO2 25 mmHg. D. pH 7.38, PCO2 36 mmHg.

pH 7.52, PCO2 54 mmHg. A patient on nasogastric suction is at risk of metabolic alkalosis as a result of loss of hydrochloric acid in gastric fluid. Of the answers given, only answer A (pH 7.52, PCO2 54 mm Hg) represents alkalosis. Normal range for pH range from 7.35-7.45. CO2 level has a normal range of 35 to 45 mmHg. Normal range for HCO3 is 22-26 mmol/L. The lower the number, the more acidotic the patient is. The higher the pH, the more base is in the blood sample. Option B: This result is a normal blood gas value. Arterial blood gas analysis assesses a patient's partial pressure of oxygen (PaO2), providing information on the oxygenation status; the partial pressure of carbon dioxide (PaCO2), providing information on the ventilation status (chronic or acute respiratory failure, and is changed by hyperventilation (rapid or deep breathing) and hypoventilation (slow or shallow breathing); and acid-base status. Option C: This result represents respiratory acidosis. Arterial blood gas interpretation is best approached systematically. Interpretation leads to an understanding of the degree or severity of abnormalities, whether the abnormalities are acute or chronic, and if the primary disorder is metabolic or respiratory in origin. Several articles have described simplistic ways to interpret ABG results. However, the Romanski method of analysis is most simplistic for all levels of providers. This method assists with determining the presence of an acid-base disorder, its primary cause, and whether compensation is present. Option D: This result is borderline normal with slightly low PCO2. The PaCO2 indicates whether the acidosis or alkalemia is primarily from a respiratory or metabolic acidosis/alkalosis. Paco2>40 with a pH<7.4, indicates a respiratory acidosis, and <40 and pH<7.4 indicates a respiratory alkalosis (but is often from hyperventilation from anxiety or compensation for a metabolic acidosis).

The healthcare provider would anticipate which of the following as a treatment option for pneumonitis? Corticosteroid administration Chest tube insertion Emergency thoracotomy Albuterol nebulization

corticosteroid administration Corticosteroids are often recommended to treat inflammatory conditions such as pneumonitis. The other options are not appropriate. Chest tubes are used for drainage. Albuterol is a drug for rapid bronchodilation. A thoracotomy is removal of the lung.

The doctor orders Zofran 8 mg PO t.i.d. The pharmacy sends a 100 mL bottle, labeled 4 mg/tsp. How many mL should be given for each dose? 10 mL 2 mL 8 mL 5 mL

10 mL 1 tsp = 5 mL and 4 mg. Multiply 4 mg × 2 to reach the ordered dose of 8 mg. It will take 10 mL (2 tsp) to give 8 mg of Zofran.

Before administering a dose of furosemide (Lasix) to a 2-year-old with a congenital heart defect, the nurse should confirm the child's identity by checking the hospital ID band and verifying the child's identity with a second nurse. verifying the child's room number. asking the child to state their name. asking the parent for the child's name.

Asking the parent for the child's name. Standards of safe medication administration require obtaining two patient identifiers before proceeding. For a child, a parent can give the child's name. Many young children do not know their full name or are accustomed to being called by a nickname. Adults can be asked the child's name and birth date. Room numbers are not a reliable means of verifying identification.

After emptying a Jackson-Pratt drainage bulb, how does the nurse reestablish negative pressure in the system? Fill the bulb with sterile saline solution. Place the bulb lower than the client's body. Compress the bulb and close the valve. Open the valve and fill the bulb with air.

Compress the bulb and close the valve- CORRECT A Jackson-Pratt drain creates negative pressure when the bulb is compressed and the valve is closed. This causes fluid around the surgical site to flow into the drain.

Nurse Lourdes is teaching a client recovering from Addisonian crisis about the need to take fludrocortisone acetate and hydrocortisone at home. Which statement by the client indicates an understanding of the instructions? A. "I'll take my hydrocortisone in the late afternoon, before dinner." B. "I'll take all of my hydrocortisone in the morning, right after I wake up." C. "I'll take two-thirds of the dose when I wake up and one-third in the late afternoon." D. "I'll take the entire dose at bedtime."

"I'll take two-thirds of the dose when I wake up and one-third in the late afternoon." Hydrocortisone, a glucocorticoid, should be administered according to a schedule that closely reflects the body's own secretion of this hormone; therefore, two-thirds of the dose of hydrocortisone should be taken in the morning and one-third in the late afternoon. This dosage schedule reduces adverse effects. Option A: Taking the medicine at this schedule may cause adverse effects. Option B: The dosage should be according to the imitation of when the body secretes glucocorticoids. Option D: Taking an entire dose might cause severe adverse effects.

A provider has prescribed isotretinoin (Accutane) for a 22-year-old female with a diagnosis of severe cystic acne. Before the nurse gives the client instructions for taking the medication, which of the following is the MOST important question that the nurse should ask the client? "Can you tolerate some slight weight gain?" "What date was your last menstrual period?" "Are you able to swallow capsules easily?" "Does your insurance cover prescriptions?"

"What date was your last menstrual period?" A teratogen is an agent that can disturb the development of the embryo or fetus. Taking isotretinoin (Accutane) during pregnancy can cause birth defects of the brain and heart as well as facial deformities. Before providing the client any instructions on taking this medication, the nurse should determine whether the client might be pregnant. The client should have a reliable birth control method. The capsules are swallowed whole with a full glass of water. Unusual weight gain or weight loss are listed as rare or unknown adverse reactions.

A nurse caring for several patients in the cardiac unit is told that one is scheduled for implantation of an automatic internal cardioverter-defibrillator. Which of the following patients is most likely to have this procedure? A. A patient admitted for myocardial infarction without cardiac muscle damage. B. A postoperative coronary bypass patient, recovering on schedule. C. A patient with a history of ventricular tachycardia and syncopal episodes. D. A patient with a history of atrial tachycardia and fatigue.

A patient with a history of ventricular tachycardia and syncopal episodes. An automatic internal cardioverter-defibrillator delivers an electric shock to the heart to terminate episodes of ventricular tachycardia and ventricular fibrillation. This is necessary in a patient with significant ventricular symptoms, such as tachycardia resulting in syncope. Indications are usually secondary where the patient has already suffered and survived cardiac arrest due to ventricular fibrillation/ventricular tachycardia, or primary when the patient is at high risk of sudden cardiac death due to VF/ VT but has never had any such event. Option A: A patient with myocardial infarction that resolved with no permanent cardiac damage would not be a candidate. ICD is a state of the art device that treats arrhythmias specifically those of ventricular origin like ventricular tachycardia and fibrillation. It has become the first line of defense in patients who are at high risk for sudden cardiac death (SCD) and has shown consistent survival benefit in cardiac arrest survivors (SCA), in patients with Heart failure and severe systolic dysfunction (left ventricular ejection fraction-LVEF less than or equal to 35%) as well as in patients with hypertrophic cardiomyopathy (HCM). Option B: A patient recovering well from coronary bypass would not need the device. ICD is essentially a pacemaker with the ability to recognize abnormally fast cardiac rhythm and provide immediate treatment which can be in the form of overdrive pacing called anti-tachycardia Pacing (ATP) or shock therapy which could be synchronized or unsynchronized, depending on the recognized rhythm and the pre-programmed rhythm detection algorithm. Option D: Atrial tachycardia is less serious and is treated conservatively with medication and cardioversion as a last resort. Secondary prophylaxis usually involves the event of cardiac arrest due to ventricular fibrillation (VF) or hemodynamically unstable, also known as pulseless, ventricular tachycardia (VT). Adequate workup and exclusion of reversible causes should be done first before deciding to put the device in, as is endorsed by the guidelines laid down by Heart Rhythm Society (HRS) and American College of Cardiology (ACC).

The term "blue bloater" refers to a male client which of the following conditions? A. Adult respiratory distress syndrome (ARDS) B. Asthma C. Chronic obstructive bronchitis D. Emphysema

Chronic obstructive bronchitis Clients with chronic obstructive bronchitis appear bloated; they have large barrel chest and peripheral edema, cyanotic nail beds, and at times, circumoral cyanosis. Option A: Clients with ARDS are acutely short of breath and frequently need intubation for mechanical ventilation and large amounts of oxygen. Option B: Clients with asthma don't exhibit characteristics of chronic disease. Option D: Clients with emphysema appear pink and cachectic.

Sickle Cell

Clients with sickle cell crises are treated with heat, hydration, oxygen, and pain relief. The supply of extra oxygen with oxygen therapy may be beneficial for some patients with sickle cell disease. But the use of oxygen therapy in sickle cell disease is controversial because high levels of oxygen are known to suppress the formation of new red blood cells. Oxygen therapy is, therefore, only recommended when oxygen levels drop below a critical threshold.

A client who has glaucoma is to have miotic eye drops instilled in both eyes. The nurse knows that the purpose of the medication is to: A. Anesthetize the cornea B. Dilate the pupils C. Constrict the pupils D. Paralyze the muscles of accommodation

Constrict the pupils Miotic eye drops constrict the pupil and allow aqueous humor to drain out of the Canal of Schlemm. Pilocarpine is a muscarinic acetylcholine agonist that is effective in the treatment and management of acute angle-closure glaucoma and radiation-induced xerostomia. Although not a first-line treatment for glaucoma, it is useful as an adjunct medication in the form of ophthalmic drops. Option A: The cornea and conjunctiva can be anesthetized by drops of any of the local anesthetics. Bupivacaine, for example, is effective as an eyedrop. Bupivacaine is a potent local anesthetic with unique characteristics from the amide group of local anesthetics, first discovered in 1957. Local anesthetics are used in regional anesthesia, epidural anesthesia, spinal anesthesia, and local infiltration. Local anesthetics generally block the generation of an action potential in nerve cells by increasing the threshold for electrical excitation. Option B: The eye doctor may dilate the pupils with a special medication called a mydriatic, so they can see the inside of the eye during a slit lamp test. Typically, mydriasis reverses within 4 to 8 hours. However, it may take 24 hours for the mydriatic effect to wear off in some individuals. Weaker strength may cause mydriasis with little cycloplegia. Option D: Scopolamine ophthalmic is an anticholinergic agent that blocks constriction of sphincter muscle of iris and ciliary body muscle, which, in turn, results in mydriasis (dilation) and cycloplegia (paralysis of accommodation). Scopolamine competitively inhibits G-protein coupled post-ganglionic muscarinic receptors for acetylcholine and acts as a nonselective muscarinic antagonist, producing both peripheral antimuscarinic properties and central sedative, antiemetic, and amnestic effects.

Which of the following diagnostic tools is most commonly used to determine the location of myocardial damage? Cardiac catheterization Cardiac Enzymes Echocardiogram Electrocardiogram

Electrocardiogram The ECG is the quickest, most accurate, and most widely used tool to determine the location of myocardial infarction. A) Cardiac Catheterization is an invasive study for determining coronary artery disease and may also indicate the location of myocardial damage, but the study may not be performed immediately. B) Cardiac Enzymes are used to diagnose MI but can't determine the location. C) An echocardiogram is used most widely to view myocardial wall function after and MI has been diagnosed.

Mario comes to the clinic complaining of fever, drenching night sweats, and unexplained weight loss over the past 3 months. Physical examination reveals a single enlarged supraclavicular lymph node. Which of the following is the most probable diagnosis? A. Influenza B. Sickle cell anemia C. Leukemia D. Hodgkin's disease

Hodgkin's disease Hodgkin's disease typically causes fever night sweats, weight loss, and lymph node enlargement. Option A: The incubation period ranges from 1 to 4 days. Peak virus shedding usually occurs from 1 day before the onset of symptoms to 3 days after. Option B: Clients with sickle cell anemia manifest signs and symptoms of chronic anemia with pallor of the mucous membrane, fatigue, and decreased tolerance for exercise; they don't show fever, night sweats, weight loss or lymph node enlargement. Option C: In some cases, the signs of leukemia may include noticeable swelling of the neck, armpit, or groin. This occurs when leukemia has spread to the lymph nodes.

Hypovolemic Shock Symptoms

Tachycardia, Tachypnea, Hypotension

The hospitalist orders IV therapy for a patient: "750 mL D5W to infuse over 7 hours." With a drop factor of 10 gtts/mL, what should the drip rate be? 36 gtts/min 18 gtts/min 75 gtts/min 100 gtts/min

18 gtts/min To calculate a drip rate, divide the total volume in mL by the time in minutes. Then multiply that rate by the drop factor in gtts/mL. For this question: 750 mL ÷ 420 minutes = 1.8 mL/min. Then 1.8 mL/min × 10 gtts/mL = 18 gtts/min. The other calculations are incorrect.

A patient is prescribed a thiazide diuretic for the treatment of hypertension. When teaching the patient about the medication, which of the following will the healthcare provider include? "I'll teach you how to take your radial pulse before taking the medication." "Stop taking this medication if you notice changes in how much you urinate." "Be sure to include a number of foods in your diet that are rich in potassium." "Take this medication every day with a large glass of water after your evening meal."

"Be sure to include a number of foods in your diet that are rich in potassium" Because thiazide diuretics produce an increase in urine output, the patient should avoid taking the medication in the evening so that sleep is not interrupted. Potassium is lost in the urine along with sodium and chloride, so the patient should be instructed to include potassium-rich foods in the diet to avoid hypokalemia. Examples of potassium-rich foods include avocados, spinach, sweet potatoes, yogurt, and bananas.

After a high school athlete sustains a fractured femur during a competition, a full leg plaster cast is applied. When the nurse provides discharge instructions to the athlete and their parents six hours later, which statement by the athlete indicates a need for further education? "I'll put an ice pack over the cast to relieve itching." "I will prop my cast on two pillows when I lie down." "I should walk around on my cast as soon as I get home." "I should call my doctor if my toes turn blue or become numb."

"I should walk around on my cast as soon as I get home."- CORRECT Plaster casts are made up of a bandage and a hard covering, usually plaster of Paris. Client instructions include: 1 Keep the limb raised on a soft surface, such as a pillow, for as long as possible in the first few days. This will help any swelling to go down. 2. Keep the cast dry. If the plaster gets wet, it weakens and is unable to support the bone. 3. Do not put anything into the cast to relieve itching. This can damage the skin and cause an infection. A hair dryer set on cool or an ice pack over the itchy area can help. 4. Immediately report any pain, tingling, or numbness, or if the toes turn blue or white.

A clinician is teaching inhaled corticosteroid technique. Which of the following should be included in teaching? "It is not necessary to rinse your mouth after using your inhaler." "Press the canister down, breathe in slowly and deeply, hold your breath for a few seconds, and then slowly exhale." "Spacers are not necessary. They are mostly for convenience and ease of handling the inhaler." "You should inhale as quickly and deeply as possible when you take a puff from the inhaler."

"Press the canister down, breathe in slowly and deeply, hold your breath for a few seconds, and then slowly exhale." Breathing in slowly and deeply and then holding one's breath allows for the medication to move farther down the patient's airway. If possible, the patient should count to ten before exhaling. Spacers are typically suggested because they prevent the medication from collecting on the tongue and the back of the throat. Because inhalers leave a residue in the mouth, they can increase the risk of developing Candida infections. The patient should always rinse their mouth after use.

During an arteriogram (angiogram), the patient suddenly says, "I'm feeling really hot." Which is the best response? "That feeling of warmth is normal when the dye is injected. It will last up to 20 seconds." "Let me get your doctor to explain this sensation to you." "The heat indicates that the clots in the coronary vessels are dissolving." "You are having an allergic reaction to the dye. I will get an order for Benadryl."

"That feeling of warmth is normal when the dye is injected. It will last up to 20 seconds." Patients should be instructed that it is normal to experience a warm sensation when IV contrast is injected. The feeling lasts 5-20 seconds.

A client with end-stage renal disease has opted for an arteriovenous (AV) fistula for long-term treatment with hemodialysis. Following the surgical creation of the AV fistula, when will the client be able to use it for hemodialysis? 2-3 weeks 4-6 months 2-3 months 4-6 weeks

2-3 months CORRECT An AV fistula is a connection of an artery to a vein, created by a vascular surgeon. An AV fistula frequently requires 2 to 3 months to develop or mature before the patient can use it for long-term hemodialysis.

While assessing a one-month-old infant, which of the findings warrants further investigation by the nurse? (SATA). 1. Abdominal respirations 2. Irregular Breathing Rate 3. Inspiratory grunt 4. Increased heart rate with crying 5. Nasal flaring 6. Cyanosis 7. Asymmetric chest movement

1. Abdominal respirations are normal among infants and young children. Since their intercostal muscles are not fully developed, they use their abdomen muscles much more to pull the diaphragm down for breathing. 2. Newborns can have irregular breathing patterns ranging from 30 to 60 breaths per minute with short periods of apnea (15 seconds). 3. Grunting occurs when an infant attempts to maintain adequate functional residual capacity in the face of poorly compliant lungs by partial glottic closure. As the infant prolongs the expiratory phase against this partially closed glottis, there is a prolonged and increased residual volume that maintains the airway opening and also an audible expiratory sound. 4. An increase in heart rate is normal for an infant during activity (including crying). Fluctuations in heart rate follow the changes in the newborn's behavioral state - crying, movement, or wakefulness corresponds to an increase in heart rate. 5. Nasal flaring occurs when the nostrils widen while breathing and is a sign of troubled breathing or respiratory distress. 6. Cyanosis refers to the bluish discoloration of the skin and indicates a decrease in oxygen attached to the red blood cells in the bloodstream. 7. Asymmetric chest movement occurs when the abnormal side of the lungs expands less and lags behind the normal side. This indicates respiratory distress.

Which individual is at the greatest risk for developing hypertension? 1. 45-year-old African American attorney 2. 60-year-old Asian American shop owner 3. 40-year-old Caucasian nurse 4. 55-year-old Hispanic teacher

1. CORRECT- African Americans develop high blood pressure at younger ages than other groups in the US. Researchers have uncovered that African Americans respond differently to hypertensive drugs than other groups of people. They are also found out to be more sensitive to salt, which increases the risk of developing hypertension. 2. The incidence of hypertension in Asian Americans does not appear to be significantly higher than the general population, according to limited US data. 3. The racial disparity in hypertension and hypertension-related outcomes has been recognized for decades with African Americans with greater risks than Caucasian's. 4. Hypertension prevalence rates in Hispanics may vary by gender and country of origin. Hispanic Americans overall have relatively low levels of hypertension, despite elevated levels of diabetes and obesity.

The nurse prepares to administer the prescribed aspirin to a client. Which client conditions cause the nurse to hold the medication? (SATA). 1. The client has a history of being in multiple alcohol treatment programs. 2. The client is receiving heparin for the presence of deep vein thrombosis. 3. The client experiences severe pain related to a diagnosis of osteoarthritis. 4. The client takes an oral hypoglycemic for treatment of type 2 Diabetes Mellitus. 5. The client has a recent history of a respiratory infection and elevated temperature.

1. CORRECT- Aspirin is a possible cause of gastric discomfort and is an increased risk for the development of gastric ulcers. With a history of alcoholism and multiple treatment events, this client is already at risk for gastric ulcers. The nurse will hold the aspirin for this client. 2. CORRECT- Aspirin is contraindicated for clients receiving anticoagulant drugs, such as heparin or warfarin. Aspirin is known to have significant anticoagulant activity and will increase a client'a bleeding time. The nurse should hold the aspirin for this client. 3. Aspirin is known to be effective for the pain and inflammation related to osteoarthritis. The nurse will not hold the Aspirin for this client. 4. CORRECT- Aspirin may potentiate the action of oral hypoglycemic drugs given for type 2 Diabetes Mellitus. Therefore, the aspirin should be held to avoid causing hypoglycemia. 5. Aspirin is effective in treating the pain and discomfort associated with a respiratory infection, and produces mild to moderate relief of fever. The nurse will not hold the aspirin prescribed to this client.

To measure an adult client's apical heart rate, where does the nurse place the stethoscope? 1. Fifth left intercostal space at midclavicular line 2. Second left intercostal space at midclavicular line 3. Fourth left intercostal space at midclavicular line 4. Third left intercostal space at midclavicular line

1. Fifth left intercostal space at the midclavicular line- CORRECT FILM = Fifth Intercostal Midclavicular Line. The apical pulse is auscultated with a stethoscope over the chest where the heart's mitral valve is best heard. For adults, the point of maximum pulse is the fifth left intercostal space at the midclavicular line. In infants and young children, the apical pulse is located at the fourth intercostal space at the left midclavicular line.

A nurse is to administer meperidine hydrochloride (Demerol) 100 mg, atropine sulfate (Atropisol) 0.4 mg, and promethazine hydrochloride (Phenergan) 50 mg IM to a pre operative client. List the order in which the nurse must carry out the following actions prior to the administration of pre operative medications. 1. Have the client empty bladder 2. Raise the side rails on the bed 3. Place the call bell within reach 4. Instruct the client to remain in bed

1. Have the client empty the bladder. The first step in the process is to have the client void prior to administering the pre operative medication. If the client does not have a catheter, it is important to empty the bladder before receiving preoperative medications to prevent bladder injury (especially in pelvic surgeries). Else, a straight catheter or an indwelling catheter may be ordered to ensure the bladder is empty. 2. Instruct the client to remain in bed. Pre operative medications can cause drowsiness and lightheadedness which may put the client at risk for injury. 3. Raise the side rails on the bed. Raising the side rails on the bed helps prevent accidental falls and injury when the client decides to get out of the bed without assistance. 4. Place the call bell within reach. Call bells should always be within the reach of a client.

A nurse working at a community health fair is administering intramuscular injections of influenza vaccine. Which injection technique will avoid leakage into subcutaneous tissue? 1. Injection by the Z-track method 2. Aspiration prior to injection 3. Massaging the injection site 4. Injection at a 45-degree angle

1. Injection by the Z-Track Method CORRECT The Z-track injection method avoids leakage into subcutaneous tissue. Pull the client's skin downward or upward, and inject the vaccination (or medication) at a 90-degree angle before releasing the skin. Aspiration is done to assure that the injection is not going into a vein or artery. Massaging the injection site can introduce the material into subcutaneous tissue. A 45-degree angle is incorrect.

The nurse teaching a 14-year-old client about her cervico-thoracolumbosacral orthosis (CTLSO) brace. Which statement by the client would indicate a lack of understanding about the brace? 1. "I should wear loose clothing underneath it." 2. "I can take it off in hot weather." 3. "I can remove it when I take a shower." 4. "I must wear it all day and night."

2. "I can take it off in hot weather." CORRECT The Milwaukee brace, also known as a cervico-thoracolumbosacral orthosis or CTLSO, is a back brace used in the treatment of spinal curvatures such as scoliosis or kyphosis in children. It is a full-torso brace that extends from the pelvis to the base of the skull. Its aim is to keep the body upright and prevent progression of the curve while the patient is growing and awaiting possible need for operative intervention. The brace must be worn long term, during periods of growth, usually for 1 to 2 years. The client's statement about not wearing the brace in hot weather is incorrect and indicates a need for additional teaching. The other statements indicate correct understanding.

The nurse is educating the parents of a young child with a recent diagnosis of cystic fibrosis. The nurse tells the parents that the child will be at risk for which vitamin deficiencies? 1. B1 and pantothenic acid 2. A, D, and K 3. B12, C, and E 4. Folic acid and biotin

2. A, D, and K- CORRECT People with cystic fibrosis have trouble absorbing fats, which means they also have trouble absorbing vitamins that need fat to be absorbed — A, D, E, and K. These fat-soluble vitamins are critical to normal growth and good nutrition. B-complex, C, folic acid, biotin, and pantothenic acid are water soluble and easily absorbed.

The palliative care nurse is caring for a client with advanced multiple myeloma. Which intervention is MOST appropriate? 1. Limit administration of opioid medications. 2. Use gentle repositioning techniques. 3. Maintain fluid intake at 1 liter per day. 4. Monitor blood work for hypocalcemia.

2. Use gentle repositioning techniques- CORRECT Multiple myeloma occurs when abnormal plasma cells (myeloma cells) collect in several bones. In 70% of multiple myeloma cases, the bones develop multiple holes, called osteolytic lesions. Multiple myeloma may also affect other tissues and organs, especially the kidneys. This type of cancer causes hypercalcemia, renal failure, anemia, and bone damage. Multiple myeloma can cause erosion of bone mass and fractures. Extra care should be taken when moving or positioning a client because of the risk of pathological fractures. Pain is intense from the bones and nerves and should be treated. Fluid hydration of 3 L daily is recommended throughout the disease course and improves overall survival.

A 10-year-old boy is admitted to the pediatric unit with a diagnosis of viral meningitis. He is experiencing a severe headache, a stiff neck, vomiting, photophobia, and drowsiness. The nurse can make him more comfortable by 1. providing him with a large, soft pillow. 2. closing the shades and dimming the lights. 3. encouraging him to drink fluids. 4. teaching him deep breathing.

2. closing the shades and dimming the lights- CORRECT The first action should be to alleviate the photophobia by darkening the room. This may alleviate the child's headache also. Deep breathing can be useful but will not eliminate the discomfort of meningitis symptoms. Fluid balance is important but not the first priority. A large, soft pillow may place the neck in an awkward position and exacerbate the child's discomfort by stretching the meninges.

The physician orders an IV theophylline drip at 40 mg/hr. The pharmacy sends a 250 mL bag of D5W mixed with 500 mg theophylline. What should the infusion rate on the pump be? 80 mL/hr 40 mL/hr 20 mL/hr 60 mL/hr

20 mL/hour To calculate the infusion rate: 1. Divide the dose that was ordered by the available dose. (40 mg ÷ 500 mg = 0.08) 2. Multiply that value by the quantity (250 mL) to be infused. ANSWER: 20 mL/hr.

What is the smallest gauge intravenous catheter that can be used to administer blood? 22-gauge 24-gauge 26-gauge 20-gauge

20-gauge - CORRECT An 18-gauge needle or catheter is generally used to administer blood or push fluids, or for testing protocols that require large IV bores. However, a 20-gauge is acceptable if the facility's policy allows it. This size is better for clients with small veins. A 22-gauge is used for IVs of short duration or for clients who are not critically ill. Usually, blood cannot be administered because of hemolysis of the RBCs. A 24-gauge is used for pediatrics and adults who cannot tolerate a larger gauge. A 16-gauge is mostly used in intensive care and surgery units because most fluids and blood products can be quickly administered. 26-gauge needles are used for injections.

The home health nurse notices that a client with a diagnosis of multiple sclerosis is recently having difficulty chewing and swallowing. What is the appropriate diet for this client? 1. Full liquid 2. Pureed 3. Mechanical soft 4. Clear liquid

3. Mechanical Soft- CORRECT A mechanical soft diet has a consistency that the client may be able to handle; this diet should be tried first. A full liquid diet can be challenging for a client with poor swallowing. Pureed foods can cause the client to regress or feel embarrassed. A clear liquid diet is not indicated and does not supply sufficient calories.

The purpose of a splint is to 1. provide permanent support for a fracture. 2. wrap around an injury for full protection. 3. immobilize and allow for tissue swelling. 4. manage complex or unstable fractures.

3. immobilize and allow for tissue swelling- CORRECT The primary purposes of splinting for limb fractures are to protect soft tissue from further damage, allow for tissue swelling, reduce the client's pain, reduce the possibility of a fat embolism, and minimize painful muscular spasms. A splint is easily applied and removed and involves fewer complications than a cast. A cast is indicated for total immobilization, to wrap completely around the fracture or injury, and to manage complex or unstable fractures.

A patient receiving vancomycin has an order for a trough level to be drawn. When should the lab collect the blood sample? 1 hour after the infusion 30 minutes after the infusion 30 minutes before the infusion 1 hour before the infusion

30 minutes before the infusion. A trough level should be drawn 30 minutes before the third or fourth dose. The other answers are incorrect times to draw blood levels. Vancomycin is indicated for serious gram-positive bacterial infections, so it is important to monitor dosage levels. The typical trough level range for vancomycin is 10-20 µg/mL. The reference range for peak levels is 25-50 µg/mL.

The nurse is providing education for a client who has just been prescribed a transcutaneous electrical nerve stimulation (TENS) unit for relief of chronic back pain. Which of the following instructions to the client is correct? 1. "It will take several days to build up tolerance." 2. "Each TENS unit session lasts about 3 hours." 3. "Muscle twitching means the TENS is working." 4. "Don't go to sleep with the TENS unit on."

4. "Don't go to sleep with the TENS unit on." -CORRECT TENS relieves acute and chronic pain by using a mild electrical current that stimulates nerve fibers to block the transmission of pain impulses to the brain. The current is delivered through electrodes placed on the skin at points determined to be related to the pain. Clients should not go to sleep with the TENS unit on. Clients are typically instructed to use the TENS unit for 30-60 minutes at a time, depending on the type of pain. TENS may result in instant and possibly prolonged pain relief. Muscle twitching may indicate overstimulation.

When instructing a patient with Addison's disease about nutrition, the healthcare provider should NOT recommend which of the following dietary modifications? 1. A diet high in grains 2. A high-protein diet 3. A diet with adequate caloric intake 4. A restricted-sodium diet

4. A restricted-sodium diet- CORRECT A patient with Addison's disease (adrenal insufficiency) requires normal dietary sodium to maintain electrolyte balance and prevent excess fluid loss. The patient should be instructed to maintain adequate caloric intake with a diet high in protein and complex carbohydrates, including grains.

A patient diagnosed with Crohn's disease has a new colostomy. When assessing the patient's stoma, which of these would alert the healthcare provider that the stoma has retracted? 1. Dry and reddish purple 2. Narrowed and flattened 3. Pinkish red and moist 4. Concave and bowl shaped

4. Concave and bowl shaped.- CORRECT A colostomy is created when the bowel is pulled through an opening in the abdominal wall, creating a stoma through which intestinal contents will pass. A healthy stoma will protrude about 2.5 cm with an open lumen at the top. The stoma should appear pinkish red and moist. A dry, dusky, or reddish-purple stoma indicates ischemia. A narrowed, flattened, or constricted stoma indicates stenosis. A concave and bowl-shaped stoma has retracted. A retracted stoma can be difficult to care for. Complications include problems maintaining appliance placement, leading to leakage and sore skin.

A patient with a diagnosis of lung cancer is receiving chemotherapy. The patient reports nausea and loss of appetite, resulting in decreased food intake. What should the healthcare provider recommend to the patient to promote adequate nutrition? 1. Eat large meals but less frequently throughout the day. 2. Eat only favorite foods to increase appetite. 3. Eat only when feeling hungry. 4. Eat small meals throughout the day.

4. Eat small meals throughout the day. CORRECT Because of chemotherapy-induced nausea, the patient may not feel hungry, but they should be encouraged to eat even if not hungry. Encouraging the patient to eat small meals frequently throughout the day can help avoid nutritional deficiencies and improve the patient's quality of life. Chemotherapy can cause changes in smell or taste; aversion to favorite foods can result.

Of the following positions, which one facilitates maximum air exchange? 1. Trendelenburg 2. Lithotomy 3. High Fowler's 4. Orthopneic

4. Orthopneic- CORRECT Orthopneic (sitting in a leaning position) allows for the most lung expansion. High Fowler's can help, but it isn't as effective as the orthopneic position. Trendelenburg is used for hypotension or low cardiac output. Lithotomy is used for vaginal examinations and childbirth.

A client returns to the unit after abdominal surgery. While monitoring the client, the nurse observes a moderate amount of red blood on the dressing. The nurse will document this type of wound drainage as 1. Purulent 2. Serous 3. Serosanguineous 4. Sanguineous

4. Sanguineous- CORRECT The nurse will document this drainage as Sanguineous. The word comes from the Latin, meaning "blood." Wound drainage is described by type, color, amount, and odor. Types of drainage are: 1. Serous: clear and thin; may be present in a healthy, healing wound. 2. Serosanguineous: containing blood; may also be present in a healthy, healing wound. 3. Sanguineous: primarily blood. 4. Purulent: thick, white, and pus-like; may be indicative of infection and should be cultured. The amount of drainage is generally documented as absent, scant, minimal, moderate, large, or copious. The presence and degree of odor can be documented as absent, mild, or foul. Foul odors can be indicative of an infection.

The nurse takes an order for 240 mg acetaminophen (Tylenol) for a 5-year-old patient. The bottle is labeled 160 mg/5 mL. The nurse will administer 10 mL. 2.5 mL. 7.5 mL. 5 mL.

7.5 mL. To calculate the correct dose, divide the desired dose by the amount on hand and then multiply that quotient by the volume (D/H × V). For this question: 240 mg ÷ 160 mg × 5 mL =7.5 mL.

A health care provider (HCP) prescribes an intravenous medication to be administered 25 mcg/kg/min. The client weighs 52 kg. How many milligrams (mg) will the client receive in one hour? 0.78 mg 7.8 mg 78 mg 780 mg

78 mg 1 mcg = 0.001 mg. To convert 25 mcg, set up a proportion: 1/0.001 = 25/x. Cross-multiply (0.001 × 25 = 0.025 mg) and then multiply by 52 kg = 1.3 mg per minute. In one hour (60 min × 1.3 mg), the client should receive 78 mg.

Before administering two units of whole blood, what type of intravenous (IV) device should a healthcare provider use? The smallest possible catheter to prevent pain on insertion. The same IV device as previously used. Consult the patient's chart. Whatever the doctor has ordered. Consult the patient's chart. A large bore catheter to allow blood cells to pass easily into the patient.

A large bore catheter to allow blood cells to pass easily into the patient Large bore catheters are necessary to prevent damage to blood cells. This also decreases development of clots from hemolysis. An 18-gauge needle is often the standard for blood administration.

The nurse is planning care for a client with status-post CVA and limited mobility. The client is scheduled for occupational therapy at 10:00 a.m. and physical therapy at 2:00 p.m. What times are most appropriate for nursing care? AM care at 0800, PM care at 1600 AM care at 0900, PM care at 1500 AM care at 0900, PM care at 1600 AM care at 0800, PM care at 1500

AM care at 0800, PM care at 1600- CORRECT Clients who are recovering from a serious condition and have limited mobility require extra time. Not only do they need more time to prepare and to move but they also need more time to rest before and after therapy sessions. The only correct option is to provide AM care at 0800 and PM care at 1600.

Before administering a scheduled 300 mL enteral feeding bolus to a comatose adult client, the nurse aspirates 100 mL of gastric residual volume. Which nursing action is MOST appropriate? Administer the bolus as prescribed. Flush the tubing with warm water. Request a different enteral formula. Hold the feeding bolus for two hours.

Administer the bolus as prescribed- CORRECT Standard practice includes measuring gastric residual volume prior to administering an enteral feeding. According to current American Society for Parenteral and Enteral Nutrition, enteral feedings can be administered with a residual up to 500 mL; however, individual HCP orders should be followed. signs of feeding intolerance include abdominal distention and/or pain, constipation, nausea, vomiting, and sense of fullness.

The pediatrician prescribes amoxicillin suspension 200 mg PO q 8 hr for a 3-year-old client with acute otitis media. The child weighs 38 pounds (17.2 kg). The recommended daily dose range for children up to 40 kg is 20-40 mg/kg/day in divided doses every 8 hours. Following the 10 Rights of Medication Administration, what is the nurse's BEST action? Administer the medication as prescribed. Refuse to give because it is not the drug of choice. Contact the pediatrician to clarify the dose. Withhold the dose because it is too low.

Administer the medication as prescribed. The nurse should administer the dose, since it is within appropriate guidelines. The child may safely receive 344-688 mg/day, in divided doses of 115-230 mg each. Amoxicillin (20-40/kg/day) is the antibiotic of choice for treating acute otitis media in clients older than 2 years who are not allergic to penicillin. Approximately 80% of children will have at least one episode of acute otitis media. The 10 Rights of Medication Administration that are applicable here: Right patient, right medication, right dose, right route, right time, and right assessment.

Despite frequent turning and skin assessment, occasional urinary incontinence has caused a bedridden resident to develop a reddened and tender area on the coccyx. The resident weighs 192 pounds. Which pressure-relieving device should be used for the client? Low air loss bed Alternating overlay Egg crate foam Natural sheepskin

Alternating Overlay- CORRECT For clients who weigh less than 250 pounds, an alternating pressure overlay is the best choice because it is liquid resistant. It has compartments that alternately inflate and deflate to relieve pressure. Foam and sheepskin surfaces are not appropriate for clients with urinary incontinence. A low air loss bed is ideal but expensive, and it can cause hypothermia if not carefully monitored.

A healthcare provider is caring for a patient receiving intravenous chemotherapy for cancer. The healthcare provider will plan to administer the prescribed antiemetic to the patient one hour after the infusion is complete. before starting the infusion. if the patient complains of nausea. one half hour after the infusion has started.

Before starting the infusion. Most chemotherapeutic agents have a high potential for causing nausea and vomiting. Up to 80% of patients experience chemotherapy-induced nausea and vomiting, which occur when the vomiting center in the brain is stimulated during the chemotherapy infusion. Administering an antiemetic prior to starting the infusion can prevent or minimize nausea and vomiting.

Before a patient can receive a transfusion of whole blood, the laboratory and blood bank require a sample of the patient's blood. Which tests will be run? Blood type and antigen screen Complete blood count (CBC) and differential Blood culture and sensitivity analysis Blood type and crossmatch

Blood Type and Crossmatch A blood type and crossmatch is necessary to ensure a match between the blood donor and the patient who is receiving the blood. An incompatible match could result in severe adverse events and possible death. CBC, differential, culture, and sensitivity are unnecessary. Blood type and antigen screen is incorrect; the proper test is blood type and antibody screen, which will be performed as part of the crossmatch.

A healthcare provider is performing an assessment of a patient who is taking propranolol (Inderal) for supraventricular tachycardia. Which assessment finding is an indication that the patient is experiencing an adverse effect of this drug? Paresthesia Urinary retention Bradycardia Dry mouth

Bradycardia Beta-1 receptors are found in the cardiac conduction system and myocytes. Beta-1 blockade will slow discharge from the SA node and decrease speed through the AV node, slowing the heart rate. Propranolol (Inderal) decreases the strength of heart contractions as well as the heart rate, resulting in less cardiac oxygen consumption.

A patient with a spinal cord injury (SCI) complains about a severe throbbing headache that suddenly started a short time ago. Assessment of the patient reveals increased blood pressure (168/94) and decreased heart rate (48/min), diaphoresis, and flushing of the face and neck. What action should you take first? Administer the ordered acetaminophen (Tylenol). Check the Foley tubing for kinks or obstruction. Adjust the temperature in the patient's room. Notify the physician about the change in status.

Check the Foley. These signs and symptoms are characteristic of autonomic dysreflexia, a neurologic emergency that must be promptly treated to prevent a hypertensive stroke. The cause of this syndrome is noxious stimuli, most often a distended bladder or constipation, so checking for poor catheter drainage, bladder distention, or fecal impaction is the first action that should be taken.

When a patient's nasogastric (NG) tube stops draining, what is the nurse's first action? Retract 2 inches. Instill 50 mL of water. Clamp for 1 hour. Check tube placement.

Check tube placement-CORRECT ALWAYS verify tube placement before taking other measures. NEVER put anything in an NG tube unless you know that its tip is in the stomach. Clamping has no effect on NG tube placement. Retracting without knowing where the tip is would be unsafe for the patient.

A pediatric patient has been diagnosed with right lower lobe pneumonia. Upon auscultation of this lung field, the healthcare provider should expect to hear which breath sounds? Rhonchi Crackles Wheezes Stridor

Crackles- CORRECT Crackles would most likely be heard because they indicate fluid in the airspace. Fluid in the airspace is consistent with pneumonia. Wheezes indicate a narrowing of the airways. Stridor is an emergency lung sound that is seen in airway constriction that can lead to complete closure. Rhonchi are heard in mixed-issue airway constriction and secretions.

The purpose of medication reconciliation is to create an accurate list of all medications that a client is taking at home and during hospitalization. document all doses of each medication that a client receives during their inpatient stay. ensure that the inpatient unit's narcotic count is correct at the start and end of every shift. compare pharmacy charges with actual doses administered during hospitalization.

Create an accurate list of all medications that a client is taking at home and during hospitalization. Medication reconciliation is the process of creating the most accurate list possible of all medications that a patient is taking — including drug name, dosage, frequency, and route — and comparing that list against the physician's admission, transfer, or discharge orders. The goal is to provide correct medications to the patient at all transition points within the hospital.

The purpose of holding a sterile gauze pad on the site of an IM injection while removing the needle is to seal off the track left by the needle. increase the absorption of the medication. decrease the discomfort of the needle pulling on the skin. prevent pathogens from entering through the puncture.

Decrease the discomfort of the needle pulling on the skin. Pressing the gauze pad against the skin while removing the needle reduces the discomfort of the pulling sensation. Once the medication has been completely injected, remove the needle using a smooth, steady motion. Remove the needle at the same angle at which it was inserted.

During the first 24-48 hours of total parenteral nutrition (TPN), what laboratory test result is MOST important for the nurse to monitor? Electrolytes Calcium BUN Glucose

Glucose A standard TPN solution has 25% carbohydrates. During the first few days of TPN therapy, the nurse must monitor closely for hyperglycemia. Check blood glucose q 4-6 hrs, administering insulin as ordered. Calcium, electrolytes, and BUN are monitored daily, but these are not the priority of initial TPN therapy.

A patient with a total hip replacement requires certain equipment for recovery. Which of the following will assist the patient with activities of daily living (ADL)? Abduction pillow TENS unit High-seat commode Recliner

High-seat commode- CORRECT A high-seat commode keeps the hip higher than the knee. A recliner is helpful because it prevents 90° flexion, but it is not necessary for activities of daily living (ADL). A TENS (Transcutaneous Electrical Nerve Stimulation) unit helps with pain management, and an abduction pillow is used to prevent hip adduction and possibly dislocation of the prosthesis, but neither are part of ADL.

Before administering penicillin IM to a client with"NKDA" on his medical record, the nurse asks the client if he has any drug allergies. The client responds, "I'm not sure. I may have had a rash one time after I had a shot when I was sick." Which is the appropriate action for the nurse? Contact the hospital pharmacist for oral penicillin. Administer the penicillin and observe for 60 minutes. Hold the penicillin and contact the prescribing provider. Ask the client for more details before giving the penicillin.

Hold the penicillin and contact the prescribing provider The nurse should hold the penicillin and contact the provider who ordered the medication. Even a mild reaction in the past puts the client at risk for an anaphylactic reaction, which typically occurs within an hour of administration. Common adverse effects of penicillin include rash; hives; itching; and swelling of the face, lips, and tongue.

A patient presents to the emergency department with a complaint of watery diarrhea for the past three days. Assessment findings include blood pressure 100/60, pulse 98, and dry mucous membranes. The healthcare provider would anticipate intravenous therapy administration with which of the following fluids? Hypotonic crystalloid Hypertonic crystalloid Isotonic crystalloid Colloid solution

Isotonic crystallloid Hypovolemia is corrected by expanding the intravascular compartment. An isotonic IV solution will expand the intravascular compartment without affecting cells and tissues of other fluid compartments. Hypertonic and colloid solutions would pull fluid into the intravascular space but at the expense of other fluid compartments.

A father brings his 6-month-old son and 3-year-old daughter for their routine check-ups. What is the nurse's best strategy? Let the 3-year-old play during the baby's examination. Ask the father to step out with one child while the other child is examined. Examine the 6-month-old first while his sister watches. Examine the 3-year-old first so she can show her brother.

Let the 3-year-old play during the baby's examination- CORRECT Start with the least anxious and most cooperative sibling. Allow the older sibling to play; this reduces anxiety and allows them to become familiar with the examination. Children are more comfortable and cooperative when a parent is present.

A client with deep vein thrombosis (DVT) is receiving heparin therapy by continuous intravenous infusion. Prior to initiating the heparin infusion, the client's activated partial thromboplastin time (aPTT) was 32 seconds. The nurse notes that the most recent aPTT result is 70 seconds. What is the nurse's BEST action? Decrease the heparin infusion rate. Administer protamine sulfate stat. Increase the heparin infusion rate. Maintain the heparin infusion rate.

Maintain the heparin infusion rate. The nurse should maintain the current heparin infusion rate. The reference range of aPTT is 30-40 seconds. To obtain an aPTT result, an activator is added that speeds up the clotting time and results in a narrower reference range. The aPTT is considered a more sensitive version of the PTT and is used to monitor the patient's response to heparin therapy. Decreasing or increasing the infusion rate is not appropriate. Protamine sulfate is a heparin antagonist, used to immediately reverse the action of heparin. It is not indicated for this situation.

During an acute exacerbation of inflammatory bowel disease, a patient is to receive total parenteral nutrition (TPN) and lipids. Which of these interventions is the priority when caring for this patient? Change the administration set every 72 hours. Monitor urine specific gravity every shift. Infuse the solution in a large peripheral vein. Monitor the patient's blood glucose per protocol.

Monitor the patient's blood glucose per protocol. Total parenteral nutrition (TPN) can cause hyperglycemia, so blood glucose levels should be closely monitored. Because of the hypertonicity of the TPN solution, it must be administered via a central venous catheter. The high concentrations of glucose and lipids make TPN an excellent medium for bacterial growth. Therefore, administration sets should be changed every 24 hours if the TPN contains lipids.

Which of the following drugs should not be refrigerated? Nadolol (Corgard) Epoetin alfa IV (Epogen) Opened (in-use) Humulin N injection Ampicillin Oral Suspension

Nadolol (Corgard) Nadolol (Corgard) is a beta blocker used to treat hypertension and chest pain. It is stored tightly closed at room temperature from 59 °F to 86 °F (15 °C to 30 °C), away from heat, moisture, and light. Humulin N is a form of insulin. After it has been opened, it is stored in the refrigerator or at room temperature below 86 °F. Humulin N must be discarded within 31 days after opening. Ampicillin is a penicillin antibiotic. The liquid suspension form should be stored in the refrigerator for up to 7 days. Epoetin alfa IV is used with patients undergoing chemotherapy or suffering from anemia from serious chronic diseases. It is stored in the refrigerator and should be protected from light.

A nurse is caring for a client whose heel has a pressure ulcer covered with intact hard, dry, black tissue. Which is the appropriate dressing for this client? Apply a hydrocolloid dressing. Cover with sterile gauze. No dressing is necessary. Do a wet-to-dry dressing change.

No dressing is necessary- CORRECT Current standard of care guidelines recommend that stable, intact (dry, adherent, intact without erythema) eschar on the heels should not be removed. Eschar works as a natural barrier or biological dressing by protecting the wound bed from bacteria. Unless it is wet, draining, or loose, it should remain in place. Unless the nurse is a certified wound specialist, removal or debridement of eschar is performed by a health care provider (HCP). The other dressings are not indicated.

The Wong-Baker FACES® Pain Rating Scale can be used by all of the following EXCEPT Mentally Disabled Adults Parents and Caregivers Non-English Speakers Children 3 years and older

Parents and Caregivers-CORRECT The Wong-Baker FACES® Pain Scale is a self-assessment tool that is understood by the patient, so they can choose the face that best matches their physical pain. It is NOT to be used by a third person, parents, caregivers, or healthcare professionals to assess the patient's pain. It was originally created for use with children, so they could communicate about their pain. It is now used around the world for people ages 3 and older. (Wong-Baker FACES® Pain Scale used with permission.)

When a drug's effect is increased after a second drug is administered, this interaction is called antagonism. absolution. synergism. potentiation.

Potentiation Potentiation is a type of drug interaction. Potentiation occurs when two drugs are taken together and the action of one drug increases the action of the other, causing the pharmacologic response to be greater for one of the drugs.

The healthcare provider is preparing a patient for a total colectomy and the placement of an ileostomy. The patient asks where the stoma will be located. The healthcare provider identifies which of the following areas as the correct stoma site? RLQ RUQ LLQ LUQ

RLQ- CORRECT A total colectomy involves the removal of the large intestine. The gastrointestinal output will occur at the terminal end of the ileum. The stoma will be located in the right lower quadrant of the abdomen. A total colectomy is performed for intestinal inflammatory conditions such as Crohn's disease or ulcerative colitis. It may also be done for severe chronic constipation.

During assessment, the home health nurse learns that the client has a fecal impaction. Before proceeding to manually remove the stool, what is the nurse's PRIORITY? Teach family members to perform the disimpaction process. Give an analgesic or sedative to make the client comfortable. Recall that cardiac dysrhythmias are a possibility. Advise the family to increase the client's fluid and fiber intake.

Recall that cardiac dysrhythmias are a possibility- CORRECT A fecal impaction is a large lump of dry, hard stool that stays stuck in the rectum. It is most often seen in people who are constipated for a long time. Treatment of a fecal impaction includes administrating an enema to soften the stool to produce a bowel movement, or manually removing the impaction. With a lubricated glove, insert the index finger into the rectum to break up the hardened stool with a circular motion. Cardiac dysrhythmias and reflex bradycardia can occur from vagal nerve stimulation.

Which of the following is the proper procedure for cleaning and storing dentures as part of a client's HS oral care? Remove dentures, clean with hot water, place in empty denture cup, leave within reach at client's bedside. Remove dentures, rinse quickly with hot water, place on a paper towel next to the sink. Remove dentures, clean with cool water, wrap them in a clean washcloth, leave next to the sink. Remove dentures, clean with cool water, place in denture cup with cool water, leave within reach at client's bedside.

Remove dentures, clean with cool water, place in denture cup with cool water, leave within reach at client's bedside- CORRECT Proper denture care is crucial to maintaining the life of the appliance and sustaining oral health. If dentures aren't properly cared for, plaque can build up, causing additional tooth loss, bad breath, and even gum disease. Dentures are fragile and easily damaged; they're also expensive, costing several thousand dollars. After removing them, the proper procedure for denture care is as follows: 1. Take the dentures to the sink, line the basin with a washcloth, and fill the basin partway with cool or tepid water, but do not place the dentures in the sink. 2. Using a toothbrush, clean the surface of the dentures as if you were brushing your own teeth. Work with one denture at a time. Using a small amount of toothpaste or denture cleaner is appropriate. Be sure to clean the areas where the denture comes in contact with the gums or roof of the mouth. 3. Rinse the denture thoroughly and place it into a clean denture cup filled with cool water. Repeat this process with the other denture. 4. Place the cup within the client's reach. 5. Assist the patient with proper oral care using sponge swabs and mouthwash. Look in the client's mouth for signs of infection such as lesions. NEVER store dentures in a washcloth or paper towel; they could accidentally be thrown away.

A client has a pressure ulcer on the sacrum. While assessing it, the nurse observes that it has partial thickness, loss of dermis, and a red-pink wound bed. Which stage will the nurse assign this pressure ulcer? Stage III Stage IV Stage II Stage I

Stage II- CORRECT Stage I pressure ulcers have intact skin with a reddened area that may be firm and painful. Stage II pressure ulcers are indicated by partial thickness, loss of dermis, and a red-pink wound bed. Stage III pressure ulcers have full-thickness skin loss and may contain slough, visible subcutaneous tissue, and tunneling. Stage IV pressure ulcers have full-thickness skin loss and exposed muscle, bone, or tendons.

The healthcare provider prepares to administer a corticosteroid to a patient with a diagnosis of asthma. What is the rationale for administering this drug to this patient? To decrease airway swelling To prevent respiratory infections To promote bronchodilation To promote expectoration of mucus

To decrease airway swelling Corticosteroids and other anti-inflammatory drugs work by reducing inflammation, swelling, and mucus production in the airways of a person with asthma. As a result, the airways are less inflamed and less likely to react to asthma triggers. Inhaled corticosteroids are the primary treatment for asthma.

A patient diagnosed with type 2 diabetes mellitus is admitted to the medical unit with pneumonia. The patient's oral antidiabetic medication has been discontinued, and the patient is now receiving insulin for glucose control. Which of the following statements best explains this change in medication? The infection has compromised beta cell function, so the patient will need insulin from now on. Stress-related states such as infections increase the risk of hyperglycemia. Insulin administration will help prevent hypoglycemia during the illness. Acute illnesses like pneumonia will cause increased insulin resistance.

Stress-related states such as infection increase the risk of hyperglycemia. Infections cause a stress response in the body by increasing the amounts of such hormones as glucocorticoids and epinephrine, which suppress the natural immune response. These stress hormones work against insulin and cause an increase in blood glucose levels. With high glucose levels, white cells are slowed and take longer to fight the infection. Type 2 diabetics may temporarily require insulin during acute illnesses and hospitalizations, but they often return to their normal medication regimen after they recover. Insulin injections also permit more accurate control of glucose levels than oral medications.

The parent of a child with a short leg fiberglass cast phones the clinic because the child complains of constant itching inside the cast. Which intervention is appropriate for the nurse to suggest? Trickle ice water into the cast. Tap on the cast at the itchy spot. Apply powder or a mild lotion. Use a blunt-ended object to scratch the itchy spot.

Tap on the cast at the itchy spot- CORRECT The most common complaint about wearing a cast is the itching sensation. Clients should never use an object (pencil, ruler, chopstick, etc.) to reach into the cast; this could break the skin and cause an infection. Powders and lotions are not recommended either. Appropriate interventions include locating the itch and tapping on it from outside the cast, blowing cool air from a blow dryer into the cast, and wrapping a watertight ice pack or a sealed bag of frozen vegetables on the outside of the cast to cool the itchy area.

For a patient on gentamicin therapy, which laboratory value should the nurse monitor? Sodium Potassium Creatinine Bilirubin

creatinine Gentamicin can cause nephrotoxicity. An elevated creatinine level indicates abnormal kidney function. Gentamicin is toxic to the kidneys, so labs must be constantly monitored and reported. The other options are incorrect.

The nurse is teaching parents to instill eye drops for their 4-month-old daughter. The parents tell the nurse that she shuts her eyes tightly to avoid the drops. Which instruction by the nurse is most appropriate? The parents should wait until their daughter is relaxed. The parents should instill the drops into the conjunctival sac. The parents should put the drops into the inner canthus. The parents should open her eyes with a thumb and forefinger.

The parents should put the drops into the inner canthus.-CORRECT Infants instinctively resist anything regarding their eyes by tightly closing them. The best way to instill eye drops is to gently restrain the baby's head while the baby is in a supine position and put the drops at the inner canthus of the eyes. When the baby opens its eyes and blinks, the medication will flow into the eyes. The nurse should encourage the parents to be gentle and speak to the baby during the procedure.

A patient who has a history of chronic back pain requires a higher dose of an opioid medication to achieve adequate pain relief. The healthcare provider suspects that these findings are a result of which of the following? Dependence Tolerance Addiction Pseudoaddiction

Tolerance Tolerance is a decrease in sensitivity to a medication. It is a common occurrence when opioids are taken for an extended period time, and leads to the patient requiring a progressively larger dose to achieve the same degree of pain management. When a patient seeks assistance to manage their pain, the patient's behavior is sometimes referred to as pseudoaddiction. Addiction is characterized by behaviors that include impaired control over substance use, cravings, compulsive use, and continued use despite harm.

When taking the blood pressure of a client who is seated, which position must the client change so that the nurse can get an accurate measurement? Sit up straight. Uncross the legs. Lean the head back. Put down the phone.

Uncross the legs- CORRECT The nurse should ask the client to uncross their legs because the position can compress blood vessels and affect the accuracy of the reading. The other positions do not affect circulation or the measurement.

The nurse is preparing to administer procedural morphine sulfate as an analgesic to a 4-year-old child who weighs 40 pounds. The health care provider (HCP) has prescribed 4 mg morphine sulfate IV. What is the nurse's BEST action? Verify the dosage for the child. Contact the pharmacy for advice. Administer the dose as ordered. Give half the prescribed dose.

Verify the dosage for the child. The nurse should verify the dosage strength (Right Dose) for the child before administering. For this child (40 lbs = 18.2 kg) the dosage should be 1.5-1.8 mg/dose. For procedural analgesia and sedation in children, morphine sulfate is given 0.08-0.1 mg/kg/dose IV, IM, or SC before the procedure and every 5-10 minutes as needed. Peak effect is 15-30 minutes after IV administration and 30-60 minutes after IM administration. Morphine sulfate is indicated for procedural analgesia because it is reliable, predictable, and easily reversed with naloxone. NOTE: An average 4-year-old weighs 40 pounds and is 40 inches tall.

A patient receiving chemotherapy is experiencing stomatitis. Which of the following should the healthcare provider offer the patient? Hot soup for lunch and dinner Warm saline rinses four times each day Vigorous oral care with a commercial mouthwash Plenty of ice chips between meals

Warm saline rinses four times each day- CORRECT Stomatitis is irritation of the lips, mouth, tongue, and oropharynx, which occurs when chemotherapy kills healthy cells that are rapidly dividing. It can impair nutrition, speech, sleep, and the quality of life. Warm saline rinses are non-irritating and help eliminate bacteria that can cause infection. Other nursing interventions include gentle oral hygiene and administration of a topical analgesic as ordered by the physician.

A client with lung cancer is going to receive an intravenous infusion therapy for bone metastases. The nurse anticipates that the physician will order which medication? Zoledronic acid (Zometa) Doxorubicin (Doxil) Capecitabine (Xeloda) Docetaxel (Taxotere)

Zoledronic acid (Zometa) Zoledronic acid (trade names include Zometa, Reclast, and Aclasta) is a bisphosphonate, a drug that slows or prevents bone loss. It inhibits osteoclasts, which are responsible for breaking down and reabsorbing minerals, including calcium. Docetaxel (Taxotere) is used for metastatic prostate cancer. Doxorubicin (Doxil) and capecitabine (Xeloda) are used to treat metastatic breast cancer.

A client with chronic renal failure (CRF) is learning to perform peritoneal dialysis at home. The nurse instructs the client to warm the dialyzing solution to 37 degrees Celsius so that it will relax the abdominal muscles. dilate the peritoneal blood vessels. maintain a constant body temperature. remove toxins from the body's cells.

dilate the peritoneal blood vessels- CORRECT The rationale for warming the peritoneal dialysis solution is that the warm solution helps dilate peritoneal vessels, which increases urea clearance. Warmed dialyzing solution also contributes to client comfort by preventing cold sensations, but this is a secondary reason for warming the solution. The other options are incorrect.

A female patient complains of abdominal discomfort. Watery stool has been leaking from her rectum. This could be a sign of fecal impaction. diarrhea. bowel incontinence. constipation.

fecal impaction- CORRECT Fecal impaction can be serious. When constipation is not resolved, stool becomes hardened and unable to pass. Liquid stool may pass around the impaction. Patients may complain of feeling bloated; having the urge to push; nausea or vomiting; or not wanting to eat. The impaction may need to be manually removed. Patient education should include increasing liquids and fiber, as well as regular physical activity.

Contraindications for administering an enema include all of the following EXCEPT suspected appendicitis. acute myocardial infarction. hypercalcemia treatment. recent colon surgery.

hypocalcemia treatment- CORRECT An enema may be used to administer sodium polystyrene sulfonate (Kayexalate) for the treatment of hyperkalemia. Kayexalate can be administered either orally or as an enema. Sodium polystyrene sulfonate is not absorbed from the gastrointestinal tract. As the resin passes through the gastrointestinal tract, the resin removes the potassium ions by exchanging them for sodium ions. Recent colon surgery, acute myocardial infarction, and suspected appendicitis are contraindications for administering an enema. With elderly clients, enemas should be used with caution because of their higher risk of hyperphosphatemia, perforation, and sepsis.

A patient with a diagnosis of hepatic coma is admitted to the ICU. The provider orders neomycin 300mg q6h to be administered via the NG tube. The nurse knows the rationale for this drug order is to decrease the pH level in the small intestine. to prevent fulminate sepsis. to decrease the serum ammonia level. to prevent further liver damage.

to decrease the serum ammonia level Although neomycin is generally prescribed to treat infection, it is also prescribed for patients with liver disease to kill intestinal bacteria. The bacteria produce ammonia when breaking down protein. The diseased liver is unable to clear the ammonia, so serum levels build and lead to hepatic encephalopathy. Neomycin does not affect pH or prevent further liver damage.


Ensembles d'études connexes

Chapter 6 DNA replication, repair and Recombination

View Set

Lesson 9.1 - 9.2 Graphing Quadratic Equations

View Set

Fundamentals of Nursing Practice final exam guide

View Set

Chapter 43: Grief and Loss PREP U and questions

View Set

health assessment exam 1 &2 prep u ?

View Set

Ch 12- Pulling It All Together- Dietary Guidelines

View Set